Pediatrics

Réussis tes devoirs et examens dès maintenant avec Quizwiz!

Case Study 5: Care of the Pediatric Patient with Seizures

A young couple enters the ED carrying a sleeping toddler wearing a snowsuit and wrapped in a woolen blanket. The mother anxiously asks, "Please check our baby. He's been sick with a cold, and he feels hot. He doesn't want to eat anything and is so sleepy." The toddler is 18-month-old Tommy Reid. Tommy is taken to a treatment room immediately, placed on a stretcher, and undressed. He opens his eyes for a few seconds and whimpers but generally appears uninterested in his surroundings. He does not resist being handled and moves all limbs spontaneously. When asked, the mother informs you Tommy weighs 11 kg. He feels very warm and appears flushed, with unlaboured respirations at 28 breaths/min. His brachial pulse is strong and regular at 108 bpm, and his capillary refill is less than 2 seconds. Tommy's diaper is dry when removed, and Mrs. Reid is very worried about him becoming chilled. What additional information do you need at this time? a. Complete history b. Temperature c. Blood pressure d. Immunization status You take Tommy's temperature rectally and find that it is 39.4º C. Mr. Reid says he gave Tommy a dose of children's Tylenol drops (Acetaminophen 120 mg) about 2 hours ago and it seemed to have had no effect in making him cooler or decreasing his irritability. What is your interpretation of Tommy's condition? How would you respond to the mother's concerns about her child becoming chilled? "We always undress our patients completely so that the physician can examine them." "Here is a warm blanket to put on him." "Tommy's temperature is 39.4 degrees after Tylenol, so he has a fever. By leaving him undressed for a short time, we can examine him and begin to cool him down a little." "We need to cool Tommy down because a fever this high is very dangerous." You begin your secondary assessment of Tommy, using the mnemonic CIAMPEDS, and obtain the following information: Complaint. Fever, lethargy, decreased oral intake. Immunization / Isolation. Up-to-date, no known contact with infectious diseases. Allergies. No known allergies. Medications. Tommy takes no regular medications. He received one dose of children's Tylenol approximately 2 hours before arrival. Past medical history / Parents' impression. Tommy was a healthy full-term infant, and his growth and development are normal for his age. He has been healthy, with no hospitalizations and only one URTI at 5 months of age. He sees his pediatrician for regular checkups. His parents are very concerned. Events. Tommy's runny nose started yesterday. Today he has been lethargic and irritable when awake. He has felt warm for about 12 hours, but has not had vomiting or diarrhea. Diet / Diapers. Tommy ate well yesterday, with normal fluid intake. He ate cereal this morning and a few bites of banana in the early afternoon, but has taken only a few sips of apple juice since approximately noon. His last wet diaper was in the early afternoon. Symptoms. Flushed, febrile, lethargic. Dr. Lee quickly reviews the history and begins to examine Tommy. After lung and heart auscultation, he examines the child's ears and suddenly Tommy's body stiffens, his eyes appear glazed, and his extremities begin to twitch. Why is this likely to be a febrile seizure? a. Tommy is 18 months old. b. The fever began less than 24 hours ago. c. All prior medical history is negative for neurological disease. d. All of the above. It has been 45 seconds since the onset of seizure activity, and Tommy appears pale, with regular, shallow respirations, a hyperextended neck, and twitching in all extremities. List at least (3) three nursing care priorities for this patient. List at least four (4) findings or observations that must be documented regarding the seizure activity. The physician tells you he would like an IV started immediately. As you gather the equipment, you also obtain Lorazepam (Ativan) IV in anticipation of a physician's order. Upon your return to the room with the IV supplies and medications, Dr. Lee informs you that the seizure has stopped and advises you to hold off on administering any anticonvulsant or sedative medications. As you monitor Tommy closely during this postictal stage, his vital signs are stable, pupils are equally round and reactive to light (PERRL) at 3 mm, and he rouses to loud verbal stimuli by moving all limbs. Tommy's parents move closer to the bedside and begin to stroke his hair. They are a little less anxious, but still shaken by what has happened. Mr. Reid hesitantly asks you, "Aren't you going to give him medicine to keep him from having another seizure?" You explain that Tommy's seizure was probably due to his fever and that febrile seizures rarely require pharmacologic treatment. Before a child who has had a seizure is discharged from the ED, medical staff must determine whether the child has a CNS infection or other disease that must be promptly diagnosed and treated. The child's risk for recurrent seizures or epilepsy must also be assessed. Tommy is monitored for 2 more hours and has not experienced any further seizure activity; he is awake and fully alert. His fever has decreased to 38.2°C (rectal), and he appears less irritable. Tommy is appropriately fearful when strangers approach him and is enjoying a popsicle. On completion of Tommy's examination and diagnostic tests, Dr. Lee has determined that he has bilateral AOM. You administer his first dose of antibiotics, and Dr. Lee provides the parents with a prescription for antibiotics. Tommy's parents should be informed that febrile seizures occur only once in the lifetime of a child who does not have epilepsy. True or false? Counseling anxious parents regarding febrile seizures is an important aspect of your nursing care. The following information should be communicated to parents in order to help them react calmly if a seizure occurs: Reassure parents that most febrile seizures are benign. Children with febrile seizures are at only slightly higher risk than the rest of the population for developing epilepsy (given certain criteria outlined previously). Most febrile seizures are brief and can be managed safely by a caregiver who stays calm. Discuss how to protect the airway and protect the child from physical injury. Explain how to document the type and duration of the seizure. Advise them how to obtain emergency assistance. Advise them that attempts to prevent a fever will not necessarily prevent a subsequent febrile seizure. What additional information do these parents need before leaving the hospital? The Reids leave the hospital tired but very relieved. A wide-eyed Tommy is trying to get out of his father's arms, and Mrs. Reid carefully carries the instructions you gave her. The correct answer is to take Tommy's temperature (b). He feels warm to the touch and is flushed, but this must be validated with an accurate temperature. Significant hyper- or hypothermia is a "red flag." Tommy's airway is clear, and his RR and HR are within normal range; his pulses are strong and regular with brisk capillary refill. His temperature, however, is significantly elevated despite having received Tylenol. His LOC requires further evaluation, as indicated by irritability and lack of responsiveness to a strange environment. His diaper is dry, thus further assessment of intake/output is warranted. There are no immediate life-threatening issues, but he needs a prompt secondary assessment and appropriate treatment. Answer (c) is correct. Explain to Mrs. Reid what you are doing and monitor her reaction to ensure you are not alarming her unnecessarily. Parents should be aware that fevers may indicate serious illness, but they should also know that fevers are most often the body's immune response to a benign condition. Parents must also be advised how to care for them (e.g., acetaminophen or ibuprofen, fan in room, but not directly blowing on the child, do not place the child in a tepid bath). The correct answer is (d). Tommy's age, onset of fever, and previous medical history are all suggestive of a febrile seizure. Nursing care priorities for a seizing child: Ensure a patent airway, suction available Protect the patient from physical injury (e.g., seizure precaution protocols) Maintain constant observation and assessment (e.g., continuous cardiopulmonary monitoring) Establish IV access; correctly calculate and prepare to administer pediatric doses of anticonvulsant medications. Observe and document the following if a child is having a seizure: Time frame Movement Respiratory status Mental status Elimination Oral injuries False. Approximately 30% of children experiencing a simple febrile seizure will have recurrent febrile seizures; however, febrile seizures are generally benign and are not associated with deleterious educational or behavioural effects. Your discharge instructions should also include information about fever control measures, antibiotic therapy, and follow-up care.

Other Pediatric Differences

Children metabolize drugs at different rates in comparison to adults. Dosages for children are more precise and are based on their weight. Not only are medications calculated according to body weight, but the volume of fluids required to maintain hydration is also calculated according to weight. Due to their greater body surface area, children have a larger amount of insensible losses, such as through evaporation. Weight can vary significantly during childhood. An accurate weight should be obtained for every pediatric patient, particularly if the child has vomiting or diarrhea and is not eating or drinking. Weight should be considered a vital sign in children. The parent(s) are often the most accurate at predicting the child's weight. Updated Advanced Paediatric Life Support weight estimation formulae are as follows: - Infants 0 - 12 months: 4 + (0.5 x age in mths) = wt (kg) - Children 1 - 5 years: 8 + (2 x age in yrs) = wt (kg) - Children 6 - 12 years: 7 + (3 x age in yrs) = wt (kg) (Advanced Life Support Group, 2013) Average birth weight = 3.5 kg. ***There are various formulae to estimate weight...keep in mind that their accuracy is dependent upon several factors (i.e., care provider, patient characteristics, patient population).*** The amount of maintenance fluid (intravenous [IV] or oral) required is calculated as follows (4-2-1 Rule): - For the first 10 kg of weight: 4 ml/kg/hour - For the next 10 kg of weight: 2 ml/kg/hour - For every kg of body weight over 20 kg: 1 ml/kg/hour Adults can convert glycogen to glucose when additional fuel is required; however, infants and small children have limited ability to store glycogen as a fuel source. As a result, their blood glucose levels will drop when additional energy is required. Infants younger than 2-3 months of age have immature immune systems and are more susceptible to infection. While some passive immunity is acquired in utero, they have not had sufficient exposure to antigens to trigger their own antibody response. After approximately 3 months of age, they should be able to develop their own protection against infection. The liver is positioned more anterior in children and is less protected by the ribs. The kidneys are more mobile in children, with less protection from subcutaneous fat. Because the osteoblasts of children are still contributing to bone growth, children recover from fractures more quickly than adults. However, injuries at or adjacent to growth plates can slow normal bone growth and development. Children also have more flexible bones than adults, since calcification of the bones is not yet complete. Skeletal trauma (including the spine) may not result in fractures, but may instead cause injury to soft tissues, tendons, or ligaments. Children have proportionally larger and heavier heads in comparison to body size. This contributes to a higher centre of gravity in children. Young children tend to pitch head-first during a fall.

Case Study 1 - Care of the Pediatric Patient with Upper Airway Obstruction

Matt Young, an 18-month-old toddler, is brought to the emergency department (ED) by his mother. Mrs. Young seems frightened and tells you that Matt is "having difficulty with his breathing, has a funny cough, and won't eat or drink." She says Matt has had a runny nose for several days and that he "feels a bit warm." When asked if this has happened before, she says that he has had colds, but has never stopped eating. She says she is worried because he seems worse today; he has had severe coughing spells followed by vomiting. You observe that Matt is pale, drowsy, and reluctant to lie down, but he does not resist undressing. You conduct a thorough respiratory assessment, knowing that airway obstruction is more common in children than adults. Based on your initial assessment using the pediatric assessment triangle (PAT; (appearance, breathing, and circulation), how sick is Matt? Give reasons for your answer. Why is airway obstruction more common in children? Complete the following table. Table 1.5. Signs and Symptoms of Respiratory Distress and Failure in Children You ask to have the emergency physician see this patient immediately. As you continue your primary survey, you observe that Matt has moderate substernal and intercostal retractions and nasal flaring. You ask his mother if she thinks he looks pale, and she tells you he "does seem somewhat pale." His pulse is strong and regular, and capillary refill is 3 seconds. You comment that he seems quiet, and his mother agrees. With regards to disability (neurological) assessment, you document that Matt is alert, but his mother states, "He is more subdued than normal." After his mother removes his clothes, you assess his vital signs (HR 160 bpm, RR 46 breaths/min), noting that there are no indications of injury and his skin is warm to the touch. Which of the following is the nursing priority intervention? Attempting to determine the cause of respiratory distress Repositioning the head to ensure maximum airway patency Examining the posterior pharynx to assess for obstruction Administering oxygen Suggest four (4) potential nursing interventions for administering oxygen to an unwilling child. Interpret Matt's vital signs. With Matt sitting on his mother's lap, you auscultate stridor in all lung fields which is related to a partial upper airway obstruction. Three potential causes of stridor are croup, foreign body aspiration/obstruction, and epiglottitis. Which of the following are indicators of croup? (Choose all that apply) Sudden onset of respiratory distress Harsh, barky cough Rapid onset of symptoms Two days of upper respiratory symptoms Drooling Age You describe your assessment findings while the emergency physician is examining Matt. Although the physician is quite confident the illness is croup, you take this time to review some of the emergency interventions for a patient with epiglottitis. List at least three (3) priority nursing interventions for a patient with epiglottitis. Matt is crying and pulling off his O2 mask. You note his SpO2 has dropped to 89%. Which of the following methods of oxygen delivery is/are appropriate? (Choose all that apply) Partial non-rebreather mask BiPAP Nasal cannula Blow-by oxygen Matt is now using a partial NRB mask and you inform the emergency physician that his SpO2 has increased to 92%. He orders laboratory investigations and Acetaminophen for a rectal temperature of 38.0°C, and makes a tentative diagnosis of croup. Matt continues to have increased WOB. What interventions do you anticipate the physician ordering at this time? Explain why each option is or is not appropriate. Endotracheal intubation Epinephrine (1:1000) 0.01 ml subcutaneously Oral fluids Nebulized epinephrine Establish IV access Dexamethasone 0.6 mg/kg PO What side effects must you observe for following administration of nebulized epinephrine? Differentiate between racemic epinephrine and L-epinephrine. Describe heliox, including its mechanism of action and rationale for use in croup. You reassess Matt after your nursing interventions. He has improved, with a temperature of 37.6 °C (rectal), minimal retractions, RR of 30 breaths/min, warm and pink skin, and mild inspiratory stridor. Mrs. Young is very anxious, thus you provide information about caring for a child with croup. List at least three (3) educational components you would discuss with Mrs. Young. Matt is admitted to the hospital, thus you explain the admitting procedures to Mrs. Young. She continues to ask questions about croup, and you decide it would be appropriate to provide her with a copy of the hospital's home instruction sheets. You listen and reassure Mrs. Young as you prepare to transfer the patient and family to the pediatric ward. Suggested answers The across-the-room assessment indicates that Matt is unwell. His abnormal breathing, pallor, and lack of resistance are all indications that he may be in respiratory distress and require immediate intervention. Obstructions are more common in children than in adults because their airways are smaller and easily occluded by secretions and swelling. Their ribs are more horizontal, decreasing anterior-posterior expansion. They have less respiratory reserve, thus they tire more quickly. Complete the following table. Table 1.5. Signs and Symptoms of Respiratory Distress and Failure in Children General appearance Reluctant to lie down, position themselves to maximize ventilation Assume "tripod" position to facilitate accessory muscle use (sit leaning forward, neck extended, supported by their arms) Assume "sniffing" position to help maintain patent airway (head up with neck extended) Respiratory effort Anxiety, concentration on breathing, use of accessory muscles Retraction occurs as the effort to draw in air against resistance causes a sinking in the soft tissues between bones and cartilage in the thoracic area. Accessory muscle use (intercostals, neck muscles) Nasal flaring occurs as the nostrils dilate to open air passages as much as possible. "Seesaw" abdominal/chest movement (increased use of diaphragm) Rate and depth of breathing Tachypnea or bradypnea Tachypnea may be an early sign of respiratory distress, which progresses to respiratory failure as fatigue sets in (ominous sign) Alteration in LOC LOC is an important indicator of cerebral perfusion Restlessness, anxiety are signs of hypoxia. When a parent indicates that a child is "not acting normal," this should be taken seriously. The correct answer is administering O2 (d). The other answers are incorrect for the following reasons: Improving oxygenation is more important than determining the cause of distress at this time (a). Children in distress normally assume a position to maximize airway opening, so it is not wise to reposition them (b). In a child with sudden onset of fever, drooling, stridor, or cough, examination of the throat (c) is contraindicated until epiglottitis has been ruled out. Initiation of the child's gag reflex upon insertion of a tongue blade, coupled with anxiety and crying caused by the procedure, could result in bronchospasm and complete airway obstruction in a child with epiglottitis. If a child will not tolerate wearing a mask, the following nursing strategies may be attempted: Hold the O2 tubing near the nose and mouth - "blow-by" technique Try role-playing Cut a hole the size of the O2 tubing in the bottom of a styrofoam cup. Thread the O2 tubing through the hole and have the child breathe from the cup. Try using a nasal cannula. Do not exceed a flow rate of 2-4 L/min to avoid irritation of the mucous membranes. Nasal cannula prongs can be trimmed slightly with scissors, as they are often too long for the child's nares and cause discomfort. Matt's HR is elevated at 160 bpm (normal range 98-140 bpm). His RR is elevated at 46 breaths/min, with the normal range being 22-37 breaths/min. His temperature, BP, and SpO2 have not been assessed as of yet, but his skin is pale and warm to the touch. The correct answers are (b), (d), and (f). A barky cough is a common finding associated with croup. Croup is a viral illness, with onset of upper respiratory symptoms over 2-3 days. It is most often seen in children < 7 yrs of age, with peak occurrence between 6 mths-3 yrs. Answers (a) and (e) are suggestive of a foreign body obstruction. Because the obstruction is caused by a mechanical object rather than an infectious process, the child will usually be afebrile. (Caution: Keep in mind that a child with a viral illness may also have aspirated a small object.) Answers (c) and (e) may indicate epiglottitis. This bacterial infection is less common today due to the Hib vaccine; however, other pathogens are known to cause epiglottitis as well. It is most often seen in children aged 2-5 yrs, with sudden onset of fever, drooling, and dysphagia. Epiglottitis is a life-threatening condition, thus may rapidly progress to complete airway obstruction and respiratory arrest without intervention. A patient with epiglottitis is always at risk for complete airway obstruction. Nursing interventions include maintenance of the airway, preparation for the possibility of intubation, and assessment of perfusion status. Allow the child to sit in any position of comfort that maintains airway patency, and apply O2 via "blow-by" Maintain constant observation and avoid situations that may upset the child and subsequently increase airway obstruction, such as removing the child from his/ her parents or performing invasive procedures (e.g., IV initiation, chest auscultation). Immediately prepare for transfer to the operating room (OR), where an IV, sedation, intubation, and antibiotics will be initiated. Prepare emergency airway equipment for transport to the OR. Any means of O2 delivery that the child is comfortable with would be acceptable. It may be helpful for the parents to assist in O2 delivery because the child is likely more trustful of a familiar person. Interventions for a child with croup Endotracheal intubation. No. Endotracheal intubation is not necessary at the moment, but it is important to prepare for a complete airway obstruction. Epinephrine (1:1000) 0.01 ml subcutaneous (SC). No. Although epinephrine given SC will facilitate opening of air passages, it is not as effective as nebulized epinephrine for croup. Oral fluids. No. While oral fluids should be encouraged to help thin secretions in the lungs and prevent obstruction, it is important to ensure Matt's condition is stable and improving first. Nebulized epinephrine. Yes. An aerosol treatment with racemic or L-epinephrine is administered to open narrow air passages and increase ventilation. IV access. No. This is an inappropriately invasive intervention at this time, as the pain and anxiety of IV initiation may further compromise Matt's airway. However, if he is severely dehydrated and/or in severe respiratory distress, the benefits would outweigh the risks. Dexamethasone 0.6 mg/kg PO. Yes. Dexamethasone (corticosteroid) PO (or IM) would reduce subglottic edema and inflammation. Antibiotics should be reserved for bacterial complications, such as acute otitis media (AOM). The effects of nebulized epinephrine as a short-acting bronchodilator are very similar to those of other routes of epinephrine - mucosal vasoconstriction, decreased subglottic edema, tachycardia, hypertension (HTN), tremors, and restlessness. Racemic epinephrine consists of a mixture of isomers, of which L-Epinephrine (Epinephrine, Adrenaline) is the active component. Racemic epinephrine is no longer readily available in North America, although some institutions still use it to get rid of their stock. L-Epinephrine 1:1000 has been shown to be as equally effective as the racemic form for the treatment of croup (Toward Optimized Practice [TOP], 2008). Heliox is a mixture of helium (He) and O2 (He 70:30 O2 blend). Helium is a lower density gas that is thought to decrease turbulent airflow in narrowed airway passages. Research has shown that it may be of benefit to children with moderate-severe croup if they have received corticosteroid therapy; however, it is not recommended for routine use in the treatment of croup. The higher concentration of He in the 70:30 blend limits the FiO2 provided to the patient, which may exacerbate hypoxemia and hypoxia. It must also be administered by trained personnel (TOP, 2008). The following educational topics are appropriate for the parents of children with croup: Causes of croup Home care and rehydration measures Signs and symptoms of moderate and severe respiratory distress When to return to the ED

Case Study 6: Care of the Pediatric Patient with Meningitis

Mrs. Baxter brings Erin, a 7-week-old infant, to the ED because she seems "sleepy." You ask Mrs. Baxter to remove the blankets. Erin wakes up, cries weakly, and quickly falls asleep again. You observe that she is pale and breathing rapidly. How sick is this patient? When you assess Erin's ABCDEs, you find rapid, unlaboured respirations, mottled extremities, a capillary refill of 3 seconds, and equal peripheral and central pulses. She feels warm to touch, opens her eyes and cries when aroused, and is moving all limbs. The mother's efforts to comfort Erin make her cry more vigorously. Her pupils are PERRL at 3 mm, and vital signs are as follows: HR 190 bpm RR 68 breaths/min BP 80 mmHg (systolic) Capillary refill 3 secs SpO2 93% on RA Indicate whether the above vital signs are normal, high, or low for a child Erin's age: a. Heart rate b. Respiratory rate c. Systolic blood pressure d. Capillary refill e. Oxygen saturation What is Erin's Glasgow Coma Scale score? You ask Mrs. Baxter if Erin has a fever, and she tells you she didn't take her temperature because she doesn't know how. The completely undressed infant is placed on the scale and weighs 4 kg. Her temperature it is 39.4 °C (rectal), and as you take it you teach Mrs. Baxter how to take a baby's temperature. An axillary temperature is easy, noninvasive and safe, and can be taken in all age groups; however, it is not a reliable assessment of core temperature in children. Its accuracy and reliability is not only dependent upon precise placement over the axillary artery, but also environmental conditions (Leduc & Woods, 2000/2015). Axillary measurement is approximately 1°C lower than rectal, and 0.5°C lower than oral route. A tympanic temperature also has the advantage of being obtained quickly. According to the Canadian Pediatric Society (CPS), In contrast with other sites of temperature measurement, the TM's [tympanic membrane's] blood supply is very similar in temperature and location to the blood bathing the hypothalamus, the site of the body's thermoregulatory centre. It is, therefore, an ideal location for core temperature estimation." (Leduc & Woods, 2000/2015, para. 10). Despite its advantages, the CPS acknowledges that its accuracy is adversely affected by several factors: incorrect technique (e.g., failing to straighten the ear canal), inappropriate probe size, ear canal structure, and manufacture design. However, tympanic measurement is not significantly affected by crying, AOM, or cerumen. Oral thermometry measures the temperature of (sub)lingual arteries. It would not be an appropriate method for a 7-week-old infant, and is generally reserved for older and more compliant children. Its accuracy is dependent upon a firm mouth seal and remaining under the tongue for 3-4 minutes, and is generally regarded as being between that of axillary and rectal measurement (roughly 0.5°C lower than rectal). Parents may also inquire about pacifier thermometers; they should be informed that they are also not as accurate as rectal measurement (Leduc & Woods, 2000/2015). Temporal artery temperature measurement is a relatively new method of estimating core temperature. It has been shown to be more accurate than the tympanic method, and is less invasive than rectal measurement. It is generally regarded as an effective febrile 'screening' tool; however, it should not be relied upon for the assessment of a seriously ill child (less accurate than rectal) (Leduc & Woods, 2000/2015). Rectal temperature is generally considered to be the 'gold standard' of reliable measurement of core body temperature, but its accuracy is known to be affected by local blood supply and the presence of stool. There may also be a delay in the change in rectal temperature following an increase or decrease in core temperature. This method is also not without risk: it must be used cautiously in children less than 2 years of age due to potential bowel perforation, and is contraindicated in patients who are immunocompromised or have bowel disease. Strict decontamination techniques must be adhered to in order to prevent the spread of pathogens, and parents are often not comfortable with this method (Leduc & Woods, 2000/2015). General guidelines for the depth of probe insertion are as follows: ½ inch for newborns; ¾ inch for infants; and 1 inch for preschoolers/older children (Potts & Mandleco, 2012) Acetaminophen is the antipyretic most widely used for fever therapy (dose = 10-15 mg/kg). Ibuprofen is also commonly used to lower core temperature (dose = 10 mg/kg). Some physicians do not treat fevers, considering them a healthy and normal response, but they may use Acetaminophen or Ibuprofen as a comfort measure. Cool or tepid baths should be avoided because they produce shivering and crying, which may increase body temperature and metabolic rate. Aspirin is contraindicated in the pediatric population due to the risk of developing Reye's syndrome, thus is not recommended for fever therapy. While placing Erin on the cardiopulmonary monitor, you obtain the remainder of the history from her mother. Erin has no known allergies, takes no medications, and was a full-term baby born without complications. She has not yet received any immunizations. Erin took only a few sips of formula an hour ago and has not had a wet diaper for 12 hours. Her last bowel movement (LBM) was yesterday. Although she seemed fussy last evening, Mrs. Baxter was not alarmed until Erin did not awaken for her mid-morning feeding. The most worrisome information from Erin's history is that she: a. refused to feed an hour ago. b. has had no stool for 24 hours. c. has not had a wet diaper for 12 hours. d. has not had any immunizations. Erin continues to be lethargic, and her mottling and capillary refill (4 seconds) are worsening. The emergency physician examines Erin and informs Mrs. Baxter that her baby seems quite ill and will have to have blood drawn and a urine sample collected to assess for infection. She also tells Mrs. Baxter that Erin will need a lumbar puncture (LP) to test for meningitis, and, depending on the results, may be admitted to hospital for observation and IV antibiotics. Erin is placed on isolation precautions, and you explain the gowning, masking, and hand-washing procedures to her mother. List at least four (4) clinical manifestations of meningitis in an infant. Preparation for a septic workup begins. CBC is obtained as a nonspecific indicator of infection, and electrolytes are drawn as a baseline. Liver function tests (LFTs), BUN, Cr, and coagulation studies are also obtained if indicated by the child's history. Blood glucose is drawn to identify hypoglycemia, a common reaction to stress and septicemia, and to compare it to the cerebrospinal fluid (CSF) glucose level. Blood, urine, and CSF are cultured, and the CSF is examined for cell count, protein, glucose, and Gram stain. A CXR is obtained to identify any possible pulmonary inflammation or infection. For a septic workup, you would require all of the equipment listed below: Infants may quickly become hypothermic in the ED, thus a heat lamp or warming bed will keep body temperature stable while they are undressed for medical and nursing interventions. Vacutainer and small-gauge needle for drawing blood from an infant's hand, foot, or antecubital fossa. Sterile feeding tube (5 Fr) to obtain a catheter urine specimen. Urine bag (U-bag) collections are dependent on voids; bacteria in the skin folds of genitalia may contaminate the sample, thus are not recommended for the collection of a urine culture. IV equipment LP tray and related supplies (e.g., spinal needles, betadine, sterile gloves) Mrs. Baxter asks why the doctor thinks Erin might have meningitis, and you tell her it is because of her baby's fever and lethargy. She wonders what will happen if something else is making her baby sick. You reassure Mrs. Baxter and encourage her to take one step at a time. You help her to understand that once the testing is complete, and the source of infection is determined, you will have more information with which to answer her questions (it is important not to give false reassurances). Consider finding someone who can provide emotional support, such as a family member, friend, social worker, or pastoral caregiver. It may also be helpful to encourage her to remain calm for Erin. An IV is initiated, and the emergency physician orders NS at a maintenance rate (ml/hr) appropriate for Erin's weight. As previously discussed, calculations are based on the child's body weight in kilograms, and follow the 4/2/1/ rule. 4 ml/kg/hr for the first 10 kg + 2 ml/kg/hr for the second 10 kg + 1 ml/kg/hr for every kg over 20 kg Volumetric infusion devices (e.g., Buretrol) and pumps must be used to deliver fluids and medications at precise dosages and rates. Accurate intakes and outputs must be analyzed and recorded. Given Erin's weight of 4 kg, her hourly IV maintenance rate would be: a. 10 ml/hr. b. 16 ml/hr. c. 14 ml/hr. d. 21 ml/hr. You follow the LP procedural policy by assessing and documenting a full set of vital signs; positioning the baby curled on her side; monitoring her airway, breathing, and colour; reassuring her mother and explaining the procedure; and conducting a post-procedure assessment. The CSF results are as follows, with normal findings (in parentheses): Appearance Cloudy (Normal: crystal clear, colourless) WBC 250 cells/mm3 (Normal: Adults = 0-5 cells/mm3; Neonates = 0-30 cells/mm3) RBC 1 cell/mm3 (Normal = up to 5 cells/mm3) Total protein 90 mg/dL (Normal: Adults = 15-45 mg/dL; Neonates = 15-100 mg/dL) Glucose 1.4 mmol/L (Normal: Adults = 2.2-3.9 mmol/L; Neonates = 3.3-4.4 mmol/L) Gram stain Moderate polymorphonucleocytes, few gram-positive cocci (Normal = no organisms present) With a diagnosis of bacterial meningitis, Erin is admitted for IV antibiotic therapy, fluid restriction (to prevent increased ICP), isolation, and observation. The physician asks you to decrease Erin's IV rate to ¾ maintenance and administer the first dose of antibiotics prior to transfer. Erin's mother asks what causes meningitis. What are the major causes of bacterial meningitis in infants and children Describe the composition of Erin's CSF with regards to bacterial meningitis, and provide a rationale for each component. The physician orders Ampicillin 200 mg IV for Erin's first dose of antibiotics. She says that she is basing the dosage on 200 mg/kg/day, to be administered every 4 hours. Erin weighs 4 kg. Is this the correct dose for her? Why or why not? Erin remains stable throughout her emergency stay. The nurse receiving Erin on the pediatric unit asks what she should watch for, and you inform her that the following serious complications can occur due to systemic infections, such as sepsis and meningitis: Cerebral edema (***greatest risk for Erin***) Septic shock Cerebral abscess Seizures DIC Syndrome of inappropriate antidiuretic hormone (SIADH) What is an early sign of increasing intracranial pressure? (Choose all that apply). a. Change in heart rate b. Bulging fontanel c. Change in blood pressure d. Cushing's triad (hypertension with a widened pulse pressure, bradycardia, and abnormal respirations) On day 4 of Erin's hospital stay, you stop by to visit and her nurse informs you that Erin's blood and CSF have cultured group B streptococci. She is currently afebrile, taking oral fluids, and continuing to receive antibiotic therapy. Suggested answers You determine that Erin is emergently sick, based on her age, rapid breathing, weak cry, lethargy, and pallor. Vital signs interpretation a. HR 190 bpm: tachycardia, given her lethargy (upper limit 160 bpm if sleepy) b. RR 68 breaths/min: tachypnea (upper limit 53 breaths/min) c. SBP 80 mmHg: within normal range (lower limit 72 mmHg) d. Capillary refill 3 secs: delayed (should be ≤ 2 secs) e. SpO2 93% on RA: hypoxia (should be ≥ 95% on RA) Fever, volume depletion, and pain can cause tachycardia; tachypnea may also be due to fever or the body's attempt to compensate for metabolic acidosis. Recall that hypotension is a late sign of shock in children/infants, thus Erin is demonstrating signs of compensated shock: tachycardia, peripheral vasoconstriction (delayed capillary refill), irritability, and hypoxia. Erin's GCS score is 13/15: she opens her eyes and moves all limbs spontaneously, but her cry is weak and worsens when she is comforted. The correct answer is (c). Decreased urine output is indicative of decreased renal perfusion, which correlates with decreased CO and peripheral perfusion. Clinical manifestations of meningitis in infants Irritability, crying Lethargy Bulging fontanel Seizures Temperature instability/fever Stiff neck (3-24 mths of age) Apnea, cyanosis Petechiae, purpura Vomiting Poor feeding The correct answer is (b) 16 ml/hr. Group B streptococcus, Streptococcus pneumoniae and Neisseria meningitidis are the leading causes of bacterial meningitis in infants and children (although S. pneumoniae and N. meningitidis incidence rates have decreased significantly in immunized populations). Composition of Erin's CSF in bacterial meningitis, with rationales Cloudy: collection of WBCs, infection WBC count: elevated, indicative of inflammatory/immune response Mostly polymorphonuclear (PMN) cells, released during infection RBC: typically indicative of a 'bloody tap', 'traumatic tap' Usually not pathological, unless subarachnoid hemorrhage suspected Protein: increased, indicative of blood-brain-barrier (BBB) injury (increased permeability) Glucose: decreased (rationale remains controversial) Initially believed to be due to microorganism's consumption of glucose; however, now thought to be due to altered glucose transport across BBB and glycolysis dysfunction CSF glucose level is normally about 60% (⅔) of serum level Gram stain: positive for specific organism The dose is not correct. The correct 1st dose of Ampicillin is 133 mg, derived from the following calculations: 200 mg x 4 kg (Erin's weight) = 800 mg/24 hrs 800 mg is divided by 6 (the number of doses per 24 hrs) = 133.3 mg The correct answer is (b). The anterior fontanel is still open in a 7-week-old infant; cerebral edema will manifest as an observable and palpable bulging fontanel. Change in HR (a) is less specific, and changes in BP and Cushing's triad (c, d) are late manifestations of cerebral edema.

Pediatric Tables

Normal Pediatric Vital Signs Age Neonate Infant (1-12 mths) Toddler (1-2 yrs) Preschooler (3-5 yrs) School-age child (6-11 yrs) Adolescent (12-15 yrs) Normal Heart Rates (beats/minute) Normal Respiratory Rates (breaths/minute) 30-60 30-53 22-37 20-28 18-25 12-20 Normal Blood Pressure (mmHg) Heart Rate (Awake) 100-205 100-180 98-140 80-120 75-118 60-100 Heart Rate (Asleep) 90-160 90-160 80-120 65-100 58-90 50-90 Systolic 67-84 72-104 86-106 89-112 97-115 Diastolic 35-53 37-56 42-63 46-72 57-76 Preadolescent (10-11 yrs) 102-120 61-80 110-131 64-83 (AHA, 2016) Pediatric Glasgow Coma Scale (GCS) Score Best Eye Opening Child (≥ 1 yr) Infant (≥ 1 yr) 4 Spontaneously Spontaneously 3 To verbal command 2 To pain 1 No response Child (≥ 1 yr) 6 Obeys 5 Localizes pain 4 Flexion - withdrawal 3 Flexion - abnormal (decorticate rigidity) 2 Extension - decerebrate rigidity 1 No response To shout, speech To pain No response Best Motor Response Infant (≥ 1 yr) Spontaneous movements Withdraws to touch Flexion - appropriate withdraw Flexion - abnormal (decorticate rigidity) Extension - decerebrate rigidity No response > 5 years 5 Oriented, converses 4 Disoriented, converses 3 Inappropriate words 2 Incomprehensible sounds 1 No response Best Verbal Response 2-5 years Appropriate words/ phrases Inappropriate words Persistent crying, screaming Moans, grunts to pain No response 0-2 years Cries appropriately, smiles, coos Cries and is consolable Persistent inappropriate crying, screaming Moans, grunts to pain No response * If patient is intubated, unconscious, or preverbal, the most important part of this scale is motor response (AHA, 2016, p. 57; ENA, 2014, p. 75) Indicators of Illness in Infants Indicator Normal Moderately ill Severely ill Colour of mucous membranes, nail beds, palms of hands, soles of feet Pink Pale Cyanotic Hydration status Moist mucous membranes Good skin turgor Flat fontanel Light, amber- coloured urine Sticky, pasty mucous membranes Slightly doughy skin turgor Fontanel slightly sunken Dark-coloured urine Dry mucous membranes Tenting skin turgor Very sunken fontanel and eyes, anuria Response to stimulation Arouses easily, then stays awake and alert Arouses with repeated gentle stimulation, then falls back to sleep quickly if unstimulated Arouses only with noxious stimulation or does not arouse at all Behaviour Unchanged Fussy, but consolable Irritable, inconsolable if awake Cry Unchanged Whimpers, sobs, or whines High-pitched and screeching or weak and moaning End-Organ Perfusion Assessment Integument Capillary refill Comparative pulses Skin temperature and colour (Central/Peripheral perfusion status) Capillary refill is normally ≤ 2 seconds, prolongation is a sensitive and early indicator. Comparing central and peripheral pulses gives a qualitative assessment of perfusion status. Compromised perfusion may produce colour or temperature variation in extremities. Mottling, pallor, and peripheral cyanosis indicate poor perfusion. Gray or ashen colour indicates vasoconstriction and poor perfusion. Brain Alteration in LOC (Hypoxia) Lack of adequate perfusion to the brain produces changes in LOC that are valuable indicators of cerebral hypoxia or ischemia. These changes may include: • Parent's statement that the child is "not acting normally." • Child's failure to recognize caretakers. • Confusion, irritability, lack of consolability Kidneys Urine output (Renal perfusion) Infants/Young children: 1.5 - 2 ml/kg/hr • Diapers: 1 g increase in diaper weight = 1 ml of urine • An infant should have a wet diaper every 2 hours, toddler every 4 hours. Older children/Adolescents: 1 ml/kg/hr (AHA,

Shock

Shock is a state of inadequate tissue perfusion, resulting in anaerobic metabolism and organ dysfunction or failure if oxygenation is not restored. Oxygen delivery to the tissues is dependent upon three factors: Sufficient O2 content in the blood Adequate cardiac output (CO; blood flow to tissues) Appropriate distribution of blood flow to tissues (AHA, 2016, Part 8) Shock is categorized according to etiology and effect(s) on body systems: hypovolemic, cardiogenic, distributive, or obstructive. Hypovolemic shock results from a decrease in circulating blood volume due to dehydration or hemorrhage, and is the most common type in children. Cardiogenic shock is due to inadequate cardiac contractility or myocardial dysfunction. Distributive shock is characterized by central nervous system (CNS) and peripheral vascular system disconnect, and may be due to anaphylactic, neurogenic, or septic causes. Obstructive shock is the result of circulatory mechanical obstruction, such as cardiac tamponade or pulmonary embolism (PE). Regardless of the etiology, "...the common denominator of shock...is inadequate delivery of oxygen and nutrients necessary for normal tissue and cellular function" (ENA, 2014, p. 227). Children have less total blood volume than adults; therefore, the loss of the same amount of fluid represents a much larger percentage of a child's total volume. Proportions of circulating blood volume are as follows: Infant: 90 ml/kg Child: 80 ml/kg Adult: 70 ml/kg Causes of hypovolemic shock in the pediatric population include vomiting, diarrhea, inadequate hydration, hemorrhage, trauma, fractures, burns, and sepsis. Children can lose up to 25-30% of their total blood volume and still remain normotensive through compensatory mechanisms, such as increasing systemic vascular resistance (SVR), HR, and shifting CO to vital organs. Pathophysiology and Clinical Presentation The etiology of shock can differ, but the body's response is the same; the pediatric patient's ability to compensate for decreased perfusion, however, is underdeveloped. The pediatric response to shock can be described as follows: Compensated shock (increased workload and O2 consumption...slippery slope) Sympathetic nervous system (SNS) stimulation Recall that CO = HR x stroke volume (SV). Unlike adults, children are not able to increase SV due to a smaller ventricle and shorter myocardial fibers, thus they increase HR in an attempt to increase CO (early sign of compensated shock). Baroreceptors sense decreased arterial pressure. Increased cardiac contractility (attempt to completely empty ventricles) Increased venous smooth muscle tone (attempt to increase venous return) Normotensive or narrow pulse pressure (increased diastolic blood pressure [DBP]) Decreased peripheral perfusion (cool, pale, mottled, diaphoretic, delayed capillary refill, weak peripheral pulses) Blood shunted to vital organs (hypothermia; oliguria (< 1 ml/kg/hr); vomiting, ileus) Irritability, lethargy (cerebral hypoxia) Renin-angiotensin-aldosterone system (RAAS) activation Renin: released due to decreased renal blood flow, SNS Angiotensin I-II: vasoconstriction, Aldosterone Aldosterone: sodium (Na+), water resorption Antidiuretic hormone (ADH, vasopressin) Released from posterior pituitary in response to increased plasma osmolality, hypovolemia, hypotension Increases renal water permeability, resorption Vasoconstriction ***Any drop in BP indicates depletion of compensatory resources, thus assessment of end-organ perfusion is paramount in recognizing early or impending shock.*** Decompensated shock (associated with systolic hypotension) Thresholds for hypotension are defined by systolic blood pressure (SBP) and age: Neonate: SBP < 60 mm Hg Infant: SBP < 70 mm Hg Children 1-10 yrs: SBP < 70 mm Hg + (age in yrs x 2) Children > 10 yrs: SBP < 90 mm Hg Fluid shifts from intravascular to interstitial space, resulting in edema Coagulation cascade activation, resulting in disseminated intravascular coagulation (DIC); petechiae, purpura, hemorrhage Uncontrolled inflammatory response, resulting in cellular acidosis/cellular death which compounds the problem with further release of inflammatory mediators Pulmonary tissue hypoxia, resulting in acute respiratory distress syndrome (ARDS) Multiple organ dysfunction syndrome (MODS) due to compromised end-organ perfusion Altered liver, renal function (microemboli) Systemic spread of bowel pathogens (e.g., Escherichia coli) Bowel necrosis....leading to sepsis, septic shock Management Management of hypovolemic shock involves addressing the etiology (e.g., hemorrhage, dehydration), and follows the systematic initial assessment approach as outlined previously (i.e., ABCDE[FG]-HI). Generally speaking, the treatment of hypovolemic shock in the pediatric population includes the following interventions: Airway maintenance, intubation if decreased LOC 100% high-flow O2 via mask with reservoir, with assisted ventilation if needed Cardiopulmonary resuscitation (CPR) if HR < 60 bpm, poor perfusion Control bleeding Vascular access (large bore x 2, intraosseous [IO] if compromised peripheral perfusion)...see IO process below Volume replacement 20 ml/kg of warmed isotonic crystalloid solution over 5-20 mins; assess response, repeat as needed (may need 4-5 boluses as only 25% remains in intravascular space) 3 ml to 1 ml Rule: in hemorrhagic shock, give about 3 ml of isotonic crystalloid for every 1 ml of blood lost (AHA, 2016, p. 218) Use smaller fluid boluses of 5-10 ml/kg over 10-20 mins in cardiogenic shock Normal saline (NS) or lactated Ringer's (LR) recommended Colloids, packed red blood cells (PRBCs; 10 ml/kg) also options Thermoregulation Metabolic/Electrolyte correction Glucose Infant/Child: D25W 2-4 ml/kg IV/IO If only D50W available, mix equal parts dextrose/sterile water (e.g., D25W 20 ml = D50W 10 ml + sterile water 10 ml) Neonate: D10W 2 ml/kg IV Sodium bicarbonate (NaHCO3) for severe acidosis Children = 1 mEq/kg of 8.4% solution Infants < 3 mths = use 4.2% solution Serial/Ongoing assessments Serial reassessment of primary survey (ABCDE-FG) and interventions Laboratory values (e.g., CBC/Diff, glucose/electrolytes, type & screen, ABGs, lactate, urinalysis) Capnography Electrocardiogram (ECG) Echocardiogram (ECHO) Family involvement (ENA, 2014, pp. 227-240) Once the child is stabilized, maintenance IV fluid therapy is administered according to the following guidelines, often referred to as the 4/2/1 rule: 4 ml/kg/hr for the first 10 kg + 2 ml/kg/hr for the second 10 kg + 1 ml/kg/hr for every kg over 20 kg An alternative to IV access in the child is insertion of an IO needle. Any medication administered via IV can also be given IO; fluid absorption rates via both routes are also comparable. The proximal tibia is the most common site used, and is described below. Position patient supine, knee flexed (support leg on rolled blanket) Stabilize lower leg by placing hand distal to knee (do not place hand under knee) Clean an area approximately 2 cm below the tibial tuberosity (below the growth plate [epiphysis]), medial aspect of proximal tibia Other potential sites include distal femur, distal tibia above medial malleolus, proximal humerus, anterior superior iliac spine (see institutional protocols) Insert IO needle at 90º angle to the bone, tilt it away from epiphysis Advance needle until loss of resistance (needle will stand unsupported), remove stylet, attach syringe, attempt to aspirate marrow to verify placement (absence of marrow does not necessarily mean incorrect placement); send marrow for laboratory studies, if possible Establish crystalloid infusion (may not infuse via gravity), use 3-way stopcock Firmly secure IO site Observe for infiltration/extravasation at site, and posterior surface (ensure no through-and-through penetration) (ENA, 2014, pp. 126-127) Visit the following website to observe insertion of an IO needle: http://www.youtube.com/watch?v=JVbPANbgxQM&feature=related

History

The history may be obtained from the parent or caregiver, child, or both. It is an important piece of the puzzle that assists in analyzing assessment findings and making decisions. The CIAMPEDS mnemonic describes the basic components of a pediatric history. C = Chief complaint Reason for visit to the ED Duration of complaint (e.g., fever for three days) I = Immunization - Isolation Current immunization status Document lack of immunization, reasons (e.g., cultural beliefs) Assess possible exposure to communicable diseases Establish isolation for potentially infectious child Establish reverse isolation for immunosuppressed child A = Allergies Evaluation and documentation of previous reactions to medications, foods, products (e.g., latex), environmental allergens M = Medications Current medication regime (prescribed, over-the-counter [OTC], herbal, dietary supplements) Evaluate dose administered, time of last dose, duration of use P = Past medical history Prior illnesses, injuries, hospitalizations, surgeries, chronic physical and psychiatric illnesses Evaluate alcohol, tobacco, substance use (if appropriate) Infant Complications during pregnancy or delivery Gestational age, birth weight Number of days in hospital after birth Menarchal female Date and description of last menstrual period (LMP) Sexually active Type of birth control used, barrier protection Prior treatment for sexually transmitted infections (STIs) Gravida (# of pregnancies), Para (# of deliveries > 20 weeks) Parent's impression of child's condition Identification of the primary caregiver Consideration of cultural differences or special health care needs that may affect the caregiver's impressions Evaluation of the caregiver's concerns and observations of child's condition E = Events surrounding the illness or injury Illness Length of illness Date of onset Sequence of symptoms Exposure to others with similar symptoms Treatment provided prior to ED visit Injury Time/Date injury occurred, description of circumstances, witnessed/ unwitnessed MIST (pre-hospital history from prehospital providers) M = Mechanism of injury (MOI), including use of protective devices (e.g., seat belts, helmets) I = Injuries sustained/suspected S = Signs and symptoms (in the field) T = Treatment (in the field) D = Diet Assessment of recent oral intake, changes in eating/drinking patterns Time of last meal/fluid intake Usual diet: breast milk, type of formula, solid foods Diet appropriate for age and developmental level, cultural differences, special or restricted items Diapers Time of the last void Frequency, odour, and colour of the urine over last 24 hours Time of last bowel movement (LBM) Changes, colour, consistency, and frequency S = Symptoms associated with the illness or injury Identification of symptoms since the onset of the illness or injury Social and family histories should also be included. It is important to observe how the parent and child are interacting while taking the history. The HEADS mnemonic can be used to assess health risks for an adolescent: H = Home E = Education and Eating A = Activities and Affect D = Drugs S = Suicide ideations/attempts and Sexual history

Case Study 2

Tommy, a 20-month-old toddler, is carried by his mother, screaming and clinging, into the ED. She says he woke frequently during the night with "inconsolable" crying and thinks he is seriously ill because he "never cried like this before." The initial impression is that his airway is clear (crying), his nose is running but there is no nasal flaring, and his breathing is non-laboured. His skin is pink, and mucous membranes are also pink and moist. Tommy's irritability leads the triage nurse to triage him as emergently ill (Level II). Give reasons why you disagree with this assessment. Continuing with the primary survey, you ascertain that Tommy has no immediate airway, breathing, circulation, or disability (neurologic) concerns, and you attempt to obtain some vital signs. How do you gain Tommy's cooperation with obtaining his vital signs? (Choose all that apply.) a. Ask his mother to step outside b. Allow his mother to hold him c. Reason with him d. Offer a treat if he cooperates e. Distract him with a stethoscope or toy f. Tell Tommy that "big boys don't cry" Tommy's vital signs are as follows: HR 126 bpm RR 30 breaths/min Temp 38.9ºC (rectal) SpO2 95% on RA BP Unable to obtain due to fighting/screaming You ask his mother to undress Tommy. You note that his capillary refill is brisk, peripheral and central pulses are strong, and he has no rashes; however, his skin does feel hot. During your history-taking, you ask how long he has had this fever; his mother tells you he has felt hot for the last 2 days. She says he is not on any medications, has not had any Acetaminophen, and is not allergic to any medication. What additional information do you need from the history? You ask Tommy's mom to put a hospital gown on him. When you return to the cubicle, he screams. What responses are appropriate? (Choose all that apply). a. Sit down as you get more history b. Use direct eye contact when you speak to Tommy c. Listen attentively to concerns d. Ask Tommy to tell you why he doesn't feel well e. Communicate indirectly with Tommy by speaking to his mother Tommy's tears subside, and you are able to elicit more history. Tommy has had a fever, diarrhea, and irritability for 2 days. He has had four loose (not watery) stools in the past 24 hours, soaking half of his diaper each time. On the advice of the pediatrician, the parents have been withholding solid food and giving him clear fluids. During the night, he drank 8 oz of Gatorade. Tommy is adopted, thus additional medical and/or family history is unknown. His mother is frightened and feels helpless. She confides that she has waited so long for a child that she does not know what she will do if anything happens to Tommy. The appropriate response to Tommy's mother's disclosure is to: a. tell her you are very concerned and ask if it would be helpful to see a counsellor. b. call a social worker to speak to her. c. listen to her concerns and reassure her appropriately. d. ask Tommy's father to come to the hospital to help in calming her. You complete your secondary survey and explain that the emergency physician will be in shortly to see Tommy. Based on the history and physical assessment information, do you think Tommy is dehydrated? Give reasons for your answer. Tommy curls up in his mother's lap and begins to suck his thumb as you prepare to leave the cubicle. His mother is very anxious and asks several times if the physician can come now because "My child has a fever so he must be sick." How do you respond to this statement? The emergency physician examines Tommy in his mother's arms. Tommy is diagnosed with bilateral acute otitis media (AOM) and Amoxicillin is prescribed. The correct method for giving Tommy the medication is to: a. hold his nose so he will open his mouth. b. use a syringe without a needle and slowly distribute the medication into the posterior portion of his cheek. c. tell Tommy the medicine is candy and good for him. d. disguise the medicine in food. Once Tommy's mother has been shown how to give the medication, she calms down and asks more appropriate questions. She is given written discharge instructions regarding fever, URTIs, and AOM, and told to return if she has any concerns. You do not believe Tommy is emergently ill for the following reasons: His airway is clear. His breathing is non-laboured. His colour is pink, with moist mucous membranes. He is alert and frightened (normal for a toddler). Allowing Tommy to remain in his mother's arms (b) fosters security. Providing distraction (e) helps to gain his cooperation. It would not be helpful to ask his mother to step outside (a) or to reason with Tommy or offer him a treat for cooperating (c and d). Telling Tommy that "big boys don't cry" (f) may foster guilt if he cannot comply and is not recommended for any age group. We still need information about Tommy's immunization status, past medical history, events leading up to this and any possible exposures, intake and output, and more details about the symptoms. The correct answers are (a), (c), and (e). A friendly, warm, unhurried atmosphere (a) encourages positive responses; it is helpful to allow concerns to be expressed (c); and indirectly communicating with a toddler while speaking to a parent (e) is often a useful technique. Frightened toddlers feel threatened by direct eye contact (b), and Tommy's cognitive abilities have not developed to a point where he can tell you why he does not feel well (d). Following are additional hints for gaining cooperation from a toddler: Allow the child to remain in the parent's lap Speak to the parent initially Use games or distractions, such as blowing out the light, looking for Cheerios Adapt the examination to the situation and be observant Offer choices when possible The correct answer is (c). Allowing the mother to voice concerns and providing positive reinforcement about her ability to care for Tommy may help relieve her fears. Ask her about any feelings of responsibility so she can be reassured appropriately. Tommy does not appear to be dehydrated for the following reasons: he is still producing tears; he has moist mucous membranes, brisk capillary refill, strong peripheral pulses, and normal vital signs; and he is alert and drinking fluids well. Note: Although Tommy's mother has the best of intentions in attempting to keep him hydrated with Gatorade, she should be advised that Gatorade has no health benefits for children and is discouraged by most pediatricians. Energy drinks tend to be high in sugar, caffeine, and other amino acids that may be harmful to children. Pedialyte, or other preparations that are specifically designed for the replacement of fluids/electrolytes in children, is the recommended rehydration regimen. You inform Tommy's mother that he does not appear to be seriously ill and is growing and developing normally. You listen to her concerns and give positive feedback regarding her abilities, as she is in need of significant support and reassurance. The best answer is (b). The syringe is most effective. Medication can be disguised in food, but toddlers catch on very quickly. They should never be told that medicine is candy, as they can mistake harmful coloured pills for treats.

Pediatric neurologic emergencies

result from a wide variety of diseases and injuries, and the presenting signs and symptoms can be related to primary CNS disorders or secondary to other diseases. Clinical presentation may range from very subtle changes in the child's activity to comatose states. It is important for health care providers to have the knowledge and ability to anticipate and react quickly to changes affecting the CNS. Table 3.1 below provides a broad summary of possible causes of altered mental status in children. Table 3.1. Possible Causes of Altered Mental Status (AEIOU-TIPS) A = Alcohol Alcohol abuse is rare in young children, but should be considered a possibility in older children. E = Epilepsy Postictal state is a common cause of decreased LOC. Seizures may result from head injuries, fever, or epilepsy. I = Insulin Hypoglycemia or hyperglycemia. O = Overdose Ingestion, inhalation, or skin absorption may occur accidentally in younger children or intentionally in the older child. U = Uremia Metabolic causes, such as electrolyte disorders; renal, hepatic, or adrenal insufficiency; and congenital enzyme defects. T = Trauma Head injuries, hypoxia or shock. I = Infection Meningitis, encephalitis, sepsis, Reye's syndrome. P = Psychological Factitious coma is very rare in children. S = Shock, stroke, or syncope Inadequate brain perfusion. Early signs and symptoms of illness in infants may be subtle, as they are often unable to mount a fever due to immature thermoregulatory responses. In infants less than 3 months of age, manifestations of serious infection may be limited to a change in skin colour, irritability, poor feeding, and excessive sleeping. This module will help you to recognize common presentation patterns, refine your assessment skills, and identify management principles for pediatric neurologic emergencies. Specifically, this module describes the nursing care of children with the following neurological emergencies: Febrile seizure, caused by AOM Meningitis and sepsis

Appendix: CIAMPEDS Mnemonic for a Pediatric History

C = Chief complaint Note reason for the visit to the emergency department. Determine duration of the complaint. I = Immunization / Isolation Determine current immunization status. (See link https://www.canada.ca/en/public-health/services/provincial-territorial-immunization-information.html for provincial/territorial immunization schedules) Document lack of immunization. Ask about possible exposure to communicable diseases. Establish isolation for the potentially infectious child. Establish reverse isolation for an immunosuppressed child. A = Allergies Note type of reaction to medications, foods, products (latex), and environmental allergens. M = Medications Document medications, both prescribed and over-the-counter, including dose administered, time of the last dose, and duration of the medication use. P = Past medical history / Parents Include information about prior illnesses, injuries, hospitalizations, surgeries, chronic physical and psychiatric illnesses. Infant Complications during pregnancy or delivery, birth weight Number of days in hospital post-birth Menarche female Date and description of the last menstrual period Sexually active adolescents Type of birth control used, pregnancies Prior treatment for sexually transmitted diseases Parent's impression of the child's condition Identification of the primary caregiver Consideration of cultural differences that may affect the caregiver's impressions Evaluation of the caregiver's concerns and observations of the child's condition E = Events / Exposure / Environment / Evidence Illness Length of illness Date of onset Sequence of symptoms Exposure to others with similar symptoms Treatment provided prior to ER visit Injury Time and date injury occurred and a description of the circumstances M = Mechanism of injury, including the use of protective devices (seat belts, helmets) I = Injuries suspected V = Vital signs pre-hospital T = Treatment pre-hospital D = Diet / Diapers Recent oral intake and changes in eating and drinking patterns Foods consumed, e.g., breast milk, type of formula, solid foods, diet for age and developmental level, cultural differences, special or restricted items Time of the last void, as well as the frequency, odour, and colour of the urine over last 24 hours Bowel movements, including changes, colour, consistency, and frequency S = Symptoms Identification of symptoms since the onset of the illness or injury Social and family histories should be included. It is important to observe how the parent and child are interacting while taking the history. The following mnemonic can be used to assess health risks for an adolescent: H = home, E = education and eating, A = activities and affect, D = drugs, S = suicide ideations and attempts and sexual history (HEADS).

Case Study 1

Case Study 1 Ola Fari, a 9-month-old infant, arrives at the ED with a febrile illness. The physician has ordered fever management, blood work, urinalysis, and a lumbar puncture (LP). As you are preparing for the septic workup, you notice that a tearful Mrs. Fari is able to comfort her fussy baby with cuddling. When you try to lay Ola on the stretcher to check her temperature, she screams, hides her head, and clings to her mother. How would you interpret Ola's stranger anxiety? (Choose one answer.) a. It is normal. b. It is indicative of underlying psychological problems. c. It requires investigation. d. It should be ignored. Which of the following behaviours in a 9-month-old infant are normal? (Choose any that apply.) a. Crying with painful stimuli b. Clinging to mother c. Avoidance of eye contact d. Going to a stranger List three (3) interventions that reduce separation anxiety. List three (3) infant pain responses. Name two (2) nursing interventions for an infant in pain. How would you respond to the mother's statement, "I'm now going to call the doctor with every sniffle"? (Choose one answer.) a. "Watching your baby more carefully may not help. They get sick quite often." b. "That's a good idea. It's hard to tell when a child is really sick, and there is a risk of the illness being serious." c. "You should get your husband to make the call." d. "Sometimes parents feel responsible for their child's illness. Are you feeling this way?" It is determined that Ola has a viral infection. Discharge instructions are given, and a relieved Mrs. Fari takes her baby home. The correct answer is (a). Ola is demonstrating normal separation and stranger anxiety. She relies on her parents' comforting for anything that is unexpected, confusing, or painful. It would be a concern if Ola did not protest, as this might be an indication of serious illness or lack of attachment. The infant is learning to feel safe and develop trust by having needs for food, comfort, and sucking met. Touching, cuddling, and playing help the infant develop a healthy body image. The correct answers are (a) and (b). It is normal for infants to respond to painful stimuli by crying (a). Stranger anxiety is common from approximately 8 months to toddlerhood, and they will cling to a parent for comfort and safety (b). Answer (c) is incorrect because infants will usually meet your eyes even when they are afraid. Interventions that will help to reduce separation anxiety: Have the parent hold the child as much as possible Provide sensory stimulation (rocking, holding, cuddling, offering the child something to suck) if possible Use distractions if the infant shows an interest in the environment With pain, the infant will cry, withdraw an extremity, be irritable, and regress. Other signs of pain are increases in HR and RR, pallor, diaphoresis, and dilated pupils. It was once believed that infants did not experience pain due to their immature neurological systems. It is now accepted that they experience pain to the same degree as adults, although localized pain may be experienced as whole body pain. Ola will probably cry and struggle throughout the septic workup as a way of coping with separation, anxiety, and pain. Encourage self-comfort (sucking on a soother) and comforting by a parent. It is helpful to restrain an infant firmly and provide distractions with a soft voice, touch, and music. Allow movement as soon as possible after the procedure, as stress and frustration are diffused through motor activity. The correct answer is (d). Mrs. Fari is wondering if she could have changed the course of Ola's illness with earlier intervention. Allowing her to discuss concerns will help to reduce guilt, as well as provide an opportunity for education. Answer (a) may promote a sense of helplessness, answer (b) may set Ola up for overprotection, and answer (c) implies lack of confidence in Mrs. Fari. Reassure Mrs. Fari that she was timely in seeking care.

Case Study 3: Care of the Pediatric Patient with Gastroenteritis, Dehydration, and Shock

Mrs. Toll rushes in with Joey, a 14-month-old toddler, and tells you in a panicked voice that her son needs to be seen right away. She says he has been having diarrhea and refuses to drink anything. You observe a tired, frantic young mother holding a toddler who is crying weakly, appears pale, and has very dry lips. List three (3) assessment findings indicating Joey requires rapid nursing and medical intervention. What is the nurse's first priority intervention? a. Completing the assessment and obtaining a full set of vital signs b. Starting an IV and administering a fluid bolus c. Ensuring the airway is clear and administering oxygen d. Establishing cardiac monitoring In assessing the ABCDEs, you observe Joey's airway is patent and his breathing is rapid. While administering O2, you note that he does not resist the partial NRB mask and cries weakly as you reposition him. His HR is rapid, brachial pulse feels weaker than femoral pulse, and his capillary refill is 6 seconds. You request assistance from an experienced colleague and page the emergency physician. You find that Joey's lung sounds are clear. The emergency physician arrives to examine Joey and obtain more history from Mrs. Toll. Why is Joey's capillary refill 6 seconds? a. Body temperature b. Hypovolemia c. Medication d. Age You complete your assessment of Joey's ABCDEs by assessing his neurologic status and undressing him. He has a weak cry and responds to loud verbal stimuli by turning his head. When you expose him, you find no injuries and normal limb movement. His skin feels warm. You ask your colleagues to gather equipment for an IV and obtain a set of vital signs, which are as follows: HR 180 bpm BP Searching for an appropriate-sized cuff RR 34 breaths/min SpO2 93% on RA Temp 38.2ºC (rectal) Interpret Joey's vital signs and neurological status. What would Joey's GCS score be? The emergency physician orders an IV and laboratory investigations (i.e., CBC/Diff, glucose, electrolytes, blood urea nitrogen [BUN], creatinine [Cr], blood C & S, urinalysis, ABG). The RT is called to draw the ABG. Unable to find the appropriately-sized BP cuff, your colleague uses an infant-sized cuff to obtain a BP of 88/60 mmHg. Joey has now been in the ED for 10 minutes. You find an appropriately-sized BP cuff and obtain a lower and more accurate BP of 80/58 mmHg, but you are unsuccessful in starting an IV. The emergency physician is informed. What does the blood pressure indicate about Joey's condition? Which of the following IV accesses do you anticipate being ordered? a. Intraosseous b. Subclavian vein c. Femoral vein d. Scalp vein As the physician prepares to place an IO line and administer fluid, you estimate Joey's weight to be 11 kg, prepare the equipment, and review the procedure. The insertion site of the IO needle shows no signs of extravasation, and the fluid is flowing well as you stabilize it. How would you rate Joey's level of dehydration? a. Mild b. Moderate c. Severe d. Irreversible How much fluid should be given for Joey's first bolus? a. 300 ml b. 550 ml c. 220 ml d. 440 ml Joey receives his first bolus over 5 minutes. The reassessment vital signs are as follows: HR 160 bpm BP 80/54 mmHg RR 30 breaths/min Capillary refill 4 seconds What additional assessments are required? Joey's lungs are clear, and his heart sounds are normal. When Mrs. Toll tearfully asks you how Joey is doing, you inform her that his condition remains the same, but his heart rate has decreased slightly which is a positive sign. You are hydrating him with IV fluid and hope to see more of an improvement soon. The emergency physician orders another fluid bolus, for a total of 440 ml. Following this bolus, his vital signs are: HR 148 bpm BP 88/48 mmHg RR 30 breaths/min Capillary refill 2 seconds Joey's colour remains pale, but he is crying more vigorously and fighting the O2 mask; his peripheral and central pulses are also stronger. You give his mother some reassurance and explanations and assist her in comforting her child. A maintenance IV fluid infusion is ordered, and you calculate the fluid requirements to ensure the physician's order is accurate. What is the appropriate IV maintenance rate for Joey? a. 42 ml/hr b. 110 ml/hr c. 25 ml/hr d. 66 ml/hr The next steps are explained to Mrs. Toll, and the insertion of an in/out urinary catheter yields 10 ml of clear, dark yellow urine from which a urinalysis is obtained. The physician's diagnosis is severe dehydration and gastroenteritis, and Joey is admitted to hospital for further treatment and observation. What information do you give the family about gastroenteritis and oral rehydration? Joey's mother is relieved that Joey is feeling better and appears much calmer. She apologizes for rushing in and making a fuss. You reassure her, and she thanks you as you take Joey upstairs to his room. Suggested answers The ability to make assessments quickly, differentiating between a very ill child and a child who needs less urgent care, often comes with experience. Any of the following findings indicate Joey needs immediate attention: Pale skin colour Dry mucous membranes Weak cry History of diarrhea and lack of oral intake Parent's panicky behaviour (One cannot always be sure this anxiety is directly related to the condition of the child, but in most cases parental concerns should be taken seriously.) The correct answer is (c). Joey is weak and dehydrated, and his tissues are not receiving adequate oxygenation; therefore, the first priority is ensuring an open airway with adequate ventilation and oxygenation. An assessment (a) must be completed, but the primary survey (ABCDE-FG) indicates immediate intervention is required. This patient is probably hypovolemic and may require a fluid bolus (b) and should be placed on a cardiac monitor (d), but these are not the first priorities. The correct answer is (b). The decreased tissue perfusion and vasoconstriction present with hypovolemia cause prolonged capillary refill (normal is ≤ 2 seconds). However, keep in mind that a child who has a low body temperature, is in a cold room, or is taking medication that causes vasoconstriction may also have slower capillary refill (a), (c). Age (d) does not alter capillary refill. Although Joey is just 2 months older than the infant age range, his HR is elevated for an awake toddler (98-140 bpm). His clinical presentation (e.g., crying weakly, pale, weak peripheral pulses) is indicative of compromised perfusion. His RR is also within normal range, although one could argue it is approaching high-normal. Joey's SpO2 is 93% on RA, which is indicative of decreased oxyhemoglobin and tissue perfusion. Lastly, Joey's low-grade temperature is likely the result of viral gastroenteritis. With regards to his GCS, Joey responds to verbal stimuli, has a weak cry but appears consolable, and has normal limb movement; these findings reflect a GCS score of 13/15. Joey's BP is below the normal range for a toddler, keeping in mind that the AHA (2016) identifies hypotension as SBP < 70 mmHg + (2 x age in years). Recall that hypotension in the pediatric patient is an ominous sign of decompensated shock. At this moment, Joey is progressing towards decompensated shock as evidenced by circulatory compromise and decreased LOC. Intraosseous (a) cannulation would be the best choice. When peripheral access is difficult to establish, the temporary use of IO cannulation is recommended for reliable fluid and medication administration. Insertion of a subclavian catheter (b) carries a risk of pneumothorax. Catheterizing femoral or saphenous veins (c) is a lengthy procedure, and the scalp veins (d) are not only fragile and unreliable, but contraindicated in children > 12 months (ENA, 2014). The correct answer is (c). Joey is severely dehydrated. Although Joey is not hypotensive as of yet, other clinical findings are indicative of severe hydration (e.g., very dry mucous membranes, pallor, unequal central/peripheral pulses, delayed capillary refill, lethargy with weak cry [GCS 13/15]). The answer is 220 ml (c). For pediatric patients, fluid boluses are calculated on a per-kg basis. The standard bolus of crystalloid solution is 20 ml/kg, thus Joey's weight (11 kg) requires a first bolus of 11 x 20 = 220 ml. Auscultation of breath and heart sounds is necessary to ensure Joey is not becoming fluid overloaded (e.g., crackles, S3). Most children can tolerate a bolus without ill-effects, and since Joey is hypovolemic, fluid overload would be an unusual finding. Capillary refill, skin temperature and color, and quality of central/peripheral pulses must also be assessed. Urine output is also important, as it is a reflection of CO and renal perfusion. Joey weighs 11 kg, so the correct answer is 42 ml/hr (a) (4/2/1 rule: 4 ml x 10 kg for the first 10 kg, plus 2 ml x 1 kg for the second 10 kg). The family should be given the following information about gastroenteritis and oral rehydration: Good hygiene to prevent spread or exacerbation of gastroenteritis Oral rehydration should involve a solution containing pediatric formulations of glucose and sodium (e.g., Pedialyte), given via syringe (5 ml volumes), every 2-5 mins. Avoid sports or high energy drinks (e.g., Gatorade) Avoid juice, as high in sugar (worsen symptoms) Avoid plain water if possible, as hyponatremia and hypoglycemia may result Monitor the child's intake/output, weight When to return to ED

Intubation of the Pediatric Patient

A patient with imminent respiratory failure may require intubation and mechanical ventilation. Signs and symptoms of impending respiratory failure include: Decreasing level of consciousness (LOC) Increased WOB and respiratory rate (RR) Decreased air movement on auscultation Possible absence of adventitious lung sounds, such as wheezing due to reduced airflow Hypoventilation or apnea Fatigue Ashen colour, cyanosis Acidosis (hypercapnia, hypoxemia) Failure to respond to therapy Many patients who require intubation still have a gag reflex, muscle tone, and clenched teeth, thus induction may be required. Rapid sequence intubation (RSI) is used to "...render the patient unconscious and paralyzed in order to intubate the trachea without the need for bag-mask ventilation, which can cause gastric distension and risk of aspiration" (O'Neal, 2013, p. 82). The following equipment is required: Universal precautions equipment (e.g., gloves, mask) Continuous cardiopulmonary monitor Two laryngoscopes (curved and straight blades), video laryngoscope; ensure they are working (i.e., lights) Carbon dioxide (CO2) monitoring device (e.g., end-tidal CO2 detector, exhaled CO2 capnography, esophageal detector device) O2 supply, BVM Oral/Tracheal suction equipment (ensure in working order) Oropharyngeal airway (OPA)/Nasopharyngeal airway (NPA) (various sizes) IV/Intraosseous (IO) infusion equipment ETTs (all sizes, with/without cuffs; 0.5 mm above/below expected size), as well as stylets Syringes to test inflate ETT balloons (e.g., 3, 5, and 10 ml) Adhesive cloth/tape to secure ETT, or commercial ETT holder Towel/Pad to align airway, place under head/shoulders/torso Medications, each drawn up in a clearly labelled syringe Equipment for difficult airway management (e.g., cricothyrotomy) The procedure generally follows the 7 P's: Preparation ("zero minus 5-10 mins") Assemble a trained and competent team (e.g., physician/nurse practitioner [NP], emergency nurse(s), respiratory therapist [RT]) Prepare equipment, as outlined above Establish IV/IO access Connect patient to continuous cardiopulmonary monitor Preoxygenation ("zero minus 5 mins") Pre-oxygenate the patient with 100% high-flow O2 (allows for up to 8 mins of apnea before desaturation) Pretreatment/Premedication ("zero minus 3 mins") Medications that blunt physiologic responses to intubation Atropine: blunts reflex bradycardia, secretions Lidocaine: blunts cough reflex, increased intracranial pressure [ICP]) Fentanyl: analgesia, amnesia Pancuronium/Vecuronium: defasiculating dose, if Succinylcholine given Sedating agent Ketamine: dissociative induction agent (***agent of choice in asthma***) Etomidate: sedative/hypnotic (***agent of choice in head injury***) Propofol: induction agent (***caution if soy/egg allergy***) Thiopental: barbiturate (largely replaced by above agents) (***contraindicated in asthma***) Paralysis ("zero") Depolarizing neuromuscular blocking agent [NMBA]): Succinylcholine (rapid onset, short duration of action) Cause muscle fasciculations Contraindicated in head injury/increased ICP, hyperkalemia, cardiovascular disease, renal failure, muscular syndromes, crush injuries, open-globe injuries, malignant hyperthermia (or family history of) Non-depolarizing NMBA: Rocuronium (***drug of choice if Succinylcholine contraindicated***), Vecuronium, Pancuronium Do not cause muscle fasciculations Placement ("zero plus 45 secs") Cricoid pressure (Sellick maneuver): serves to improve visualization of vocal cords, limit gastric inflation that may interfere with ventilation, and prevent aspiration. There is no evidence to support its use in the pediatric population, thus routine use of cricoid pressure is not recommended, as it may obstruct the airway even when performed properly (AHA, 2016; Emergency Nurses Association [ENA], 2014; Tucker & Coussa, 2009). Cricoid pressure should only be considered in an unresponsive victim and only if an additional provider with no other resuscitative duties is present (AHA, 2016). Intubate patient, inflate cuff (if applicable) Placement Verification Confirm placement by chest rise/fall, auscultation, CO2 detector Secure the ETT Post-intubation Management Administer additional medications as needed for ventilatory management (e.g., sedation) (O'Neal, 2013; Tucker & Coussa, 2009)

The Toddler in the Emergency Department

Age Developmental Milestones Common Fears Approach Health Promotion 1-3 years Average monthly weight gain is 200 g and height is 1 cm. Being left alone, interacting with strangers. Changes in the usual routine, such as seeing a parent in a hospital gown. People who have masks on or who are dressed in costumes. Getting hurt (falls, cuts, skin abrasions). Losing control or having to revert to 'baby things'. If possible allow independence (e.g., encourage to dress/ undress, open doors, turn on lights, etc.) Let parents remain with the child. Feels vulnerable in supine position; when possible do assessments in a sitting or standing position. Loves bandages on cuts. Loves guessing games. Important to be able to get dressed and undressed by self. May have a 'transitional object', such as a doll or blanket. Allow the toddler to hold this object or keep it in sight. May cooperate with treatments if one pretends to be a fighter pilot with the O2 mask, for example. May have fears from previous hospital visits that are very real to them. Keep explanations simple, use non-threatening language. Use gentle but firm restraining. Reassure family and gain their trust; talk calmly, quietly, confidently. Prevent injuries and poisonings as with infants; teach street safety. 2 years Uses 2-word sentences and likes saying "No" Learns and uses one or more new words a week Develops handedness Runs well Goes up and down stairs (one step at a time) Loves sharing stories and turning the pages one at a time, stringing beads, sorting, pouring Wants to become independent 3 years Speaks 3-word sentences clearly enough to be understood most of the time Understands 2-step directions Goes up and down stairs (alternating feet) Stands on one foot briefly Plays make-believe games with actions and words Shows affection with words and actions Cooperates with requests half of the time Loves guessing games, sorting, Lego, surprises May develop new fears Is developing autonomy Explores objects (banging, tasting, feeling, seeing what they will do and where they will go) Explores body (may attempt to fit small objects into orifices) Pain Responses Nonverbal Withdrawal of extremity; nonspecific aggression (hitting, biting, kicking); use of the entire body to resist; regression. Verbal Crying, screaming, identifying location ("leg ouches,and "arm hurts," points to abdomen, touches ear).

Rhythm Disturbances

The most common causes of cardiopulmonary arrest in adults are lethal arrhythmias related to heart disease. In children, however, the most common causes of cardiopulmonary arrest are conditions that lead to shock or respiratory failure. Many cardiac rhythm abnormalities in children are not life-threatening; however, some require close monitoring and treatment. Although they may have the same rhythm abnormality, keep in mind that several important differences exist between children and adults. As discussed in the previous section, CO = HR x SV. Stroke volume is in turn determined by preload, afterload, and contractility. Therefore, alterations in HR, preload, afterload, and/or contractility will result in an alteration in CO. Blood pressure is determined by blood flow and the size of blood vessels; in other words, BP is dependent upon CO and SVR. If CO falls, but SVR increases, BP may be maintained. The relationship of these factors is outlined in the figure below. Figure 2.1. Factors Determining Blood Pressure Although an in-depth review of ECG interpretation is beyond the scope of this module, the table below provides a summary of sinus, supraventricular, and ventricular tachycardia. Table 2.1. Differentiating Sinus, Supraventricular, and Ventricular Tachycardia Sinus tachycardia (ST) Supraventricular tachycardia (SVT) Ventricular tachycardia (VT) QRS Normal QRS ≤ 0.09 sec Normal QRS ≤ 0.09 sec Wide QRS >.09 sec Heart rate Variability with stress, activity Infants < 220 bpm Children < 180 bpm No variability Infants ≥ 220 bpm Children ≥ 180 bpm Regular ≥ 120 bpm History Gradual onset Vomiting, diarrhea, dehydration, fever, anxiety, blood loss Abrupt onset/termination/or both Nonspecific: irritability, poor feeding, pallor Hypoxia, acidosis, poisoning Physical examination Consistent with hypovolemia: pallor, poor skin turgor, dry mucous membranes, altered LOC Infants: consistent with cardiac failure (increased WOB, crackles, altered LOC, hepatomegaly) Children: palpitations Depends on CO; possibility of poor perfusion and altered LOC Management of tachyarrhythmias with a pulse involves primary and secondary surveys, as outlined previously, in addition to the following measures according to the child's severity: Vagal maneuvers Medication therapy (pharmacologic conversion) Cardioversion (electrical conversion) Other interventions Vagal maneuvers can be performed on a child of any age, but should be reserved for those who are hemodynamically stable and should not delay cardioversion. Monitoring of ABCs during the procedure is essential, as well as obtaining a baseline ECG prior to commencing. Vagal maneuvers appropriate for the pediatric population include: Ice to the face: upper half of the face, 15-20 secs (repeat x 1, utilize other measures if rhythm does not convert) Valsalva maneuver: blow through straw (older children) Carotid sinus massage (older children) If vagal maneuvers are unsuccessful (or not appropriate), medication therapy can be attempted. Adenosine is the drug of choice for SVT, followed by amiodarone and procainamide. Only adenosine will be discussed here. The pediatric dosage of adenosine IV/IO for SVT is as follows: Initial dose: 0.1 mg/kg (max 6 mg) rapid IV bolus, followed by 5-10 ml NS flush If effective, will convert within 15-30 secs Second dose: 0.2 mg/kg (max 12 mg) rapid IV bolus x 1 dose, followed by 5-10 ml NS flush Acute management for unstable children who do not respond to adenosine involves synchronized cardioversion and overdrive pacing. The use of overdrive pacing in the pediatric population is recommended for expert consultation only, thus synchronized cardioversion is considered the emergency intervention and will be discussed here. The goal of synchronized cardioversion is to interrupt an abnormal electrical circuit within the heart that is allowing an arrhythmia to continue. Synchronized cardioversion uses a defibrillator that can be set to "synchronous" mode, which produces an electrical shock to convert the arrhythmia to sinus rhythm. Synchronization is what differentiates cardioversion from defibrillation. Once the synchronize mode is activated on the defibrillator, the machine delays delivery of the shock until it has synchronized with the patient's QRS complexes, specifically the 'R' wave . If a shock is delivered at the wrong time in the cardiac cycle (e.g., T wave), it can put the patient into ventricular fibrillation (VF). Prior to cardioversion, the child should be NPO for 4-6 hours and must be well sedated. Following the procedure, the emergency nurse should document the subsequent rhythm with an ECG. The steps for cardioversion are as follows: Ensure patent, large bore IV insitu Ensure adequate sedation Turn on the defibrillator (monophasic or biphasic) Attach defibrillator pads to the patient (paddles rarely used today) and ensure proper display of patient's rhythm Engage the synchronization mode by pressing the "Sync" control button Look for markers or spikes on R waves indicating sync mode is activated If necessary, adjust the monitor/pads until sync markers occur with each R wave Select the appropriate energy level Initial dose: 0.5-1 joules[J]/kg Second dose: 2 J/kg (or more gradually, from 0.5 to 1.0 to 2 J/kg) Announce to team members: "Charging. Stand clear!"...press "Charge" button When defibrillator is charged, state firmly: "I'm clear"...look to see that you are clear "You're clear"...look to see that all others are clear "All clear"...look again to ensure you and all others are clear "Shocking!"...press "Shock" button Check the monitor...if rhythm persists, increase the joules, "Shock" again ***Reset sync mode after each synchronized shock because most defibrillators default back to unsynchronized mode*** If synchronized cardioversion is unsuccessful, expert consultation is advised prior to administering amiodarone or procainamide. Visit the following website for more information on SVT in the pediatric patient: https://www.youtube.com/watch?v=-XKx8aO2ejE Visit the following website to view cardioversion of SVT: https://www.youtube.com/watch?v=AJY2yoQUqzA

Case Study 10: Care of the Pediatric Patient with Projectile Vomiting due to Pyloric Stenosis

Early one morning, a mother rushes in with her small baby, frantically speaking in Cantonese. The baby does not show any signs of respiratory distress, has normal skin colour, and is awake and crying. The unit clerk finds a nurse in the hospital who speaks Cantonese, and you find out that the baby is 3 weeks old and has been vomiting with feeds since yesterday. Mrs. Kang tells you that Jin is vomiting after being breastfed for about 10 minutes. He always seems hungry despite vigorous sucking. She tearfully informs you that he cried most of the night and his diaper was dry this morning. With the interpreter's help, you find that Jin has no known allergies and is not taking any medications. The pregnancy and delivery were uncomplicated, and he has been "a good baby" until now. Jin nursed for about 10 minutes just before coming to hospital and his diaper is still dry. Jin's vital signs are as follows: HR 140 bpm RR 30 breaths/min Temperature 38°C (rectal) SpO2 98% on RA Capillary refill < 2 secs What laboratory studies do you anticipate? (Choose all that apply.) a. CBC and electrolytes b. Blood cultures c. Urinalysis d. Type and cross-match Jin is undressed, and your observation of his upper abdomen reveals peristaltic waves from left to right. Suddenly he vomits about 20 ml of white curdish fluid; there is no blood in the emesis. You turn him on his side to protect his airway; he is vomiting so forcefully that the emesis reaches the leg of your scrub pants, over half a metre away. You realize that he may be very ill. Next to each symptom, place the letter (I) for intussusception, (P) for pyloric stenosis, or (I and P) for both. a. Bile emesis ______ b. Dehydration ______ c. Emesis without bile ______ d. Altered level of consciousness ______ e. Projectile vomiting ______ f. Rectal bleeding ______ The emesis is cleaned up, and Jin is given to Mrs. Kang to hold and comfort. The emergency physician arrives, and you tell him about the peristaltic waves and projectile vomiting. You inform him that no blood was visible in the emesis and it was cleaned up before you thought about testing for occult blood. After he examines Jin, the doctor states he is considering pyloric stenosis as a possible diagnosis and orders laboratory studies and an abdominal ultrasound. Mrs. Kang seems very worried, and you explain the plan with assistance from the nurse (interpreter). She wants to know what medicine is given for this and how soon she can take her baby home. What is the best response for the mother? a. "You probably won't be able to take your baby home once the tests are done." b. "I have no idea if you will be able take your baby home or not." c. "We will wait for the doctor's decision, but your baby may have to stay in the hospital for a few days." d. "We should know what is going on in an hour, and you and your baby will be on your way home shortly after that." Jin is more relaxed after vomiting and is lying quietly in his mother's arms. The emergency physician takes this opportunity to do a more extensive abdominal examination. He palpates a small density in the RUQ, making him fairly certain this is a pyloric stenosis. With the interpreter's help, it is explained that Jin may require surgery. Mrs. Kang says she wants to take Jin home and come back tomorrow after she has discussed this with her family. The interpreter offers to phone Jin's father, and after the parents converse on the phone, Mrs. Kang calms down and states her husband is coming in to get some more information and sign the consent forms. The interpreter accompanies Mrs. Kang and her baby to the radiology department. The correct answers are (a) and (c). Any child who is vomiting or who may not be taking adequate fluids should have baseline electrolytes assessed. CBC is also appropriate in this case as a pre-op baseline, as well as to rule out infection given Jin's high-normal temperature of 38°C (fever at age < 2-3 mths = > 38°C [rectal]). A urinalysis should also be performed due to persistent vomiting and lack of urine output, and to rule out infection in the neonate. Obtaining blood cultures (b) is not a priority, as the symptoms are likely not consistent with a systemic infection. Jin has no history of trauma and is not severely hypovolemic, thus a cross-match (d) is not a priority. Symptoms of intussusception (I) and pyloric stenosis (P) a. ___I___ Bile emesis b. __I, P__ Dehydration c. ___P__ Emesis without bile d. __I, P__ Altered LOC e. ___P___ Projectile vomiting f. ___I___ Rectal bleeding The correct answer is (c). It seems very likely from the clinical presentation that Jin has an obstruction requiring hospitalization, and most likely surgery. The surgery of choice for pyloric stenosis is a pyloromyotomy - an incision into the muscle fibres that leaves the mucous layer intact. In any case, he will have to be hospitalized due to dehydration and increased risk of complications at 3 weeks of age - he has had no urine output for several hours.

Case Study 1

Patricia Jones, a 6-month-old infant, is carried into the ED by her young mother. The mother states that her baby hasn't been eating well for the last three days. Patricia is sound asleep in her car seat, and the initial assessment indicates that she is not seriously ill. The components of the Pediatric Assessment Triangle (PAT) that assist with the initial assessment are: a. Respiratory rate, work of breathing, skin colour b. Skin temperature, work of breathing, fontanels c. Response to the environment, capillary refill, peripheral pulses d. Diaphoresis, skin temperature, urine output Patricia is roused and begins to cry, but is easily consoled. You note she has nasal congestion but her airway is clear, with no unusual sounds. Her breathing pattern is regular, respirations are non-laboured, and there is no nasal flaring. Her lips and skin are pink, and capillary refill is less than 2 seconds. She is comfortably warm to touch and not diaphoretic. The ABCs are normal, and you continue with D (disability), E (exposure/environmental control), and the secondary survey. List at least two (2) pediatric neurologic assessment criteria. While undressing Patricia, you note she has no rashes, bruises, or lesions. Her peripheral and central pulses are strong and regular, and the capillary refill is less than 2 seconds. When Patricia is uncovered she cries in protest, and her mother offers appropriate comforting measures. Patricia's primary survey is normal, and she does not appear to be acutely ill, but the sleepiness and fussiness should be further assessed. Her vital signs are as follows: HR 185 bpm RR 64 breaths/min Temp 38.3ºC (rectal) SpO2 95% on RA BP 87/55 mmHg Weight 6.4 kg Utilizing the information from Table 1, interpret Patricia's vital signs. While interpreting these vital signs, you recall that tachycardia and tachypnea are often caused by fear, anxiety, fever, and crying, but they may also indicate hypoxia. Vital signs must be interpreted in context with the assessment findings. Normal vital signs do not necessarily mean a child is well; for example, BP may remain normal in hypovolemic children due to their ability to compensate. It is important to consider mental status as well, since lethargy and irritability may be signs of hypoxia. Children with high fevers may have a benign illness, and children with little or no fever may be significantly ill. Consider the following scenarios. A 6-month-old infant has a temperature of 38.3ºC (rectal). She is pink, flushed, awake, and tachycardic, with brisk capillary refill, strong peripheral pulses, and flat anterior fontanel. These are normal findings for an infant. True or False? A 6-month-old infant with a temperature of 36.6ºC (rectal) is sleepy, slow to arouse, and pale. Capillary refill is 5 seconds, and HR is 80 bpm. These are normal findings for an infant. True or False? A 2-week-old infant has a rectal temperature of 38.3ºC with brisk capillary refill, pink colour, and easy arousal. These are normal findings for an infant. True or False? With nothing alarming in your interpretation of the vital signs, you continue your assessment by taking the history. You are respectful and non-judgmental and provide knowledgeable, culturally and psychosocially appropriate care. What information should be obtained when taking the history? (Choose all that apply.) a. History of present illness b. Presence of underlying chronic illness c. Observation of parent-child interaction d. Significant past medical history Ms. Jones states again that Patricia has been "fussy, sleepy, and has a runny nose." You ask about allergies, and she tells you that Patricia does not seem to be allergic to anything. Ms. Jones does say that Patricia remained in the hospital for one week following her birth, and she has been to the health clinic once. Ms. Jones says everyone in the family is healthy. She tells you that she is 20 years old and manages to go to school and work despite having no assistance from Patricia's father. What additional information do you need to get from the history? You learn that Patricia's mother received no prenatal care and the baby was one month premature, weighing 2.1 kg at birth. She was observed in hospital for breathing problems requiring oxygen but did not require a "breathing tube." You ask Ms. Jones whether she takes Patricia to the doctor regularly and how many "shots" she has received. Ms. Jones states that Patricia has not seen a doctor and that she has not received any "childhood shots." She asks you when Patricia is supposed to have them. What immunizations should Patricia have already received? You reassure Ms. Jones that is it not too late to begin immunization. Ms. Jones tells you that Patricia is taking Similac formula and baby food. She has not been eating well for 2 days but continues to drink 6 oz of formula every 4 hours during the day. She has wet diapers frequently and soft brown stools once a day. Ms. Jones thinks her baby appears small and is concerned about her growth. Is a weight of 6.4 kg normal for an infant of Patricia's age? Patricia is now awake, and her mom is making eye contact while comforting her. You know that gathering information about this relationship is essential in ruling out the possibility of child abuse or neglect. List at least two (2) signs that may indicate a dysfunctional parent-child relationship. The interaction between Ms. Jones and Patricia seems normal. Patricia is more comfortable because she is less warmly wrapped, and you tell Ms. Jones about Patricia's fever. You provide the following information about infants with fevers, using vocabulary Ms. Jones can easily understand: They may be cranky, irritable, and sleepy. They may breathe more quickly than they normally would. They have higher temperatures than adults because their nervous system is immature. Infants with fevers can have illnesses that are not serious, and infants with little or no fever can be very sick. Infants 3 months old or less should have a medical examination if they have a fever. When asked who takes care of the baby, Ms. Jones says she lives with her mother, who takes care of her most of the time. Also living in the home are Ms. Jones's 12-year-old sister and 18-year-old brother. She talks about her family positively and says that her mother and siblings are very supportive. Patricia's mother gently places her on the stretcher for you to complete your assessment. Her fontanel is flat, and while you are listening to her breath sounds she grabs your stethoscope and tries to put it in her mouth. As you palpate her abdomen you speak quietly to her and she squeals with delight. She rolls over and you grab her to prevent her from rolling off the stretcher. Her breath sounds are clear to auscultation and her abdomen is soft and non-tender. Which of the following observations indicates Patricia is developing normally for her age? a. Flat fontanel, rolling over, ability to take bottle well b. Rolling over, grasping the stethoscope, squealing c. Normal vital signs, normal size on the growth chart, drooling You think Patricia has a mild upper respiratory tract infection (URTI), but while the emergency physician examines her, you review potential problems that can exist even though a child looks "not sick." Name at least four (4) serious health problems that can exist even though a child does not appear to be unwell. After a thorough examination, the emergency physician decides Patricia does have a mild upper respiratory tract infection (URTI). The discharge order includes Acetaminophen 80 mg every 4 hours as needed for fever and a follow-up with a family physician. Your discharge instructions should include which of the following? (Mark each 'Yes' or 'No'.) a. Ask social services to check on Ms. Jones b. Teach Ms. Jones how to suction Patricia's nose c. Teach Ms. Jones how and when to administer Tylenol d. Instruct Ms. Jones to increase Patricia's feedings e. Help Ms. Jones find a family physician f. Stress the importance of immunizations and explain where they can be obtained Answer (a) is correct. Patricia's RR and skin colour contribute to the general impression that she is not acutely ill. If she was awake, observation of her response to the environment would have provided additional information. Respiratory rate, colour, WOB, and response to the environment can be assessed without touching the child. It is important to obtain information about perfusion when touching the child during the primary survey. Assess capillary refill (>2 seconds may indicate poor perfusion), hydration, and neurologic status (e.g., palpate anterior fontanel with child sitting supported at 45°). Skin temperature can be difficult to assess, especially when the child is wrapped in clothing. The child may feel warm even when she/he is afebrile. The neurologic status of a child is assessed by: Age-appropriate response to the environment AVPU (see primary survey chart), GCS Arousability and response to comforting measures Recognition of, and response, to parents/caregivers Condition of anterior fontanel (< 2 years of age) Quality of cry (high-pitched, moaning, or whimpering cries are abnormal) Pupil size, PERRL Muscle tone: abnormal flaccidity, rigidity, gait, and/or posture Note: Paradoxical irritability is described as when a child cries when comforted and is quiet when left alone (may indicate meningitis). Children with paradoxical irritability and those who seem unaware of or unconcerned with the hospital environment may be very ill and require immediate intervention. Patricia's HR is high (165), RR is high (64), and her temperature is elevated (38.3°C rectal); her SpO2 is borderline at 95% on RA. It is not clear at this point why the vital signs are altered. True. The symptoms associated with the febrile infant are normal. False. This infant, although afebrile, is slow to arouse, has delayed capillary refill, pallor, and bradycardia. These are not normal findings. False. This infant is only 2 weeks of age so should be assessed and seen by a physician immediately. The correct answers are a, b, c, and d. Questions should be structured to elicit the following information: (a) history of present illness, (b) presence of underlying chronic illness, (c) parent-child interaction, and (d) significant past medical history. A useful tool in obtaining a rapid history is the mnemonic CIAMPEDS. In this case, missing information is identified by a question mark. C = Chief complaint √ I = Immunizations ______? Isolation √ A = Allergies √ M = Medications √ P = Past medical history ______? Patricia was hospitalized for 1 week after birth. Parent's impression of the child's condition √ E = Events √ D = Diet _______? Diapers_________? S = Symptoms √ 9. According to the Public Health Agency of Canada (PHAC, 2017), Patricia should have received the following immunizations by 6 months of age: Diphtheria, Tetanus toxoid, acellular Pertussis, inactivated Polio, and Haemophilus influenzae Type b (DTaP-IPV-Hib) at 2, 4, and 6 mths Meningococcal conjugate (MenconC) at 4 mths Pneumococcal conjugate 13 valent (Pneu-C13) at 2 and 4 mths (6 mths also, if high risk) Rotavirus at 2, 4 and 6 mths Note: The above schedule is reflective of Alberta guidelines (slight variations exist across provinces/territories). Provincial and territorial guidelines can be found at the following link: https://www.canada.ca/content/dam/phac-aspc/documents/services/provincial-territorial-immunization-information/childhood_schedule-12-2017.pdf. Yes, Patricia's weight is normal. Infants generally double their birth weight by 6 months and triple it by 1 year of age. Although she was one month premature and weighed 2.1 kg at birth, she has already tripled her birth weight. Showing the mother a pediatric growth chart would be useful. Following are signs of a dysfunctional parent-child relationship: Holding the baby at a distance Making little or no eye contact Not talking to the baby Calling the baby "it" and noting defects Expressing dissatisfaction when caretaking Readily surrendering the baby to someone else Ignoring the baby's communication of needs Handling the baby roughly 12. The correct answer is (b). Normal development for a 6-month-old infant includes rolling over; grasping objects; babbling, laughing, and squealing socially; and visually following objects. 13. The following serious health problems can exist even through a child does not appear to be seriously ill: Ingestion of toxic material Fever in infant < 3 mths History of altered LOC Seizure Possible anaphylaxis History of trauma Discharge instructions would include the following: Ask social services to check on Ms. Jones. No. You have observed Ms. Jones responding appropriately to her baby and discovered she has an adequate support system. You also know that Patricia is developing normally. That being said, it may be useful for Ms. Jones to be aware of this resource in case of future need. Teach Ms. Jones how to suction Patricia's nose. Yes. Teach her how to use a bulb syringe to clear Patricia's nostrils. Teach Ms. Jones how and when to administer Tylenol. Yes. Teach about fever management by showing her how (e.g., dose, method) and when to give Tylenol. Instruct Ms. Jones to increase Patricia's feedings. No. Reassure her that it is normal for children to have a decreased appetite when they are feeling unwell. Also reassure her that Patricia's growth is normal for a premature baby. Help Ms. Jones to find a family physician. Yes. Help her to understand why it is important to have a family physician, and suggest how she can find one (e.g., offer regional health authority contact information). Stress the importance of immunizations and explain where they can be obtained. Yes. Inform her where the closest community or public health centre is, and provide contact information.

Upper Airway Emergencies

Respiratory distress can result when structures of the upper airway are occluded by edema, secretions, foreign bodies, or anatomic defects. This module describes the nursing care of children with the following upper airway emergencies: Pertussis Croup (laryngotracheobronchitis) Epiglottitis Foreign body aspiration Pertussis Pertussis, or 'whooping cough', is an acute infectious disease caused by the bacterium Bordetella pertussis. It is easily spread from person to person via air droplets, from sneezing, coughing, etc. A person may also become infected by touching an object that contains infected secretions, such as toys. The bacteria can survive up to 2 - 5 days on dry objects, such as toys, paper, and clothes. B. pertussis attach to the cilia of the respiratory epithelial cells, producing toxins that paralyze the cilia and cause inflammation of the respiratory tract, which interferes with the clearing of secretions. The incubation period is commonly 7-10 days but may occur up to 28 days post-infection. The clinical course of the illness is divided into three stages: Catarrhal stage (1-2 weeks) - sneezing, runny nose, low-grade fever, mild occasional cough (most contagious first 2 weeks after coughing starts). Paroxysmal cough stage - paroxysms of cough due to difficulty expelling mucous from the tracheobronchial tree. At the end of the paroxysm, a long inspiration is often accompanied by a characteristic 'whoop'. Patients, especially infants and children, may become cyanotic and appear very ill, with vomiting and exhaustion following each episode. Convalescence (weeks to months) - The cough becomes less paroxysmal and eventually disappears. Pneumonia is the most common complication as it spreads to the lower respiratory tract, although dehydration and weight loss are also of concern in infants and young children. The management is primarily supportive, with macrolide antibiotics (e.g., Erythromycin, Azithromycin) also being prescribed. Children should be isolated until no longer infectious (i.e., at least 5 days of antibiotic therapy) or for 3 weeks if not treated. Pertussis is a preventable disease via immunization with the DTaP vaccine. Visit the following site to listen to an infant with pertussis: https://www.youtube.com/watch?v=S3oZrMGDMMw Croup Croup refers to several illnesses affecting the pediatric population that may cause respiratory distress. Various viral illnesses may contribute to the development of croup, including parainfluenza, influenza A, influenza B, respiratory syncytial virus (RSV), and rhinovirus. Croup occurs more commonly in late fall/winter, and in children (> boys) between the ages of 6 - 36 months (Beier & Vandenbranden, 2012). Croup results in tracheal edema just inferior to the larynx, and may be accompanied by inflammation and increased secretions. Clinical signs and symptoms of croup typically follow a viral infection (e.g., rhinorrhea, sore throat, fever), and are related to airway narrowing and include a hoarse, seal-like 'barky' cough (> at night), inspiratory stridor, and retractions of the neck and/or sternum. Keep in mind that acute signs/symptoms may not be present with impending respiratory failure, due to fatigue and lethargy. To view a baby with stridor and retractions, go to: http://www.youtube.com/watch?v=QkaX83H31QY&feature=player_detailpage To view a baby with seal-like 'barky' cough and stridor, go to: http://drstephsomers.blogspot.ca/2010/10/croup-and-croupy-cough.html Diagnosis of croup is determined based on the history and physical examination. The severity of croup with related interventions is divided into mild, moderate, and severe as follows: Mild (no stridor or substantial chest wall indrawing at rest) Systemic corticosteroids (e.g., dexamethasone 0.6 mg/kg per os [PO]) Moderate (stridor or chest wall indrawing at rest without agitation) Minimize intervention (keep child on parent's lap, position of comfort) Dexamethasone 0.6 mg/kg PO (otherwise keep nil per os [NPO]) Nebulized epinephrine 1:1000 (5 ml) or racemic epinephrine 2.25% (0.5 ml in 2.5 ml of saline)...observe for at least 2 hours to monitor for rebound effect Consider heliox (helium & O2) if severe (controversial) Severe (stridor and indrawing of sternum, agitation or lethargy) Minimize intervention Blow-by O2 (optional unless cyanosis is present) Assisted ventilation (e.g., bag-valve-mask [BVM]) Dexamethasone (0.15-0.60 mg/kg, depending on severity) Endotracheal intubation (½ size smaller endotracheal tube [ETT] than predicted to avoid injury to edematous subglottic area) Surgical airway, if necessary (American Heart Association [AHA], 2016) As noted above, administration of nebulized epinephrine may result in a rebound effect, resulting in the return of respiratory distress within minutes or hours after treatment. Children receiving this medication should be observed for at least two hours after administration, as they may require hospital admission. Epiglottitis Epiglottitis is a life-threatening infection or inflammation of the epiglottis and surrounding area that may mimic croup, resulting in complete airway obstruction. Historically, the most common cause of epiglottis has been Haemophilus influenzae Type b (Hib), which can be prevented via vaccination; however, it is known today that other organisms (i.e., bacteria, viruses, fungi) can cause epiglottitis as well. Non-infectious irritants, such as chemicals and inhaled hot substances (e.g., crack cocaine), may also cause inflammation. Originally more prevalent in adults, it has become a predominantly pediatric illness seen most often in children between the ages of 2 to 7 years (Solheim, 2018). Signs and symptoms include abrupt onset of high fever (> 39.5°C), sore throat with muffled voice, and inspiratory stridor; drooling due to dysphagia are the classic signs of epiglottitis, with increased risk of complete airway obstruction and respiratory failure. For this reason, examination of the throat must be avoided to prevent laryngospasm and subsequent respiratory collapse. The patient may also be in the sniffing position with neck extended or tripod position to maximize air entry. Diagnosis of epiglottitis is generally based on the history and physical examination. Radiographs (e.g., soft tissue x-rays of neck) may be performed depending on the severity of signs/symptoms (avoid if it will cause unnecessary agitation). Treatment of epiglottitis is focused on the maintenance of airway patency, thus preventing any agitation or anxiety is essential. Emergency airway equipment (e.g., endotracheal, cricothyrotomy) must be readily accessible at the bedside. High-flow humidified O2 should be administered via any method the patient will tolerate, in addition to nebulized racemic Epinephrine. Invasive treatment interventions (e.g., intravenous [IV] access) should be avoided until the airway is controlled or secured. Once the airway is secured, antibiotics, corticosteroids, and IV hydration can be administered. The patient should also be prepared for pediatric intensive care unit (PICU) admission (Andreoni, 2013; McCance & Huether, 2019). Visit the following site for a review of epiglottitis: https://www.khanacademy.org/science/health-and-medicine/respiratory-system-diseases/laryngeal-conditions/v/what-is-epiglottitis Foreign Body Aspiration Foreign body aspiration is more commonly seen in children less than 3 years of age. Because the airways of children are smaller than those of adults, they are more likely to suffer airway obstruction secondary to aspiration of a foreign object. In addition, the child's larynx is positioned more anterior and inferior than that of an adult, increasing the risk of aspiration. A thorough history-taking is essential when assessing the child suspected of choking on or aspirating a foreign object. If lodged in the larynx, for example, the child will not be able to speak and will likely be in respiratory distress. If the object bypasses the larynx and becomes lodged in lower airways, coughing or gagging may have occurred initially followed by apparent relief of symptoms. Respiratory infection, and possibly severe respiratory distress, may follow days later depending on where the object is situated. Treatment of foreign body aspiration focuses on basic life support (BLS) and choking algorithms. Depending on the object's material, chest x-rays (CXRs) may be of assistance, as well as bronchoscopy for removal. Of growing concern are button batteries, such as those found in small toys, remote controls, and watches, due to their toxic leakage (e.g., acid, lithium) and potentially catastrophic sequelae. The following table identifies characteristics of croup, epiglottitis, and foreign body aspiration. Table 1.2. Upper Airway Obstructions Parameter Croup Epiglottis Foreign Body Aspiration Age < 7 yrs, peaks at 6 mths - 3 yrs 2-7 yrs, peaks at 2 - 5 yrs Any age, most common in children aged 2-3 yrs Etiology Viral (rarely bacterial) Bacterial Mechanical Onset of symptoms Over 2-3 days Over a few hours (rapid progression) Acute Fever Low-grade High Absent Cough Barky Absent Present with partial and absent with complete obstruction Inspiratory stridor Present Present Present Upper respiratory infection signs and symptoms Present for several days Absent Absent Drooling Absent Usually present Possible

Head-to-Toe Assessment

The pediatric head-to-toe assessment is very similar to the assessment of an adult. It is important to inspect and palpate the head, neck, face, eyes, ears, nose, pelvis, genitalia, extremities, and posterior surfaces; and to inspect, palpate, and auscultate the chest and abdomen. Following, however, are some major differences: The assessment of children should be developmentally appropriate, based on knowledge of children's anatomical and physiological differences. For example, the nurse should be aware that a child's anterior fontanel can be palpated until approximately 2 years of age. Assessment findings should be compared with normal pediatric vital signs (see Table 1 below). An extensive neurologic examination should be based on the Pediatric Glasgow Coma Scale (GCS, see Table 3.2 below). Inspect Posterior Surfaces If appropriate, posterior surfaces should be assessed while maintaining full spinal precautions (log-roll). Examining posterior surfaces involves inspection and palpation as follows: Inspection Bleeding, abrasions, wounds, impaled objects, hematomas, ecchymosis Rashes, petechiae, purpura, edema Patterned injuries, injuries in various stages of healing Foreign bodies Palpation Spinal tenderness, deformity Costovertebral angle (CVA) tenderness, flank hematoma The secondary survey is followed by any necessary diagnostic procedures (e.g., radiography, computed tomography [CT]), as well as further planning and implementation (e.g., IV maintenance therapy, medications, specialist consult, prepare for admission) and ongoing assessment. The diagnostic and ongoing assessment components of this latter phase are components of the tertiary survey. Table 3.1 Normal Pediatric Vital Signs Age Normal Heart Rates (beats/minute) Normal Respiratory Rates (breaths/minute) Normal Blood Pressure (mmHg) Heart Rate (Awake) Heart Rate (Asleep) Systolic Diastolic Neonate 100-205 90-160 30-60 67-84 35-53 Infant 1-12 mths) 100-180 90-160 30-53 72-104 37-56 Toddler (1-2 yrs) 98-140 80-120 22-37 86-106 42-6 Preschooler (3-5 yrs) 80-120 65-100 20-28 89-112 46-72 School-age (6-11 yrs) 75-118 58-90 18-25 97-115 57-76 Preadolescent (10-11 yrs) 102-120 61-80 Adolescent (12-15 yrs) 60-100 50-90 12-20 110-131 64-83 (AHA, 2016) Table 3.2 Pediatric Glasgow Coma Scale (GCS) Score Best Eye Opening Child (≥ 1 yr) Infant (< 1 yr) 4 Spontaneously Spontaneously 3 To verbal command To shout, speech 2 To pain To pain 1 No response No response Best Motor Response Child (≥ 1 yr) Infant (< 1 yr) 6 Obeys Spontaneous movements 5 Localizes pain Withdraws to touch 4 Flexion - withdrawal Flexion - appropriate withdrawal 3 Flexion - abnormal (decorticate rigidity) Flexion - abnormal (decorticate rigidity) 2 Extension - decerebrate rigidity Extension - decerebrate rigidity 1 No response No response Best Verbal Response > 5 years 2-5 years 0-2 years 5 Oriented, converses Appropriate words/ phrases Cries appropriately, smiles, coos 4 Disoriented, converses Inappropriate words Cries and is consolable 3 Inappropriate words Persistent crying, screaming Persistent inappropriate crying, screaming 2 Incomprehensible sounds Moans, grunts to pain Moans, grunts to pain 1 No response No response No response

Primary Survey

The primary survey involves the ABCDE-FG mnemonic, specifically the evaluation of: Airway and Alertness (with cervical spine [c-spine] stabilization), Breathing and ventilation, Circulation, Disability, and Exposure and Environmental control, plus resuscitative adjuncts (Full set of vital signs/Family presence and Get adjuncts/Give comfort) (Steinman, 2020). Life-threatening conditions discovered during ABCDE are treated accordingly before moving on to the secondary survey. PRIMARY SURVEY Assessment Abnormal Findings Interventions Airway and Alertness (with spinal stabilization) Brief neurological assessment (AVPU) (helps determine if intervention required to open Airway, need for intubation) AVPU mnemonic: A-Alert V-responds to Verbal stimulus P-responds to Painful stimulus U-Unresponsive Vocalization - can the child talk or cry? What is the child's preferred position? Inspect Airway for: Tongue obstructing airway in unresponsive child Loose teeth or foreign objects (e.g., gum, small toys) in oropharynx Vomitus, bleeding, or other secretions in mouth Edema of lips and/or oral mucosa Drooling Dysphagia Airway sounds Airway compromise (see Assessment column for inspection findings) No sounds at all or audible sounds, such as stridor, gurgling, snoring If the child is awake and breathing, a tripod (sitting up, leaning forward, neck extended, head tilted up) or sniffing position may be assumed to maximize air entry. In the unresponsive child, flat surfaces and their large occipital area can cause c-spine to flex anteriorly, further compromising the airway. Stabilize the spine. In the alert child: Maintain the preferred position to maximize air entry or comfort If the child is unresponsive and/or unable to maintain spontaneous airway position, manually open the airway with the following techniques: Jaw thrust Head tilt/chin lift, if no trauma suspected Ensure neutral alignment of the c-spine by placing pad under shoulders to bring them in horizontal alignment with the external auditory meatus Suction oropharynx to remove debris, as well as nose of infant/young child as obstruction of nasal passages may cause respiratory distress If the child is unable to maintain an airway after positioning: Insert a nasopharyngeal airway (NPA) if the child is conscious with no facial trauma or skull fracture. If the child is unconscious, insert an oropharyngeal airway (OPA), maintain proper positioning of the head and jaw, and prepare for intubation. ***Do not rely on airway adjunct alone...may still need to open airway*** Breathing and Ventilation Once an airway has been established, assess for: Respirations Spontaneous? Rate, depth Symmetrical chest rise, fall Presence, quality of breath sounds Indicators of work of breathing (WOB) Jugular vein distension (JVD) and tracheal position Paradoxical respirations due to a flail segment Soft tissue and bony chest wall integrity Increased WOB Abnormal respiratory rate (RR) for age Anxious facial expression Nasal flaring Retractions: substernal, intercostal, supraclavicular, suprasternal Abnormal sounds: expiratory grunting, stridor, gurgling, snoring Accessory muscle use: tracheal tugging, abdominal 'seesaw' Head bobbing Position to facilitate breathing and comfort Administer oxygen (O2) via nasal cannula, simple or non-rebreather (NRB) mask with reservoir, blow-by (blow-by insufficient for critically-ill child) If the child has apnea or hypoventilation, assist with a bag-valve-mask (BVM) device with 100% O2 as indicated. Indications for intubation Apnea Compromised or ineffective ventilation Prolonged airway control GCS ≤ 8 and/or deteriorating neurologic status Severe maxillofacial fractures Inhalation injury Laryngeal/tracheal injury, neck hematoma Shock Caring for the intubated patient Cuffed ETTs now recommended for < 8 yrs of age (redesigned) Confirm tube placement Observe chest rise and fall Listen for breath sounds in chest and epigastric area Assess for improvement of skin/mucous membrane colour Assess end tidal carbon dioxide (CO2) Chest x-ray (CXR) Insert orogastric (OG) tube Administer medications Be prepared to use resuscitation equipment or assist with treatment of pneumothorax Document care provided Circulation Uncontrolled external hemorrhage Central and peripheral pulses, rate and quality (central = brachial in infant; carotid/femoral if > 1 yr) Skin colour, temperature Capillary refill ≤ 2 secs to evaluate peripheral perfusion Weak or absent peripheral pulses Pale, mottled, dusky, or cyanotic skin; skin tenting; skin abnormally warm or cold Capillary refill > 2 secs in warm, ambient environment Control uncontrolled bleeding with direct pressure Obtain intravenous (IV) access Use large bore catheter Consider intraosseous (IO) early...do not have to attempt IV first if compromised perfusion Administer 20 ml/kg warmed isotonic crystalloid boluses over 5-20 mins (10 ml/kg for neonate), as ordered by physician Reassess after each bolus (e.g., breath sounds, heart rate [HR], blood pressure [BP]) If 3 x boluses given, consider packed red blood cells (PRBCs); 10 ml/kg Cardiac compressions if HR < 60 beats/minute (bpm) and inadequate perfusion despite O2 Administer prescribed pharmacological agents Initiate defibrillation or cardioversion, if indicated Disability (neurologic status) Assess Pediatric Glasgow Coma Scale (GCS) Check with parents or caregivers to determine the child's normal status and compare findings. Assess pupil size, shape, equality, and reactivity to light (PERRL) Assess fontanel(s) in infants Altered level of consciousness (LOC) = cerebral hypoxia until proven otherwise Restless, anxiety, fussiness, crying, irritability, combativeness may be early signs of hypoxia Slow to arouse Inconsolable yet quiet when left alone (paradoxical irritability) Unaware of or unconcerned with surroundings Bulging, tense fontanel when upright and not crying; sunken in the presence of other signs of dehydration If this brief neurological assessment indicates a decreased LOC, conduct further investigation during secondary assessment Consider capillary blood glucose Consider moderate prophylactic hyperventilation (maintain PaCO2 30-35 mmHg) to decrease ICP....contraindicated in traumatic brain injury (TBI), may be considered if increased ICP despite interventions.... temporary rescue measure only Initiate pharmacologic therapy (e.g., dextrose, naloxone, mannitol, hypertonic saline), as ordered by physician Exposure and Environmental control Skin temperature, colour abnormalities Because of their larger body surface area to body weight ratio, children are at risk of rapidly losing body heat when exposed. Body temperatures may be too high (e.g., fever, head injuries, post-resuscitation) and cooling measures instituted. Skin that is clammy, cool, pale, mottled (note difference between trunk, extremities) Cyanosis Injuries, wounds Ecchymosis; petechiae, purpura Rashes Cold stress in critically ill or injured infants can increase metabolic demands, exacerbate the effects of hypoxia and hypoglycemia, and affect responses to resuscitation. Undress the child (one area at a time) to examine and identify any underlying injury or signs of illness, while maintaining c-spine precautions if appropriate. Provide age-appropriate explanations. Take accurate and frequent assessments of body temperature (e.g., rectal, core) Provide measures to maintain or raise body temperature: Warm blankets, especially around the head Overhead lights, radiant warmer Ambient environment Warmed IV fluids Warmed O2 If cooling is required: Remove excessive clothing Administer antipyretics Consider administering IV fluids at normal body temperature Consider need for transfer Full set of vital signs and Family presence Attempt to assess vital signs when child is calm (e.g., held by parent/ caregiver, use distraction) Respirations Rate, depth Assess before hands-on assessment (when calm) Pulses/HR Palpate brachial pulse as central pulse in infant, carotid in older child. Compare central (femoral) and peripheral (radial) pulses bilaterally for strength and equality on an uninjured extremity. Auscultate apical pulse as baseline in all pediatric patients. Pulse oximetry (SpO2) BP Appropriately-sized cuff covers 2/3 of the child's upper arm. Temperature Rectal for < 2 yrs (rectal [core] should be used for all critically-ill children) Avoid rectal if child is immunocompromised Oral for > 2 yrs Temporal artery, axillary also options Weight Obtain weight (kg) if possible If measured weight is not possible, ask parent the child's last weight Broselow tape to estimate the child's weight (although Broselow tape may underestimate child's weight due to prevalence of obesity) Weight Formulae Wt (kg) = 0 - 12 months: 4 + (0.5 x age in mths) 1 - 5 years: 8 + (2 x age in yrs) 6 - 12 years: 7 + (3 x age in yrs) Fast rates - tachycardia and tachypnea are often caused by fear, anxiety, agitation, fever, and crying, but they may also indicate hypoxia. Slow rates - bradycardia and bradypnea are abnormal findings in all infants and children. Large BP cuff gives false low readings. Small BP cuff gives false high readings. BP obtained from leg usually higher than arm. BP may be normal despite significant blood loss (up to 30%). Be aware that non-invasive monitors greatly affected by: crying, movement; poor perfusion; extreme values Children with fevers may be tachypneic, irritable, and sleepy. They may also exhibit higher temperatures due to immature central nervous system (CNS...thalamus). The following may be signs of serious illness in children: Rectal temp ≥ 38ºC in infants < 2-3 mths Rectal temp ≥ 40ºC in 3 mths - 2 yrs with no localized signs of infection Rectal temp < 36.1ºC in all pediatric patients Analyze vital signs to determine if they are within normal limits for the child's age: Use vital sign charts Estimate normal systolic BP (SBP) for child > 2 yrs of age by using the formula: Normal SBP = 90 + (2 x age in yrs) Lowest acceptable SBP limit in children > 2 yrs of age: Hypotension SBP = 70 + (2 x age in yrs) Continuous cardiorespiratory monitoring (if not already done so) Ensure pediatric alarm settings reflect pediatric parameters Obtain ECG Administer antipyretics, as ordered; institute warming measures, as needed Involve the family in all aspects of the care, including presence during resuscitation Have family members provide comfort measures for their child Assess the needs of the family, taking into consideration cultural variances Have someone give support and explanations Note any underlying psychosocial or cultural issues Get adjuncts and Give comfort LMNOP mnemonic Laboratory studies Monitor Nasogastric or orogastric tube Oxygenation and ventilation Pain assessment and management Pain level (use pediatric scales; assess behavioural cues)....environmental and emotional factors can also potentiate pain Physiologic and traumatic stress Laboratory studies, such as: Serum or capillary glucose (serum more accurate) Arterial blood gas (ABG) Type & screen (T & S) Lactate Monitor Cardiorespiratory monitoring, if not already done so Nasogastric or orogastric tube Relieve gastric pressure or empty contents Adjunct to endotracheal intubation Oxygenation and ventilation SpO2 Capnography (end-tidal CO2, ETCO2) Pain assessment and management Facilitate family presence Age-appropriate distraction Apply heated/cold packs, as needed Administer analgesia, as ordered Stabilize fractures, above/below injury Dress open wounds Other interventions that may be considered: Urinary catheter Normal Hourly Urine Output Infants / Young children: 1.5 - 2 ml/kg/hr (Diapers: 1 g = 1 ml urine) Older children / Adolescents: 1 ml/kg/hr (American Heart Association [AHA], 2016; Steinman, 2020)

cardioversion dosage

the correct initial dosage would be 0.5-1 J/kg, thus Meghan would require 2.5-5 J continued in SVT, the next intervention would be cardioversion at 10 J (c). The second cardioversion is delivered at 2.0 J/kg; however, the second dose can also be delivered in a progressively increasing fashion (i.e. 1.0-2.0 J/kg).

Exposure

Infants younger than 3 months of age are unable to shiver. They must burn brown fat in order to produce heat, which increases their metabolic rate. Pediatric patients have a higher body surface area compared to weight than adults; therefore, radiant heat loss is higher in infants and children. Their proportionally large heads account for much of the exposed surface area. Figure 1.7. Large surface area of head may cause significant heat loss in infants and children. Reprinted from Twinklebelle, by F. Luo, 2016

Part B: Select Gastrointestinal Emergencies

Gastroenteritis Gastroenteritis is defined as inflammation of the GI tract often related to infection, caused by viruses, bacteria, or parasites, with viruses being most common. Vomiting, diarrhea, and loss of appetite may lead to dehydration and hypovolemia. Oral rehydration is preferred over the parenteral route, if possible. See Module 2 above for a review of medical and nursing care related to pediatric patients with cardiovascular compromise (hypovolemia). Appendicitis Appendicitis is an inflammation of the vermiform appendix, and is the most common condition requiring surgical intervention in the pediatric population. Appendicitis is believed to be caused by obstruction of the appendiceal lumen by food particles, fecal matter, inflammation, or tumors. The inner walls of the appendix continue to secrete mucus which, in the presence of an obstruction, increases intraluminal pressure and obstructs venous and lymphatic drainage. Eventually, the mucus breaks down, allowing bacterial invasion and resultant edema and inflammation which further increases intraluminal pressure. The appendix becomes hypoxic due to decreased mucosal blood flow and, if left untreated, leads to gangrene and perforation. Initial clinical features: Complaints of abrupt-onset colicky, periumbilical (visceral) pain Inability to localize the pain Malaise Progressive clinical features (~12-24 hours after onset) Localized right lower quadrant (RLQ) pain (somatic) *** Steady ache; sense of constipation Aggravated by any movement ("Appy walk") Positive Rovsing's sign Nausea, vomiting; anorexia Fever (often does not present until late abscess/rupture)*** ***Younger children may not present with 'classic' symptoms (i.e., fever, RLQ pain)*** The following laboratory and diagnostic tests are frequently used to investigate suspected appendicitis: CBC/Diff Elevated WBC count is indicative of infection and inflammation but not specific to appendicitis (only 70% develop leukocytosis) Urinalysis Used to explore possible urinary tract cause of pain Leukocytosis of urine often suggests UTI Abnormal findings include pyuria, hematuria, and bacteriuria Ultrasound, x-ray, and computed tomography (CT) scan Observation in the ED Treatment involving nursing care includes administration of analgesia, antiemetics, and preoperative antibiotics; IV rehydration and correction of electrolyte imbalances; and patient education. The patient should also be kept NPO. Intussusception Intussusception involves the telescoping of a segment of bowel into the segment just distal to it (telescopes into itself essentially), occurring most often at the level of the ileocecal valve. It is most common among children aged 3-12 months (4 x more in boys), with a small risk of the condition developing within 2 weeks of Rotavirus immunization. Signs and symptoms of intussusception are variable, but often include the triad of abdominal pain (intermittent, colicky), "currant jelly" stool (bloody mucous), and a palpable sausage-shaped abdominal mass (< 50%); however, many children present with only one of these signs or symptoms. Bile emesis and abdominal distension may also be present. Lethargy and somnolence may be the most striking feature of the child with intussusception, although the cause of this alteration in mental status is not clearly understood. Traditionally, intussusception was diagnosed by barium enema, which is performed under fluoroscopy (isolates the obstruction because the contrast medium will not pass through the involved segment of bowel). In approximately 75% of cases, the barium enema will be therapeutic, causing sufficient hydrostatic pressure to reduce the obstruction by unfolding the involved bowel (> successful within the first 24 hours). Due to the risk of bowel perforation with this procedure, the diagnostic tool mainly used today is abdominal ultrasound, with barium enema reserved for reduction purposes in select patients. Those who do undergo barium enema should receive prophylactic antibiotics to decrease the risk of peritonitis. Ultrasound-guided hydrostatic reduction and surgical intervention (e.g., laparoscopy) may also be performed to reduce the telescoped bowel. Complications associated with intussusception rarely occur in the first 24 hours of symptom onset, but may include: dehydration, bowel obstruction and/or perforation, intestinal ischemia, sepsis, and shock. Intussusception may be viewed at the following website: https://myhealth.alberta.ca/health/pages/conditions.aspx?Hwid=hw43863 Volvulus/Malrotation Volvulus refers to the twisting of the intestine onto itself, which may result from malrotation of the gut due to a problem in embryologic development (Figure 4.1). Mid-gut volvulus (most common) may occur when there is a lack of mesenteric attachment and the loops of intestine twist onto themselves, producing an obstruction; the entire mid-gut twists around the superior mesenteric artery, leading to inadequate perfusion and possible bowel necrosis. Other specific sites of volvulus include the stomach, small intestine, cecum, transverse colon, and sigmoid colon. Volvulus is a surgical emergency that usually occurs at birth or during the first year of life. Figure 4.1. Volvulus of Intestine Regardless of the cause, volvulus results in two major conditions: bowel obstruction and loss of tissue perfusion/ischemia to the affected area. These sequelae also account for the signs/symptoms associated with volvulus: severe pain, gas/fluid accumulation, abdominal distension, melena, bilious emesis, and constipation. Older children tend to present with more atypical symptoms (e.g., chronic episodes of abdominal pain, due to volvulus and devolvulus) (ENA, 2014, p. 156). Diagnostic interventions are similar to those discussed above, as well as abdominal CT scan, stool guaiac, and laboratory studies (e.g., CBC/Diff, electrolytes, LFTs, BUN, Cr). Emergent surgical intervention is the treatment of choice for volvulus, which may involve bowel resection if the affected segment cannot be untwisted or salvaged. A colostomy or ileostomy may also be required post-operatively, as well as antibiotics to prevent peritonitis. Hypertrophic Pyloric Stenosis Hypertrophic pyloric stenosis (HPS) is the narrowing of the gastric outlet secondary to hypertrophy of the circular muscle of the pylorus, most commonly occurring at 2-8 weeks of age. An infant is more likely to have HPS if there is a family history of the condition. Projectile vomiting (without bile) after feeding is common, thus the infant is constantly hungry. Frequent emesis leads to weight loss, constipation, and dehydration. Upon palpation, an olive-shaped mass ('pyloric olive') may be felt in the right upper quadrant (RUQ), as well as observable abdominal peristaltic waves from left to right soon after feeding and just prior to vomiting. Management includes IV fluid replacement, gastric decompression (NG tube), and preparation for surgical pyloromyotomy. Antispasmodic medications administered before feedings may help to relax the pylorus sphincter muscle, but do not replace the 'gold standard' of surgical repair. Table 4.2 outlines potential causes for lower GI bleeding in pediatric patients. Intussusception (in toddlers) and gastroenteritis (in people of any age) are the most common causes of bloody stools or rectal bleeding. Table 4.2. Causes of Lower Gastrointestinal Bleeding in Pediatric Patients Neonate Toddler Any Age Hemorrhagic disease Hemorrhagic gastritis Necrotizing enterocolitis Milk allergy colitis, milk allergy Mid-gut volvulus Swallowed blood Anal fissure Esophagitis Intussusception Meckel's diverticulum Nodular lymphoid hyperplasia Acute gastritis, gastroenteritis Pharyngitis Candidiasis Epistaxis Infectious diarrhea Esophagitis Peptic ulcer Ingestion (e.g., acetylsalicylic acid [ASA], alcohol) Vascular malformation Bleeding disorders Post-surgical complications Post-dental procedures Mallory-Weiss tear/syndrome Preschool School-Age Juvenile polyps Hemolytic uremic syndrome (HUS) Esophageal varices Foreign body Peptic ulcer Crohn's disease Ulcerative colitis Hemorrhoids

Lower Airway Emergencies

Examples of lower airway emergencies include obstruction due to asthma, bronchiolitis, pneumonia, and foreign body aspiration. Less common causes of lower airway obstruction in children include tumours and pulmonary edema. This section also discusses the intubation of children. Asthma In both children and adults, asthma is a chronic disease with acute exacerbations characterized by hyperresponsiveness and an inflammatory/immune response involving MAST cells, eosinophils, neutrophils, T-lymphocytes, and macrophages. This inflammatory cascade results in hypertrophy of bronchial walls and bronchial smooth muscle contraction, causing narrowing of the airways. Bronchial mucosal edema also contributes to airway narrowing and resistance to airflow. Increased numbers of goblet cells produce excess mucous. The prevalence of asthma has increased, but morbidity and mortality have plateaued, indicating improved management. That being said, the most significant risk factors for ICU admission and respiratory arrest are: previous asthma-related hospitalization (ICU), endotracheal intubation/mechanical ventilation, deterioration despite daily systemic corticosteroid use, hospitalizations or ED visits for asthma within the last 12 months, using more than one canister of salbutamol per month, inadequate asthma action plan (or poor adherence), comorbidities, and socioeconomic and/or psychosocial concerns (Trottier et al., 2012, para. 6). _____________ Figure 1.1. - Comparison of Normal and Asthmatic Bronchioles Clinical manifestations of asthma vary with the degree of bronchospasm, but include wheezing, coughing, chest tightness, dyspnea/tachypnea, tachycardia, accessory muscle use, restlessness/anxiety, and prolonged expiration. The chest may also be hyperresonant on percussion. The person experiencing an asthma exacerbation will sit in a position that maximizes airflow, often upright with hunched shoulders and braced arms (tripod position). Younger children may also exhibit nasal flaring and grunting. As the patient decompensates, hypercapnia (hypoventilation) ensues which occurs more quickly in young children than adolescents. Asthma is classified according to severity (mild, moderate, severe), which involves the assessment of five key components: oxygen saturation on room air (SpO2 on RA); work of breathing, specifically suprasternal retraction and scalene muscle contraction; air entry; and wheezing . These parameters are then scored out of 12 indicating an overall Paediatric Respiratory Assessment Measure (PRAM) score (see Table 1 in Trottier et al., 2021 for how each parameter is assessed and scored). Nasal flaring, decreased activity level, difficulty feeding (infant), difficulty speaking in full sentences, and altered mental status (e.g., agitation, drowsiness, confusion) may also be considered as indicators of severity. Bedside spirometry is an objective tool that may also be helpful in assessing severity of airway obstruction; airflow is measured by forced expiratory volume in 1 second (FEV1). An FEV1 of > 80% predicted "normal" value for that patient is considered normal (Trottier et al., 2021). Review Trottier et al. (2021) for a comprehensive outline of each classification, as well as the treatment regimens (and side effects) discussed below. The main goal of therapy is to reverse hypoxemia/hypoxia (SpO2 < 92% on RA), open and clear the airways, and decrease inflammation. The mainstays of treatment include O2, bronchodilators, anticholinergics, and corticosteroids. Therapy is dependent upon the severity of asthma exacerbation as described above, and is as follows: Mild (PRAM 0 to 3, FEV1 > 70%) Salbutamol (metered-dose inhaler [MDI] with aerochamber, preferred route) Corticosteriods (oral route, particularly if high risk for severe exacerbation) Moderate (PRAM 4 to 7, FEV1 50%-70%) Keep patient calm, seated Salbutamol ± ipratropium (intermittent MDI with aerochamber, nebulizer if progressing to severe) Corticosteroids (oral route) Severe (PRAM 8 to 12, FEV1 < 50%) Keep patient calm, seated Salbutamol with ipratropium (intermittent nebulizer) PRAM 11 to 12 or poor response, consider: Keep NPO Continous cardiorespiratory monitoring; 1 to 2 IV access Salbutamol with ipratropium (continuous nebulizer) Corticosteroids IV Magnesium sulphate (MgSO4) IV CXR, arterial blood gas (ABG) PICU consultation Salbutamol IV Heliox-driven nebulized treatments (if refractory) Noninvasive ventilation until PICU arrives (ensure intubation equipment is readily accessible at bedside) Impending respiratory failure (confused, drowsy, lethargy, cyanotic, decreasing respiratory effort) Keep calm, seated; NPO O2 100% via non-rebreather (NRB) mask Ensure PICU consulted (need HELP!) Continue to support ventilation, as required Consider tension pneumothorax Continuous cardiorespiratory monitoring (as above); IV access x 2 (intraosseous [IO] if not accessible) ABG, serum electrolytes Support hemodynamics Salbutamol with ipratropium (continuous x 60 minutes); consider IV route Corticosteriods (IV, intramuscular [IM]) MgSO4 IV CXR, when possible Heliox-driven nebulized treatments (salbutamol, ketamine) Rapid sequence intubation (RSI) with ketamine IV (Trottier et al., 2021, Figure 1) Keep in mind that young children may not be able to perform these assessments, thus physical assessment findings are even more important in this population. Peak flow meters may be considered, but tend to be less reliable in children under 10 years of age. Note: CXRs and ABGs are not routinely recommended for the diagnosis of asthma exacerbation, unless complications are suspected (e.g., pneumothorax, impending respiratory failure). Patient education is paramount in preventing further exacerbations. Teaching should involve the child and family, and include information regarding the following: Avoidance of triggers Vaccinations (e.g., seasonal influenza) How and when to administer rescue medication and ICS (cornerstone of inflammatory prevention) How to clean/maintain inhaler device Preparation of a collaborative actin plan, to include where and when to seek medical attention (e.g., rescue medication needed > every 4 hours) Ensure follow-up with family physician to review asthma care plan (Trottier et al., 2021) Bronchiolitis Bronchiolitis is an acute viral infection of the bronchioles, most often due to RSV but coinfection with other viruses (e.g., rhinovirus, adenovirus, influenza) may occur in up to 30% of cases. Bronchiolitis involves bronchial obstruction, acute inflammation and edema, epithelial necrosis, and increased mucous production. It most commonly affects children < 2 years of age, and is the most common cause of hospital admission in children < 1 year of age (Friedman et al., 2014). Bronchiolitis is a clinical diagnosis, one that is arrived at based on history and physical examination. As with asthma, the severity of bronchiolitis varies from a mild upper respiratory tract infection (URTI) to impending respiratory failure. Typically, the infection presents with 2-3 days of viral prodromal signs/symptoms (e.g., fever, cough, rhinorrhea), progressing to more acute signs such as wheezing, tachypnea, and crackles on auscultation. Respiratory distress is characterized by grunting, nasal flaring, and accessory muscle use (e.g., retractions, abdominal breathing). If appropriate, diagnostic investigations may include the following: CXR: depends on severity, refractory to treatment Nasopharyngeal (NP) swabs (e.g., RSV): only if cohorting admitted patients) ABG: if impending respiratory failure suspected Blood culture & sensitivity (C&S): mainly if febrile infant < 2 mths of age Management of bronchiolitis involves the following interventions: O2: maintain SpO2 ≥ 90% Nebulized hypertonic saline (may decrease length of hospitalization) Hydration: if RR > 60 breaths/min, oral feeding may be ill-advised due to high aspiration risk; nasogastric (NG) or IV fluids are viable alternatives Nasal suctioning: if required, must be performed superficially and frequently Note: Salbutamol, corticosteroids, antibiotics/antivirals, and cool mist therapies are not recommended. Discharge criteria for the management of bronchiolitis in the home are: improved WOB without tachypnea; SpO2 ≥ 90% on RA, or stable enough for home O2 therapy; adequate oral feeding; provision (and understanding) of appropriate education; and adequate follow-up (e.g., family physician, pediatrician) (Friedman et al., 2014, p. 487). Foreign Body Aspiration As aforementioned, a foreign object may also become lodged in the lower airway. Following removal of the foreign body, it is imperative that the patient be monitored for airway patency, as subsequent inflammation may cause airway obstruction. See the above section on Upper Airway Emergencies for a review of foreign body aspiration. Characteristics of asthma, bronchiolitis, and foreign body aspiration are compared in the following table. Table 1.3. Comparison of Lower Airway Obstructions Asthma Bronchiolitis Foreign Body Aspiration Most commonly seen in children > 1 yr of age, particularly if family history or allergies Signs and symptoms vary with severity Wheezing, prolonged expiration (may be absent with severe distress) Dyspnea, tachypnea Cough Use of accessory muscles Tachycardia Anxiety, restlessness Upper respiratory symptoms (may precede exacerbation) Fever (if underlying infection) Most commonly seen in children < 2 yrs of age Wheezing, prolonged expiration Cough Tachypnea Tachycardia May or may not be febrile Gradual onset of respiratory distress May progress to apnea Anxiety, restlessness Mainly caused by RSV Can occur at any age, usually seen in children < 5 yrs of age Choking Coughing Abrupt onset Unilateral wheezing Afebrile Anxiety, restlessness History of playing with small objects, eating foods such as peanuts Pneumonia Acute infectious pneumonia is an inflammation of alveolar lung tissue that can lead to fluid accumulation and alveolar consolidation (see Figure 1.2 below). Pneumonia may result from a(n) defect in host defenses, virulent organism, overwhelming inoculation of the organism, or a combination of these factors. Pneumonia is classified as community-acquired pneumonia (CAP) or hospital-acquired pneumonia (HAP). Streptococcus pneumoniae is the most common pathogen in CAP across age groups; other pathogens include fungi, viruses (e.g., RSV), atypical bacteria (e.g., Mycoplasma pneumoniae), and methicillin-resistant Staphylococcus aureus (MRSA), of which the latter is occurring with increased incidence causing empyema (AHA, 2016). _______________ Figure 1.2 - Normal Alveoli Compared to Alveoli with Fluid/Consolidation Pneumonia alters the normal ratio of ventilation and perfusion (V/Q ratio) within the lung. Clinical manifestations may include dyspnea, tachypnea, productive cough if bacterial, crackles and/or rhonchi on auscultation, dullness on percussion, fever, and myalgias. In addition to history and physical examination, diagnostic interventions include CXR (diffuse/patchy infiltrates, consolidation), complete blood count with differential (CBC/Diff; elevated white blood cells [WBCs, bands), and ABGs if respiratory distress. The management of pneumonia includes antibiotics (if bacterial) and supportive care (e.g., hydration, antipyretics, nutritional support), as well as supplemental humidified O2, bronchodilators, and suctioning as needed. Spacing out nursing care and interventions, for example, may also help to decrease O2 consumption.

Case Study 2: Care of the Pediatric Patient with Lower Airway Obstruction

Four-year-old Angie Kubic is carried into the ED by her mother, who states that Angie has been sick for a week with a cough and fever. Mrs. Kubic is concerned because Angie started wheezing today; she has not been eating or drinking for several days and vomited three times this morning. Angie's mother states that Angie has had colds but is otherwise healthy and not taking any medications. You note that Angie is quiet, pale, and breathing rapidly, but not audibly wheezing. Angie allows you to assess her vital signs: Temp 37.4ºC (oral) HR 140 bpm RR 40 breaths/min BP 110/70 mmHg SpO2 91% on RA Weight 18 kg On the basis of the initial assessment (PAT), discuss whether Angie is well or unwell? Give reasons for your answer. Upon assessment, you note that although she has been vomiting, her mucous membranes are moist and her lips are slightly dry. What questions should be asked during the history-taking? Interpret Angie's vital signs. As you remove Angie's shirt to auscultate her chest, you observe intercostal and substernal retractions. Her lung sounds reveal decreased air entry to the lower lobes and bilateral expiratory wheezing. You discuss her case with the emergency physician, who examines Angie quickly and provides a tentative diagnosis of asthma. What other conditions may cause wheezing in children? (List the two (2) most common.) Considering Angie's signs and symptoms, would you rate her respiratory distress as mild, moderate, or severe? Give reasons for your answer. Upon reassessment, you note that Angie's SpO2 has dropped to 88%, most likely the result of ineffective gas exchange secondary to bronchospasm. You explain to Angie that you are giving her O2 to help her breathe and using a monitor to take a picture of her heart. What is the most appropriate method of oxygen delivery for Angie? (Choose the best answer.) Partial rebreather mask at 10-12 L/min Nasal cannula at 6 L/min Rebreather mask Laryngeal airway with 100% oxygen With supplemental O2, Angie's SpO2 rises to 90% but quickly drops back down to 88%. You notify the emergency physician, anticipating the need for more advanced airway interventions, and consult with a RT to advise him/her of the potential for airway and ventilatory compromise. List three (3) nursing interventions that serve to improve oxygenation. The physician orders a nebulizer treatment with a beta-2 agonist (Salbutamol). After the Salbutamol treatment, Angie is more alert and her breath sounds reveal increased bilateral high-pitched wheezing. Her RR has decreased slightly to 36 breaths/min and her SpO2 is 90%. Describe your interpretation of these assessment findings. Which of the following is a common side effect of nebulized Salbutamol? Blurred vision Thirst Tachycardia Drowsiness The physician orders a second Salbutamol treatment, ABGs, and an IV for the administration of fluid and Methylprednisolone (Solu-Medrol). Angie becomes agitated during the insertion of a 22-gauge IV catheter, but settles upon successful completion of the procedure. Your reassessment findings following the second Salbutamol treatment reveal that Angie is confused with a SpO2 of 86%, RR of 48 breaths/min, HR of 180 bpm, and diminished breath sounds throughout the lung fields. These assessment findings demonstrate what four (4) signs and symptoms of impending respiratory failure? After a third nebulizer treatment, Angie's condition has not improved. While you reassess Angie, you send a co-worker to ask an emergency physician to come immediately. You ask a third co-worker to organize emergency airway equipment and locate the RT. Angie measures to the 'white zone' on the Broselow Tape. With reference to this section of the Broselow Tape, indicate the types and sizes of emergency equipment required to intubate Angie. The RT is keeping the airway open and maintaining high-flow O2 while you are frequently reassessing Angie. A BVM is readily accessible should assisted ventilation be required. The emergency physician assesses Angie and decides that she will intubate via RSI. What drugs do you anticipate for Angie during RSI? Angie's ABG results are as follows: PaO2 70 mmHg PaCO2 49 mmHg pH 7.34 HCO3 22 mEq/L Normal ABG values PaO2 80-100 mmHg PaCO2 35-45 mmHg pH 7.35 -7.45 HCO3 22-26 mEq/L Based on Angie's ABG results, indicate whether the following statements are True or False. Give reasons for your answers. Angie needs more oxygen. The PaCO2 level is within normal limits. A bolus of bicarbonate should be given. Angie is alkalotic. After additional steroid and bronchodilator therapy, Angie's RR and depth of ventilations normalize. She appears fatigued but more alert. The cardiac monitor shows a normal sinus rhythm (NSR), and a SpO2 of 92% on 1 L O2. Air entry is adequate bilaterally, and expiratory wheezing continues throughout. Another set of ABGs is obtained, with the following results: PaO2 94 mmHg PaCO2 40 mmHg pH 7.40 HCO3 28 mEq/L What is the interpretation of the ABG results? Angie's RR is 36 breaths/min, and her HR is now 140 bpm. You are relieved to see Angie improving as you continue to observe her in the ED. Arrangements are being made to transfer her to a pediatric ward as soon as a bed is available. Using the PAT in your initial or across-the-room assessment, it is apparent that Angie's condition is urgent. You note that she is quiet and pale with rapid ventilations. She requires urgent treatment to prevent deterioration of her respiratory status. Angie's mother should be asked about the following: Onset of symptoms - this may help to differentiate asthma from foreign body aspiration, for example Family history - asthma tends to be familial in origin Triggers in the home - pets, smoking, environmental Prior history of wheezing - although she may not be diagnosed with asthma, her mother may have noticed wheezing on other occasions Elimination patterns (urine output, stools) Possible contact with others who are ill (e.g., URTIs) Angie's HR (140 bpm) is elevated, considering she is quiet and not active. She is also tachypneic at 40 breaths/min. Her BP is high-normal for her age at 110/70 mmHg and she is hypoxic with a SpO2 of 91% on RA. In addition to asthma, two common conditions that cause wheezing are bronchiolitis and aspiration of a foreign body into the lower airway. Angie's asthma is moderate-severe, and persistent. She is alert but quiet, with decreased air entry, retractions, and use of accessory muscles. This respiratory distress may progress to failure. The best option is a partial rebreather mask at 10-12 L/min (a). A nasal cannula (b) at 6 L/min will cause drying of the mucous membranes and is very uncomfortable at high flow rates. A rebreather mask (c) causes reinhalation of expired CO2. A laryngeal airway (d) is not indicated at this time. Consider the following nursing interventions to improve oxygenation: Place the patient in a sitting position or position of comfort to ensure maximal airway patency and tidal volume Breathing exercises (e.g., nasal breathing; slow, controlled rate) Request an order for a bronchodilator Prepare for advanced airway intervention Your assessment finds an alert patient, a positive finding indicating the brain is being adequately oxygenated. The increase in wheezing is a direct response to the actions of Salbutamol - opening of the airways and increasing air flow. We would expect to auscultate a 'noisier' chest following Salbutamol treatments; a 'quieter' chest following Salbutamol treatment is often a red flag for deterioration in the asthma patient. When combined with decreased LOC, increased WOB, cyanosis, or decreasing SpO2, it indicates inadequate airflow. The decreased RR may indicate that she is working less to oxygenate and ventilate, but it may also indicate fatigue. Angie requires frequent, ongoing assessment and interventions to ensure continued improvement. The correct answer is (c). Adrenergic agents, such as Salbutamol, may cause tachycardia, tremors, and restlessness. Angie is exhibiting the following signs and symptoms of impending respiratory failure: Altered LOC (confusion), increased agitation (but may also be normal for age) Tachypnea Decreased air entry Tachycardia Hypoxia Other signs/symptoms of respiratory failure in a child include, but are not limited to: Sternal, supraclavicular, or intercostal retractions Stridor Wheezing Nasal flaring Sitting in the "sniffing" position Respiratory acidosis; bradypnea Asymmetry or inadequate chest expansion Head bobbing, grunting Inability to vocalize/speak Ashen colour, cyanosis Unusual drooling Inability to recognize caregivers The emergency physician requires the following types and sizes of equipment to intubate Angie: ETT, 5.0 (cuffed/uncuffed) Stylet, 6 French (Fr) NG tube, 10 French Laryngoscope, 2 (straight blade), handle Suction catheter, 10 Fr ETCO2, adult BVM, child NRB mask, pediatric OPA, 60 mm; NPA, 22 Fr Medications for sedation and paralysis The exact drugs and order in which they are administered will vary by institution and physician preference - follow the protocols at your site. The drugs are given by a physician, or RN with physician at the bedside. Medications often used in pediatric RSI include: Premedications Atropine - blunts reflex bradycardia, limits secretions Lidocaine - blunts rise in ICP, cough reflex Fentanyl - analgesia, amnesia Pancuronium/Vecuronium (> 5 yrs of age, > 20 kg) - defasiculating agent if Succinylcholine used Induction agents (e.g., Etomidate, Ketamine, Midazolam, Propofol) Paralytic agents Depolarizing NMBA (Succinylcholine) Non-depolarizing NMBA (e.g., Rocuronium, Vecuronium, Pancuronium) ABG results (True/False) a. True. Angie needs more O2 (Angie is hypoxemic with a PaO2 of 70 mmHg). b. False. The PaCO2 level is within normal limits (Angie is hypercapneic with PaCO2 of 49 mmHg). c. False. A bolus of bicarbonate should be given (Angie's HCO3 level is within normal limits at 22 mEq/L). d. False. Angie is alkalotic (Angie's pH is slightly acidotic at 7.34). Angie's acid/base balance has normalized. She has fully compensated for her respiratory acidosis, as evidenced by the renal reabsorption of HCO3 and subsequent normal pH.

Case Study 3

Four-year-old Maria Milan is carried into the ED by her distraught father after she cut her hand carving a pumpkin. Her father is anxious to see a physician and resists having the triage nurse assess the injury. There is no visible blood on the dressing. Maria is screaming and crying hysterically. Choose the one correct answer for each of the following questions. 1. The triage nurse should: a. unwrap the hand and measure the length of the laceration. b. ask the father to register so the injury can be assessed while he is absent. c. perform a "non-invasive" assessment while beginning to establish rapport. d. show the father to the waiting area and suggest that he return when Maria is calmer. Maria and her father are taken into a treatment room an hour later. Mr. Milan is angry about the wait. 2. You should respond to the father's anger by: a. acknowledging his frustration and assuring him that his daughter is being treated in a timely manner. b. telling him if he does not want to wait, he is free to take his daughter elsewhere. c. explaining that his daughter's injury is not severe and he had to wait because there were more serious illnesses. d. asking him what he is angry about and try to determine the root of his anger. 3. Suggest three (3) ways to establish rapport with a distraught parent. 4. When you attempt to take the dressing off, Maria screams, "Don't! My blood will come out!" Maria's reaction is because she: a. thinks her skin holds her body together. b. is afraid her father will faint. c. thinks her heart will fall out. d. knows the injury is not severe. The physician examines the laceration and explains that Maria will have to be restrained for stitches. Mr. Milan asks, "Can't you just put her to sleep so it won't hurt?" 5. You respond to this request by: a. suggesting the physician order a sedative for Maria. b. explaining the purpose of physical restraint. c. requesting Mr. Milan leave the room. d. allowing Maria to sit on her father's lap during the cleaning and suturing. 6. You explain the procedure to Maria: a. simply, one step at a time as you proceed. b. in detail, summarizing the entire process. c. with advanced warning that the procedure might hurt. d. and tell her that if she holds still and doesn't cry, it won't hurt. Maria's father, although upset, manages to reassure and encourage Maria. 7. Choose the most helpful statement to use during the procedure. a. "This would not have happened if you had let your dad do the carving." b. "Do you have any pets? Tell me about them." c. "I know you are very brave and will not cry." d. "This needle will deaden your skin." e. "Fussing will upset your father." After the suturing is finished, Maria agrees that she would like to see her stitches. Mr. Milan is concerned that this will upset her. 8. Explain how you would handle this. Maria loves having a bandage on her hand! You praise Maria for being so brave and reward her with stickers and a popsicle. Mr. Milan thanks everyone for being patient. The answer is (c). It would be best to begin with a 'non-invasive' assessment and work on developing some rapport. Insisting on examining the hand (a) could cause distress and anger that could escalate. Asking the father to leave (b) or remain in the waiting area (d) separates the child and parent, which could cause increased anxiety and fear. The correct answer is (a) because in a crisis situation, it is best to begin by acknowledging the other person's feelings. Answer (b) is not in Maria's best interest, and (c) would not be advisable because the father believes his daughter's injury is very serious. Answer (d) is not appropriate because he has already expressed his concerns, and exploring other possible causes for his anger would not be helpful until his feelings have been acknowledged. Ways to establish rapport with a distraught parent: Praise the parent for doing a good job of caring for the child. Offer empathy and an opportunity to discuss his/her feelings and thoughts. Communicate concern and availability by telling parents you will check back periodically, and showing them how to inform staff if there are changes. The correct answer is (a). Maria is at the stage where she is concerned about body integrity. Preschoolers have an incomplete and distorted understanding of how the body works. They are interested in body surfaces and believe that the skin holds the blood and organs inside the body. Preschoolers often stop crying as soon as they have a bandage! The correct answer is (b). Explain that restraint is necessary to ensure the procedure is finished quickly and prevent further injury. Reassure the father that he may remain with his daughter, offering comfort, explanations, and reassurance. Sitting on a lap (d) is appropriate for an assessment but not for a procedure that is messy and requires dexterity. Maria does not need to be sedated at this time (a) because the procedure has not begun, and it is appropriate to try a non-pharmacological approach first. The correct answer is (a), to explain simply, one step at a time. Use appropriate, non-threatening communication to describe sensations (e.g., "measure," not "take" blood). It is not helpful to describe the entire process at once (b), nor is it helpful to give warnings well in advance (c). Preschoolers are verbal, but may be unwilling to talk. They frequently misinterpret common words and distort explanations, and may develop frightening fantasies at this age. Answer (d) sets Maria up for failure if she does cry, and unexpected pain may cause fear and loss of trust. Do not cover a child's face with sterile drapes or give the impression the child was 'bad' because she/he cried or struggled. The correct answer is (b) because it helps Maria focus on something other than fear and pain. Telling Maria she should have let her father do the carving (a) implies the injury was her fault. Telling her to be brave (c) may cause her to lose self-esteem if she cries. Saying the needle will "deaden" her skin (d) is likely to frighten her. Telling her that her father will be upset (e) may only heighten guilt and anxiety. Injury prevention education is inappropriate at this point. Techniques used in calming preschoolers and helping them cope with pain include: distraction (story, repeat words, deep breaths), parental comfort, simple explanations, no surprises about pain, helping to hold still, and reinforcement of positive coping behaviours. Explain to Mr. Milan that you can understand his concern, but that once a child is not in pain or afraid he/she will be naturally curious. If Maria did not want to look at the stitches, she would not have agreed to see them. Preschoolers often imagine things to be worse than they really are and have great concerns about their body integrity.

Case Study 11: Care of the Patient with a Musculoskeletal Emergency

Jamal, age 14, arrived to the ED via ambulance after a bicycle accident. While training for the Tour d'Alberta, he rode across railway tracks and his front bicycle wheel fell into the space between the tracks. Jamal is on O2 via nasal cannula at 2 L/min. A 22-gauge IV is in situ in his left forearm with NS infusing at 90 ml/hr. He is positioned supine on a spine board with a c-collar in place. He states that he did not hit his head and did not lose consciousness at the scene. He is complaining of severe pain in his right leg and right elbow. He thinks he weighs about 110 lbs (50 kg). Jamal's vital signs are as follows: HR 120 bpm RR 22 breaths/min BP 124/70 mmHg Temp 37.2°C (oral) SpO2 96% on RA What are the initial priority nursing assessments and interventions for Jamal? Interpret Jamal's vital signs. You accompany Jamal to radiology for x-rays (c-spine, pelvis, right femur/knee, right humerus/radius/ulna). The physician indicates that his c-spine is not fractured; however, he has a right femur fracture and a sprain of his right elbow. What is a sprain? How are sprains classified? List at least four (4) components of the nursing assessment for a sprain. How is a sprain treated? What are the signs and symptoms of a femoral fracture? List at least five (5) priority nursing interventions for the patient with a femoral fracture. Identify at least two (2) major complications of a femoral fracture. What teaching does a family caring for a child with an MSK injury require? Suggested Answers As with any trauma patient, the initial priorities of care for Jamal involve the ABCDEs. Once the ABCDEs are stabilized, he must be assessed for specific MSK trauma as follows: Deformity, shortening, rotation of limb(s) Edema, soft tissue injuries (e.g., lacerations, abrasions) Decreased range of motion (ROM) or reluctance to move limb(s) Neurovascular status (6 P's) Pain Pallor Pulselessness Paresthesia Paralysis Poikilothermia ***Pressure....the 7th P specific to compartment syndrome*** (ENA, 2014) Jamal is 14 yrs of age, thus his normal vital sign ranges are similar to those of adults. He is tachycardic and tachypneic, possibly due to pain, stress/fear/anxiety, and physical trauma (e.g., blood loss). He is normotensive, afebrile, and his SpO2 is within normal range. A sprain is the pull or tear of a ligament, commonly in the knee, ankle, or shoulder. Common mechanisms of injury include twisting movements, such as during sports activity, exercise, or falls, and wrenching forces resulting in more violent muscle contraction. Unlike strains, sprains are characterized by a significant amount of force. Mild (first-degree): microscopic tearing of the ligament with local tenderness, minimal swelling, and a stable joint. Moderate (second-degree): partial tearing of the ligament, joint instability, pain, swelling, and bruising. Severe (third-degree): full tearing or rupture of ligament; characterized by significant edema and tenderness, unable to weight bear Components of the nursing assessment for a sprain Determine severity of the injury, including possibility of a fracture Monitor neurovascular status (see 6 P's above) Assess ROM, amount of tenderness Keep patient as pain-free and comfortable as possible Elevate affected limb (heart-level) to reduce swelling. Ensure tensor bandages are not wrapped so tightly that they impair neurovascular status. Sprains and strains should be given immediate RICE treatment: rest, ice, compression, and elevation (heart level). During the first 24-48 hours, apply ice for 15-20 minutes at least 3 x day, wrap the area with a tensor bandage, and elevate the affected extremity. Mild (first-degree): treated for several days with RICE until the edema and pain diminish. Anti-inflammatory drugs are also used to decrease inflammation and control pain. Moderate (second-degree): often results in more pain, point tenderness, and edema. The patient is also likely unable to use the limb for a short period of time, thus compression bandage and crutches (light weight bearing) may also be necessary, in addition to interventions above. Severe (third-degree): stabilization (e.g., cast, splint) and crutches with no weight bearing; analgesia. Femoral fractures are usually secondary to major trauma. The patient has severe pain, inability to bear weight, deformity, edema, and angulation of the affected leg. Severe muscle spasms contribute to pain and shortening of the limb. The movement of bone fragments also causes crepitus over the fracture site. The patient may also be hypovolemic, depending on the amount of blood loss, thus hypotension and tachycardia may be present. Priority nursing interventions for the patient with a femoral fracture Assessment and stabilization of ABCDEs Assess the limb, paying careful attention to neurovascular status Associated injuries frequently occur, especially of the knee Establish IV access, ideally 2 x large bore sites Continuous cardiorespiratory monitoring Provide analgesia Determine the amount of volume replacement required (e.g., crystalloid, blood products) Apply, or assist with applying, traction or splint (e.g., Sager or Thomas) Maintain traction in proper alignment, assess for correct position and functioning of ropes, pulleys, and weights Prepare the patient for definitive traction, pin placement, or surgery The most significant complication of femoral fracture is shock secondary to hypovolemia. The average blood loss due to femur fracture in a child may range from 500-1000 ml, for example. This is more commonly seen in occult femur fractures (i.e., not visible, unsuspected) vs. isolated femur fractures (Wheeler & Carcillo, 2014). The patient with a femoral fracture must also be monitored for compartment syndrome due to accumulation of excess blood, edema, and debris within the compartments containing muscles and nerves. Further pain, muscle damage, or soft-tissue injury can occur when muscle spasms move the bone ends. The popliteal artery, as well as sciatic and perineal nerves, can also be damaged. Teaching related to caring for child with MSK injury Signs/Symptoms of neurovascular compromise Pain: if continues or increases beyond 48 hrs post-injury, child must be reassessed Application and loosening of bandages or splints Tensor bandage: regular removal, assessment of injured limb Avoid placing sharp objects inside cast/splint if itchy Personal care instructions (e.g., showering/bathing), keeping cast/splint dry Ambulation (crutches, mobility aids) Three-point gait When standing with crutches, place all of your weight on the good leg. Move the crutches and injured leg forward together. Bear the weight on the palms of the hands, then step forward onto the good leg. Stairs ("Up with the good, down with the bad") When going up stairs, place the 'good' leg on the higher step. When going down stairs, place the 'injured' leg on the lower step. Safety measures (e.g., footwear, slippery surfaces)

Case Study 4: Care of the Pediatric Patient with a Rapid Heart Rate

Mrs. Echols states that her 6-week-old baby, Meghan, doesn't "seem right" because she is fussy and not feeding well, and her breathing is harder than normal. You observe a pale infant with a rapid RR and feel a pulse that is weak and too fast to count. Her hands are cold, and she responds to painful stimuli by crying. What does this assessment indicate? You remove Meghan's sleeper and assess her ABCDEs, wondering if she has decreased tissue perfusion related to her HR and rhythm. You place her head in the sniffing position to maximize air entry, administer O2 via partial rebreather mask at 10 L/min, and ensure she is warm. You place Meghan on the monitor and discover the rate and rhythm on the rhythm strip below. You page the emergency physician and show him the rhythm strip. What is the cardiac rhythm? a. Sinus tachycardia (ST) b. Ventricular tachycardia (VT) c. Supraventricular tachycardia (SVT) d. Extra systole While a 12-lead ECG is being performed, you note that Meghan's RR is 60 breaths/min and her SpO2 is 91%. You replace the O2 mask with a partial NRB mask to increase her inspired O2 concentration, and a colleague starts an infusion of NS via a 22-gauge IV catheter in her left antecubital (AC) vein. Meghan has a capillary refill of 4 seconds and a SBP of 70 mmHg. Interpret Meghan's vital signs. Cardiac output is decreased with a very rapid HR. The rapidly contracting ventricles do not have adequate time to fill, thus less blood is available to be pumped to the periphery. It is important to convert the rapid rhythm to a sinus rhythm with the use of Adenosine. After one dose of Adenosine, Meghan starts to cry more vigorously, her perfusion has improved, and the emergency physician orders laboratory investigations, repeat 12-lead ECG, CXR, and pediatric cardiology consult. Meghan's vital signs are now as follows: HR 148 bpm BP 80 mmHg (via palpation) Temp 37.5°C (rectal) RR 40 breaths/min Capillary refill 2 secs Skin Mottling has decreased What do the vital signs and rhythm strip indicate? Meghan weighs 5 kg. If she required cardioversion, what would the correct initial dosage be? a. 2.5-5 joules b. 5-10 joules c. 10-15 joules d. 15-20 joules If Meghan continued in SVT following cardioversion, the next intervention would be: a. pharmacologic agents. b. vagal maneuvers. c. cardioversion at 10 joules. d. defibrillation at 5 joules. Mrs. Echols is very concerned that her baby may need an electrical shock. What information do you give her? The cardiologist recommends transfer to a pediatric tertiary care facility, and arrangements are made for Meghan to be transported via pediatric transport team. The transport team arrives, and you give them Meghan's CXR, laboratory reports, and a copy of her chart with your nurses' notes. Mrs. Echols remains with Meghan during transport, and you reassure her that the receiving hospital is well-equipped to handle their special needs. Meghan's condition is emergent and warrants immediate assessment. She is a pale 6-week-old infant with a rapid HR and RR, and decreased peripheral perfusion as evidenced by cold hands. She may also have an altered LOC, as she responds to painful stimuli only. In addition, her mother claims she is "not right" (i.e., fussy). With a rate of 260 bpm and a narrow QRS complex, the rhythm is SVT (c). This rhythm disturbance results from accessory pathway re-entry of electrical impulses above the AV node, AV nodal re-entry, or ectopic atrial focus. Meghan is tachycardic at 260 bpm and borderline hypotensive with a SBP of 70 mmHg. She is also tachypneic at 60 breaths/min, and hypoxic with a SpO2 of 91% on RA. Her capillary refill is delayed at 4 seconds. In SVT, CO is often markedly decreased due to insufficient cardiac filling times, which can progress to congestive heart failure (CHF) and shock relatively quickly. Meghan's vital signs have stabilized (normotensive, no longer tachycardic or tachypneic); she is afebrile at 37.5°C, however this must be monitored closely (infants < 3 mths are deemed 'febrile' at > 38°C). Meghan's peripheral perfusion (thus CO) has also improved, as evidenced by capillary refill of 2 seconds, less mottling, and vigorous crying. The correct initial dosage would be 0.5-1 J/kg, thus Meghan would require 2.5-5 J (a). If Meghan continued in SVT, the next intervention would be cardioversion at 10 J (c). The second cardioversion is delivered at 2.0 J/kg; however, the second dose can also be delivered in a progressively increasing fashion (i.e. 1.0-2.0 J/kg). You tell Mrs. Echols you can understand her concern and explain that it was very encouraging that her baby converted to a more stable rhythm with the Adenosine. If cardioversion is required, the patient is given sedation before the procedure (if possible), followed by a very small dose of shock (quite often only one is needed). Once a baby's rhythm reverts to sinus rhythm, medication is usually given to control the rate. The major concern in SVT is to determine and treat the underlying cause of the rapid HR. A pediatric cardiologist will conduct testing and advise her of the treatment.

Case Study 7: Care of the Pediatric Patient with a Toxic Ingestion

Mrs. Jones arrives in the ED carrying her 3-year-old granddaughter and frantically asking for someone to help her. Apparently, Anna had been quietly playing until she was found lying on the floor with the contents of her grandmother's purse. Anna opens her eyes only momentarily as you call her name and whimpers but does not resist being taken from her grandmother's arms. Her skin feels warm, face is flushed, and limbs are flaccid. Her breathing is unlaboured and regular, and capillary refill is less than 2 seconds. What is your first priority? a. Obtain a complete history b. Assess neurological status and vital signs c. Obtain baseline laboratory tests d. Examine the purse contents A colleague begins a complete neurological assessment while the following vital signs are obtained: BP 95/60 mmHg HR 160 bpm, regular RR 26 breaths/min, unlaboured SpO2 95% on RA Temp 37.0°C (rectal) Weight 15 kg How would you interpret Anna's vital signs? Your colleague informs you that Anna is becoming less responsive. Her speech is confused, and she opens her eyes and withdraws her limbs to painful stimuli only. Her pupils are PERRL at 3 mm. What is Anna's Glasgow Coma Scale score? Your initial nursing priorities include ensuring a patent airway, administering O2 to correct or prevent cerebral hypoxia, vigilant monitoring of Anna's cardiopulmonary and neurological status, and establishing IV access. You administer O2, and ask a colleague to set up IV equipment and get the emergency physician. While placing Anna on the cardiopulmonary monitor, you obtain more history from her grandmother, who tells you Anna has not been ill or sustained any kind of injury within the last few days. At about 1130 hrs today, Mrs. Jones noticed Anna was "too quiet" and went to check on her. She thought Anna had fallen asleep on the floor and then realized she was hard to wake up and did not seem to recognize her. Anna's clothing is removed, and she is covered with a light sheet. Your colleague informs you the physician is on his way. List at least three (3) factors in the history that suggest the possibility of a toxic ingestion. What additional history must be obtained at this time? (Choose all that apply.) a. Details about the lack of supervision b. Information about the type and amount of medication c. Estimated length of time that has elapsed d. Prehospital treatment Given the possibility of a toxic ingestion, you ask Anna's grandmother if any medications were in her purse. She says the purse contained her prescription medications - Propranolol (Inderal), Hydrochlorothiazide (HCTZ; Hydrodiuril), and Amitriptyline (Elavil). The only pills missing are from the Amitriptyline bottle; you check the prescription and find that five 50 mg tablets of Amitriptyline are unaccounted for. You inform the emergency physician of your assessment. As he begins his examination, he orders a 12-lead ECG, CT scan of the head, laboratory investigations (ABG, CBC, BUN, Cr, glucose, electrolytes, LFTs, ammonia, toxicology screen, and urinalysis). You consult the Poison Control Centre about TCA toxicity. You are informed that a dose of 10-20 mg/kg can be serious, and a dose of 35-50 mg/kg can be fatal. Is Anna's ingestion enough to cause serious side effects? Why or why not? The physician orders a dose of NaHCO3 to be added to the IV, along with the prescribed rate of IV and solution; continuous cardiopulmonary monitoring; 12-lead ECG with any rhythm changes; and serial ABGs to ensure alkalosis. Anna's mother arrives and takes her daughter's hand to begin comforting her. She is very upset that Anna does not respond. You reassess Anna's neurological status by shaking her shoulders and calling out her name; she does not open her eyes, so you apply pressure to her nail bed. She opens her eyes, pulls away, and moans. Her pupils remain PERRL at 3 mm. You assign her a GCS score of 8/15 and recognize the potential for ineffective airway clearance. As soon as you notify the physician of the change, he decides to intubate. After sedating Anna, the physician inserts an endotracheal tube (ETT) with little difficulty and leaves to write an order for charcoal and intravenous lipid emulsion (ILE). Anna's airway is now protected from aspiration, and she is still breathing on her own. The RT stays with Anna to manage her airway and breathing. Anna's current vital signs are as follows: GCS 8/15 Pupils PERRL at 3 mm BP 92/54 mmHg HR 150 bpm, irregular RR 20 breaths/min, intubated, spontaneous breathing Temp 36.5°C (rectal) SpO2 98% on 100% O2 The cardiac monitor is showing sinus tachycardia with PVCs. You repeat the 12-lead ECG, ensure the QRS has not widened, and ask another nurse to bring you the supplies for administering charcoal and ILE. View the following scan of the Broselow tape used to measure Anna. Browslow Tape Which tube would be selected for charcoal administration, as well as an adjunct for ET intubation (gastric decompression)? a. 10 Fr NG tube b. 8 Fr feeding tube c. 28 Fr OG tube d. 14 Fr Swan-Ganz catheter If a child has an altered LOC or is extremely uncooperative, charcoal is administered slowly via NG tube (patent airway must be present). Slow administration helps reduce the risk of vomiting and subsequent aspiration. Multiple doses are usually unwarranted for TCA toxicity (Tintinalli et al., 2011). If the child is alert and cooperative, a NG tube is likely not necessary. The charcoal can be mixed with flavored drinks (e.g., juice, chocolate milk) in an opaque container, and the child is encouraged to drink the mixture through a straw. Ensure the child is given a time limit to facilitate the likelihood of its successful toxin-binding, as charcoal is most effective when administered within 1 hour of toxic ingestion. Describe the mechanisms of action of NaHCO3 and ILE with regards to TCA toxicity and rationale for their use. Anna is given charcoal 25 g, and she opens her eyes at the sound of her mother's voice. The physician informs them that Anna will be transferred to the PICU to be monitored and observed for complications. Her grandmother is upset and crying. This may be an appropriate time to start discussing safety issues. What are some safety issues you would discuss with the family? Suggested answers The correct answer is (b). Assuring the adequacy of airway, breathing, and circulation is always the first and most important step in the management of pediatric neurologic emergencies. The signs and symptoms of pediatric neurologic emergencies can be caused by and/or aggravated by unstable ABCs. Anna is tachycardic, but the remainder of her vital signs are within normal limits (although her SpO2 is borderline low at 95% on RA). Anna's GCS score would be 10/15, calculated as follows: Opens eyes to painful stimuli 2 Withdraws to pain 4 Inappropriate verbal response 4 Total score 10 Any of the following factors in the history suggest the possibility of a toxic ingestion: Sudden onset of altered LOC with no history of preceding trauma or illness Physical findings of altered LOC without obvious injury or illness Found lying on the floor with grandmother's purse contents Persistent tachycardia without respiratory distress Age and developmental level of the child The correct answers are (b), (c), and (d). The assessment of a toxic ingestion must be thorough and rapid. It should focus on gaining as much information as possible about the type and amount of toxic exposure and any subsequent changes in the child's clinical status. Emergency management of the child's presenting symptoms always takes precedence over identification of the poison. If possible, obtain the container of the toxic substance to assist with identifying the type and amount ingested. If it is necessary to estimate, assume the largest amount possible. Counseling the family or caregiver about lack of supervision is inappropriate at this time. Education regarding child safety is important though, and should be conducted when the child stabilizes or is discharged home, for example. The correct answer is Yes. Given Anna's weight (15 kg), a dose of 150 to 300 mg (10-20 mg/kg) could cause serious side effects. Anna may have ingested five 50 mg tablets, a total of 250 mg. The correct answer is (a). An appropriately-sized NG tube is recommended for charcoal administration and gastric decompression, which is size 10 Fr for Anna who measures in the white area of the Broselow tape (15 kg). One of the most significant adverse effects of TCAs is myocardial Na+ channel blockade, which results in slowed Na+ influx into myocardial cells during phase 0 of the action potential. Sequelae include conduction delays, ventricular dysrhythmias, and hypotension. NaHCO3 serves to increase the alkalinity of the blood, thus enhancing TCA-protein binding and 'uncouples' drug from Na+ channels (Doherty, Cave, & Harvey, 2012; McGill University, 2015). It also provides a 'Na+-load' which may compensate for Na+ blockade (Doherty et al., 2012). Other than for total parental nutrition (TPN) purposes, ILE is most often used as a reversal agent in local anesthetic toxicity (e.g., Bupivicaine). There is a growing body of research supporting its use in severe and/or intractable TCA toxicity as well, mainly due to TCA's highly lipophilic properties. ILE's benefits are mainly related to TCAs' cardiotoxic effects. Three proposed mechanisms of action of ILE are as follows: Lipophilic toxins are sequestered in lipids within the intravascular space ('lipid sink' theory) Restores fatty acid transport to cardiac mitochondria Direct activation of cardiac calcium (Ca2+) channels, resulting in increased cellular Ca2+ and cardiac performance (Doherty et al., 2012; Hellig et al., 2012; Hendron et al., 2011). Any of the following could be incorporated into a teaching moment: Child-proof containers and safe keeping of all medicines Use of child-resistant cupboard locks Safe storage of toxic household products Posting of the Poison Control Centre number in a central place More vigilant supervision in "high-risk" environments, such as farm buildings, seniors' homes, and sites with accessible alcohol and drugs

Case Study 2

Mrs. Thompson brings her 2-year-old son, Neil, to the ED claiming he has "put something up his nose." When you arrive, you observe an attentive Mrs. Thompson putting away a pencil that her son has managed to get out of her purse. You observe that Neil is playful, curious, and does not appear to be in any distress. Based on your knowledge of developmental stages, explain why Mrs. Thompson's assumption that there is something in her son's nose is likely correct. As you attempt to perform your assessment, Neil clings to his mother, cries, and resists being examined. Is this behaviour developmentally appropriate? Why or why not? A brief examination reveals unilateral purulent drainage and a foreign body lodged in the posterior nasal cavity. While you are preparing the equipment for removal, Mrs. Thompson asks how she can be helpful during the procedure. What do you tell her? As the procedure begins, Neil frantically starts to scream, kick, and bite. Which of the following may be helpful? (Select all that apply). a. Stopping the procedure briefly and planning a coordinated approach b. Encouraging Mrs. Thompson to reassure Neil c. Asking the physician to order a sedative d. Asking Neil's mother to leave the room e. Asking the mother to do most of the restraining f. Using a 'mummy' restraint The physician removes a tiny plastic toy from the nasal cavity and hands the diaphoretic, tired toddler back to his mother. Mrs. Thompson's assumption is likely correct because toddlers are developing autonomy and are learning to explore their world. They experiment with all new objects of interest by banging, tasting, and feeling them, as well as seeing what they will do and where they will go. They also like to explore their bodies and may attempt to fit small objects into body orifices. This behaviour is very common at this age because toddlers are frightened of being separated from familiar people, places, or things. To help them feel secure, they often use transitional objects, such as a small blanket or toy. Toddlers are generally able to comprehend very simple explanations and requests. They are very egocentric, generally seeing things from their point of view. Negativism and insistence prevail, and they cooperate about half of the time. You might give Mrs. Thompson the following information: Inform Mrs. Thompson that it is helpful for her to stay in the room to calm and soothe her child. If she is distressed, unable to help, or asks to leave, she should be allowed to do so without judgment. Holding Neil on her lap during the assessment may help decrease the vulnerability a child associates with being supine. It is important for her to understand that her major role is to offer comfort and distraction. It is also helpful for the child to hold a security object and to receive explanations that are simple, non-threatening, and gentle but firm. Because the child may become upset if a parent is obviously distressed, it is important to reassure the family and gain their trust by being open, calm, quiet, and confident. The correct answers are (a), (b), (c), and (f). It might be helpful to stop briefly and plan a coordinated approach, allowing time for Neil to calm down before gently restraining him with a 'mummy' wrap. The most effective person to help Neil cope is a calm, soothing mother, but the nurse can also help by remaining calm and restraining him as gently as possible and for only as long as necessary.

Seizures

Pediatric seizures may be caused by one or more of the following: Fever "A febrile seizure is a seizure accompanied by fever (temperature ≥ 100.4°F or 38.0°C by any method), without central nervous system infection, that occurs in infants and children 6 through 60 months of age." (Subcommittee on Febrile Seizures, 2011, p. 390) Peak incidence ranges from 8-20 mths 30% chance of recurrence Traumatic head injury Nontraumatic increased ICP CNS infections, congenital disorders, or tumours Ingestion of or exposure to toxins Anoxia Fluid and electrolyte imbalances Metabolic disorders Epilepsy A thorough clinical examination must be performed to determine the cause of a pediatric seizure. Common characteristics of febrile seizures are listed below: Febrile seizures usually occur between the ages of 6 mths - 3 yrs. They are associated with acute fever of < 24 hrs in duration. The child has no history of neurological problems or CNS disease. The seizures are usually nonfocal. Seizure activity typically ends within 15 mins. The family history is positive for febrile seizures and negative for nonfebrile seizures. At least two questions should be asked about all children presenting with seizures: Does the child have a CNS infection or other severe underlying disease that must be promptly diagnosed and treated? Is the child at risk for recurrent seizures or epilepsy? If so, diagnosis and interventions are necessary before the child leaves the ED. Risk factors for epilepsy include the following: Neurological or developmental abnormality before a seizure Initial seizure < 12 mths of age, with relatively low fever Initial febrile seizure that is complex (often with focal attributes), multiple, or > 15 mins in duration History of nonfebrile seizures in either parent The following nursing priorities are appropriate for a seizing child: Ensure a patent airway. Give O2, position the child on his/her side, prepare to suction if necessary, insert oral airway if possible, and avoid placing potentially occlusive objects into the mouth (e.g., padded tongue blades). Protect from physical injury. Do not restrain, move the patient away from dangerous areas, loosen restrictive clothing. Maintain constant observation and assessment. Note colour, seizure activity, vital signs, LOC including pupil reaction and eye position, incontinence, injuries secondary to the seizure, and events after the seizure. Prepare to administer anticonvulsant medications. Establish IV access, correctly calculate and administer pediatric doses of anticonvulsant medications. Nurses should document the following observations of a child who is having a seizure: Time frames Initial onset and duration of seizure Movement Part of body affected first, progression to other body parts, type of seizure (e.g., spastic, tonic, tremors) Post-seizure tone - flaccid, hypertonic Lack of movement in any body part Respiratory status Airway patency RR and effort, length of apnea Secretions - amount, colour, need for suctioning Mental status LOC, duration of changes, postictal status Pupils, movement of the eyes Behaviour Elimination Incontinence Oral injuries Postictal condition of the tongue and mucous membranes Blood in the saliva or sputum Parents are encouraged to stay with their child unless they physically interfere with the medical care provided. The teamwork, coupled with explanations about what is happening, may decrease the parents' anxiety. If circumstances warrant it, another staff member, pastoral care or social worker could be called in to support the parents.

Initial Assessment

The initial assessment is an organized, systematic approach to patient assessment that ensures subtle signs of physiological compromise are not missed. This approach can be utilized and adapted to virtually any nursing care environment. The initial assessment consists of the primary survey and secondary survey. It is important to remember that components of the primary and secondary surveys often occur simultaneously.

Breathing

Adults utilize the diaphragm and intercostal muscles for breathing. Infants have poorly developed intercostal muscles and are more dependent on effective use of the diaphragm to maintain ventilation. If an infant is in respiratory distress, measures to enhance the function of the diaphragm will in turn enhance ventilation. Infants breathe primarily by using their abdominal muscles, thus any pressure on the diaphragm might hamper breathing. The diaphragm is positioned more horizontally until approximately 12 years of age. The chest wall is thinner in children than in adults; therefore, sounds are transmitted more easily and are louder. The metabolic rate of infants is approximately twice that of adults. Because of this, anything that increases metabolic demands will in turn increase respiratory workload in order to increase intake of oxygen and removal of metabolic wastes such as carbon dioxide. Because of their size, children have less compensatory reserve than adults and can experience sudden fatigue and decompensation. The pediatric skeleton has not fully ossified, thus children have a softer and more pliable chest wall than adults. Their ribs and sternum may retract, which diminishes the size of their thoracic cavity and provides less protection to abdominal organs. Children's ribs are oriented more horizontally from the sternum, and the thoracic cage lacks the ability to expand laterally (towards the sides of the chest). Development of collateral connections in the lungs is incomplete; therefore, children have limited ability to compensate in the event of an airway obstruction. Elastic tissue of the adult lung helps to support alveoli and keep them open at the end of expiration; however, pediatric patients have less elastic and collagenous tissue in their lungs, which also results in lower tidal volumes. The pediatric patient has fewer alveoli, which are smaller in size than those of adults; therefore, there is less surface area for gas exchange between the alveoli and capillaries.

Age-Appropriate Assessment

Keep in mind the following characteristics of children at various stages of development as you prepare for the initial assessment. Infants In early infancy, warmth and comfort are keys to keeping a child content. In later infancy, when separation anxiety becomes an important issue, allow the infant to stay on the parent's lap if possible, and use toys for distraction. If the illness or injury is not urgent, begin with a brief history-taking, move to the physical assessment, and perform the most traumatic portions of the examination last. Toddlers A toddler may be negative and uncooperative. If appropriate, have the parent hold the child while a quick physical assessment and history are completed. Preschoolers These visits can be very pleasant, as preschoolers are usually cooperative and can participate in their examination. It is best to do invasive portions of the examination last. School-Age Children All procedures must be explained in age-appropriate language. Allow school-age children to participate in the examination as much as possible. Respect their privacy. Adolescents Adolescents are acutely aware of their appearance, which is often a source of concern or fear related to being perceived as "different". They may be prone to mood swings or depression, and may engage in risk-taking activity. They also require concrete explanations as much as possible. The following sections describe basic elements of pediatric assessment: initial impression, primary and secondary surveys, and history-taking techniques.

Case Study 5

Miles Nygano, a 16-year-old male, is brought to the ED by his father with complaints of scrotal pain. Miles is grimacing and guarded in his movements, as well as being irritable with his father, as he enters a treatment room. Miles is reluctant to provide details about the pain and to undress. You should handle this situation by: assuring him that he will be able to cover himself with a sheet. reassuring him that nurses are accustomed to seeing patients undressed. telling him that this is routine and he will have to remove his clothes. acknowledging his discomfort and telling him you have a brother his age so he does not need to feel uncomfortable. Miles finally cooperates, and with a diagnosis of testicular torsion, a surgical consult is requested. Miles senses the urgency of his problem and asks if he is going to die. Why is this a typical question for an adolescent to ask? The surgeon arrives and explains the urgent need for surgery. Miles asks appropriate questions, but Mr. Nygano becomes upset and refuses to sign the consent form. You should respond by: a. explaining that this is a life-threatening situation and there is no choice. b. asking about his reasons for objecting. c. explaining that you can obtain a court order if he is unwilling to sign. d. encouraging Miles to convince his father that he needs surgery. You discover that Mrs. Nygano died from surgical complications and Mr. Nygano feels guilty, lonely, and inadequate. The necessity of surgery is emphasized, concerns are listened to, and finally Mr. Nygano consents. While his father is gone, Miles asks if a torsion can be caused by sexual activity and how the surgery will affect his future relationships. You should respond to his concerns by: a. assuring him he is good-looking and telling him not to worry. b. telling him testicular torsion is not related to sexual activity. c. encouraging him to discuss his fears and giving him some written material on testicular torsion. d. telling him he is too young to be in a sexual relationship. You transport Miles to surgery, and Mr. Nygano thanks you for being so patient. The correct answer is (a) because allowing Miles as much privacy as possible will lessen his anxiety. The acute onset of scrotal pain is an emergent situation, and one option in this circumstance may be to have a male physician see him immediately. Insisting or ordering (b) and reassuring him you have "seen it before" (c) will only increase his resistance. Mentioning your own family (d) or experiences may further isolate him because these statements do not acknowledge his feelings. This is a typical question for an adolescent to ask because any type of illness or injury can be of great concern to adolescents. They are very aware of their bodies, and when cues from the environment suggest a serious situation they may worry about dying. Miles is worried, and providing a non-judgmental atmosphere in which he can discuss concerns may help to alleviate some of his anxiety. Encouragement to discuss and ask questions combined with simple explanations about anatomy, physiology, and treatment of testicular torsion is helpful. The correct answer is (b), as it would be most helpful to discuss Mr. Nygano's concerns with him. Listen, empathize, and attempt to diffuse negative feelings and resolve conflict peacefully. The parent who has feelings of helplessness, fear, or anxiety may respond with resistance, anger, or demands. Rather than opposing Mr. Nygano, it may help to address the underlying concerns and emotions. This approach is more likely to result in a better outcome than trying to coerce him into signing the consent form (a) and (c) or precipitating a confrontation (d) between father and son. Concerns reflect normal adolescent anxiety about physical disfigurement and attractiveness. It is common at Miles's age to be concerned about sexual identity and body image. Sexual curiosity and experimentation are normal parts of development. Adolescents often interpret illness or injury as related to something they have done. Their knowledge of physical function and illness is frequently incomplete and distorted. Answers (a) and (b) are incorrect because it is not helpful to ignore, judge, or belittle an adolescent's concerns or fears. Answer (d) is also incorrect: a judgmental attitude only increases guilt and fear. Providing a non-judgmental atmosphere (c) that allows Miles to ask questions freely is the best way to provide information about sexuality and sexual practices.

4/2/1 rule

4 ml x 10 kg for the first 10 kg, plus 2 ml x 1 kg for the second 10 kg

The Adolescent in the Emergency Department

Age Developmental Milestones Common Fears Approach Health Promotion Adolescence (13-18 years) Growth spurts last about 2 yrs. Girls gain about 8¾ cm/yr; boys gain about 10 cm/yr. Continued sexual maturation; skin becomes oily; changes in body fat, musculature. Has a sense of invulnerability. Is self-conscious, wants privacy, and is self-centred. Has a need for belonging, desire for independence or freedom to do what he/she wants and with whom. Becomes logical, abstract, idealistic in thinking. This is the age of 'all or nothing', 'black or white', 'here and now' reactions. Teenagers generally do not think in terms of the future or 'shades of gray'. Although they understand what is occurring, they are prone to underplay major injuries or 'catastrophize' minor injuries. Overriding concerns are about being normal, belonging, being left out or socially isolated. Concerned about sexuality (gender role identity, sexual preference), prospects for future happiness and well-being. May resist treatment if modesty is compromised. May fear they will inherit shortcomings (alcoholism, mental illness). With increased rate of homicide, suicide, and motor vehicle collisions [MVCs], they face fear of death. Should be allowed to choose who they want to have with them during an examination; they separate easily from parents. Ask probing questions about pregnancy, sexual activity, drug or mental health issues in private. Recognize their modesty and maximize privacy by drawing curtains and closing doors. Remain calm, empathetic, and friendly in your interactions while respecting their opinions and feelings. Do not engage in a power struggle - you will not win! Promote responsible decision-making and moderation of risk taking by helping adolescents recognize that for every action there is a consequence. Help them recognize the dangers of drugs, speed, unprotected sex, fights, etc. Encourage them to talk about their feelings rather than 'suffer in silence'. Feelings of invincibility in normal circumstances are typical but dangerous. There can be conflict, rebellion, and 'breaking away' in the parent-child relationship.

The School-Age Child in the Emergency Department

Age Developmental Milestones Common Fears Approach Health Promotion School age (6-12 years) Average yearly weight gain is 1-2 kg, height gain is 1.25 cm. Greater variation in physical development in children of same age. Puberty in girls begins with breast buds (9-11 yrs) and menarche (10-16 yrs); growth spurt starts anywhere from 7½ to 11½ yrs. Puberty in boys begins with scrotal enlargement, lengthening of penis (10-12 yrs); growth spurt starts anywhere from 10½ to 16 yrs. Has increased gross motor ability, strength; masters reading, writing, arithmetic; capable of cooperative play; likes competitive games; engages in concrete thinking; is aware of bodily functions, internal organs. Pain Response Nonverbal Passive resistance (rigid body, clenched teeth or fists), regression. Verbal Denial of pain. Identifies location and intensity in more detail; includes physical and psychological aspects. Being unable to compete in school, sports, or play. Interruptions to daily routine. Being rejected by peers. Works hard to make friends, follow rules. Tries to please authority figures (parents, teachers, coaches, health care providers). Hero worship is very common. Even minor surgery can trigger fear of disfigurement or death. Need to appear strong; may portray false bravado, denial of pain. Talk directly to the child. When talking to the parents, include the child. Explain all procedures, tell child what to expect. Refer to benefits/positive outcomes of the procedure. Ask the child if she/he has questions. Make sure the child has understood what you have said. (Ask the child to repeat it, as misconceptions are common.) Use positive phrasing, such as "You can...." Treat all questions, fears, concerns seriously. Promote sports safety (proper conditioning, use of equipment). Encourage child to avoid overexertion, false bravado of "playing with pain." Foster good sportsmanship. Encourage use of seat belts, helmets (bicycling, in-line skating, skate-boarding). Reinforce water safety (swimming with buddy, avoiding deep water, proper diving techniques). Caution against inadequately frozen bodies of water. Advise proper supervision.

temperature measurement

An axillary temperature is easy, noninvasive and safe, and can be taken in all age groups; however, it is not a reliable assessment of core temperature in children. Its accuracy and reliability is not only dependent upon precise placement over the axillary artery, but also environmental conditions (Leduc & Woods, 2000/2015). Axillary measurement is approximately 1°C lower than rectal, and 0.5°C lower than oral route. A tympanic temperature also has the advantage of being obtained quickly. According to the Canadian Pediatric Society (CPS), In contrast with other sites of temperature measurement, the TM's [tympanic membrane's] blood supply is very similar in temperature and location to the blood bathing the hypothalamus, the site of the body's thermoregulatory centre. It is, therefore, an ideal location for core temperature estimation." (Leduc & Woods, 2000/2015, para. 10). Despite its advantages, the CPS acknowledges that its accuracy is adversely affected by several factors: incorrect technique (e.g., failing to straighten the ear canal), inappropriate probe size, ear canal structure, and manufacture design. However, tympanic measurement is not significantly affected by crying, AOM, or cerumen. Oral thermometry measures the temperature of (sub)lingual arteries. It would not be an appropriate method for a 7-week-old infant, and is generally reserved for older and more compliant children. Its accuracy is dependent upon a firm mouth seal and remaining under the tongue for 3-4 minutes, and is generally regarded as being between that of axillary and rectal measurement (roughly 0.5°C lower than rectal). Parents may also inquire about pacifier thermometers; they should be informed that they are also not as accurate as rectal measurement (Leduc & Woods, 2000/2015). Temporal artery temperature measurement is a relatively new method of estimating core temperature. It has been shown to be more accurate than the tympanic method, and is less invasive than rectal measurement. It is generally regarded as an effective febrile 'screening' tool; however, it should not be relied upon for the assessment of a seriously ill child (less accurate than rectal) (Leduc & Woods, 2000/2015).Rectal temperature is generally considered to be the 'gold standard' of reliable measurement of core body temperature, but its accuracy is known to be affected by local blood supply and the presence of stool. There may also be a delay in the change in rectal temperature following an increase or decrease in core temperature. This method is also not without risk: it must be used cautiously in children less than 2 years of age due to potential bowel perforation, and is contraindicated in patients who are immunocompromised or have bowel disease. Strict decontamination techniques must be adhered to in order to prevent the spread of pathogens, and parents are often not comfortable with this method (Leduc & Woods, 2000/2015). General guidelines for the depth of probe insertion are as follows: ½ inch for newborns; ¾ inch for infants; and 1 inch for preschoolers/older children (Potts & Mandleco, 2012)

Guide to Emergent Pediatric Presentations

Children who present with any of the following signs/symptoms are considered Emergent and must be seen immediately by a health care professional: Apnea, bradypnea Choking Drooling Stridor, grunting, sternal retractions Irregular respirations SpO2 < 92% on room air (RA) Peak expiratory flow rate (PEFR) < 40% baseline Cyanosis Delayed capillary refill Diaphoresis Tachycardia, bradycardia Hypotension Petechiae or purpura Hypothermia Hyperthermia (> 38°C if < 3 mths or immunocompromised; > 38.5°C otherwise) Altered LOC (GCS 10-13) Hypotonia (floppy child, unable to support head) Pain, acute-severe (8-10/10) Inconsolable infant, abnormal vital signs Decreased urine output Decreased tearing Sunken or bulging fontanels History of chronic illness Child acting as surrogate parent History of family crisis Risk of flight Signs of maltreatment Return to ED Newborn delivered in, or on way in, to ED

Initial Impression

Direct observation, prior to touching the child, is a useful technique for triage and bedside nurses. This 'as you approach the child' assessment is sometimes referred to as the Pediatric Assessment Triangle (PAT) because it prompts the nurse to ask, "Does the child 'look well' or 'look sick'?". The PAT has proven to be an effective tool to assist emergency health care providers objectively identify those children who are at risk of, or who are showing warning signs of, clinical deterioration (Horeczko, Enriquez, McGrath, Gausche-Hill, & Lewis, 2013). This tool helps to identify the need for reprioritization from ABC to C-ABC, such as in cardiopulmonary arrest or external hemorrhage (Steinman, 2020). The PAT, or 'across-the-room assessment' should be completed within 3-5 seconds, and includes the following: Appearance - evaluation of Tone, Interactiveness, Consolability, Look/gaze, Speech or cry (TICLS mnemonic) Breathing - presence of nasal flaring, abnormal airway sounds, particular position of comfort, altered respiratory rate, intercostal and/or subcostal retraction Circulation - whether the child is pale, cyanotic, and/or mottled Figure 3.1. Pediatric Assessment Triangle and its components. Reprinted from "The pediatric assessment triangle: Accuracy of its application by nurses in the triage of children," by T. Horeczko, B. Enriquez, N. E. McGrath, M. Gausche-Hill and R. J. Lewis, 2013, Journal of Emergency Nursing, 39(2), p. 183 (https://doi.org/10.1016/j.jen.2011.12.020).

Secondary Survey

If primary survey findings do not indicate immediate intervention is required, or once vital functions have been stabilized, initial assessment continues with the secondary survey. The secondary survey consists of HI, specifically History/Head-to-toe assessment and Inspect posterior surfaces (Steinman, 2020).

Case Study 8: Care of the Pediatric Patient with Gastroenteritis

Ian Miller is a 4-year-old child carried into the ED by his father. Ian has been complaining of a stomach ache, and has watery diarrhea and a slight fever. His parents are concerned because he has "taken nothing" but apple juice and water for 2 days. Ian opens his eyes briefly and falls back to sleep. He has good air movement with unlaboured, slightly fast respirations. His hands are slightly cool, skin is pale with normal turgor, and capillary refill is 2 seconds. Ian needs further examination to determine the cause of the pain and assess his fluid and electrolyte status. What are three (3) signs of dehydration in a preschool-aged child? You reassure the parents that they were correct in giving Ian fluids and that he does not appear to be very dehydrated. His ABCDEs are stable, and vital signs are taken while his father is holding him. When you put the BP cuff on his arm, Ian begins to cry and pull his legs up. His vital signs are as follows: HR 140 bpm RR 38 breaths/min Temp 37.4°C (rectal) BP 100/60 mmHg SpO2 96% on RA Indicate whether Ian's vital signs are normal, high, or low for a child his age. a. Heart rate b. Respiratory rate c. Temperature d. Blood pressure e. SpO2 Ian is undressed, and the physician observes that his skin tone is normal, with no injury or rashes. The secondary assessment involves a thorough focused examination of the abdomen and careful gathering of more history. The possibility of an acute abdomen must be considered in all pediatric patients with abdominal pain until a thorough examination has been completed. Which findings would be indicative of acute appendicitis? Choose all that apply. a. Vomiting, watery diarrhea b. Pain with movement c. Intermittent right lower quadrant pain d. Constant periumbilical pain The mnemonic CIAMPEDS is used to gather more information from the Millers. You ask if Ian has allergies, and they tell you he has none. He is taking no medications. They tell you that Ian is generally a healthy child who has had the occasional normal childhood illness, including chicken pox. They say he developed vomiting, diarrhea, and a temperature of 39°C while at preschool 3 days ago. Although his vomiting stopped after 24 hours, he has had persistent watery, non-bloody diarrhea 8-10 times/day. They tried to keep him on a liquid diet with only clear liquids and no solid food over the past 48 hours. Although Ian was well enough to be playing with his building blocks this morning, he started complaining of abdominal pain this afternoon. Mrs. Miller tells you for the second time about Ian's friend who had appendicitis and asks if her son will need to be operated on tonight. What do you tell Mrs. Miller? a. "It is a good sign that Ian can move around and fall asleep, and it is doubtful that he will need surgery." b. "Tell me more about the friend's symptoms." c. "Don't worry. Ian will need x-rays and lab work before any decisions are made." d. "Tell me what worries you about Ian, and then I will give you some information about what will happen in the next hour or so." You tell Dr. Brown what you have learned about this family and give him a brief synopsis of the history and clinical presentation. Dr. Brown asks you if Ian has had any pain with urination or a sore throat. Other than appendicitis, what diseases or syndromes are associated with abdominal pain in children? (Choose all that apply.) a. Urinary tract infection b. Toxic ingestion c. Testicular torsion d. Appendicitis e. Intussusception f. Strangulated hernia g. Constipation You anticipate (or have completed according to protocol) laboratory investigations, urinalysis, x-rays (anteroposterior/lateral chest, flat plate, upright abdominal films). Ian is toilet-trained, thus a midstream urine specimen is obtained. Following the emergency physician's examination and review of the laboratory studies, Ian is discharged with the diagnosis of gastroenteritis and mild dehydration. Ian's parents seem relieved that he does not have appendicitis. They think his diarrhea has slowed down since he came to the ED. Mrs. Miller wonders what Ian should be eating and drinking tonight and tomorrow. You tell her current recommendations are to 'feed through' gastroenteritis in all but very severe cases of diarrhea. Pediatric glucose and electrolyte solutions (e.g., Pedialyte) have the best carbohydrate-to-sodium ratio for replenishing fluid losses. They are superior to clear liquids, such as juices, which have inadequate sodium, high sugar, and high osmolality that worsen diarrhea by encouraging movement of fluid into the GI tract. The Millers still seem nervous, and you give them written discharge instructions on oral rehydration for diarrhea and abdominal pain. You suggest that they call their family physician to arrange follow-up in the next few days. Suggested answers Signs of dehydration in a preschooler Tachycardia Cool hands and feet Lack of tears Concentrated low urine output Thirst Dry mucous membranes Fatigue, lethargy Ian's vital signs a. HR 140 bpm - tachycardia (normal HR is 80-120 for 4 yr old; however, given his 'sleepy' state he is tachycardic at 140 bpm) b. RR 38 breaths/min - tachypnea (compensating for decreased dehydration, decreased perfusion) c. Temp 37.4ºC (rectal) - normal d. BP 100/60 mm Hg- normal e. SpO2 - 96% on RA - within normal limits, although low-normal (perhaps early indication of worsening tissue perfusion) The correct answers are (b) and (d). In the early stages, pain related to appendicitis is often periumbilical, constant, and exacerbated by movement, with later localization to the RLQ. Intermittent pain (c) or pain that moves from one location to another, with vomiting and diarrhea (a), is more characteristic of gastroenteritis. Vomiting with appendicitis generally follows the onset of pain. A lack of appetite (anorexia) is not specific to appendicitis, although the child who is hungry for pizza or hot dogs probably does not have an acute abdomen. The correct answer is (d). When a parent appears to be very concerned, it is best to discuss specific fears related to the child. It is also helpful to offer information about what will happen in the very near future. Give concrete information, and encourage the parent to take one step at a time. It would not be appropriate to offer predictions, comparisons, or false reassurances (a, b, c). Appendicitis, especially in preschool-aged children, often has a rapid and insidious onset. All of the diseases and syndromes listed may be associated with abdominal pain in children. It is often difficult to determine the cause of abdominal pain in a child. To complicate matters even further, preverbal children cannot tell you where the pain is located, and older children may not be able to distinguish nausea or general malaise from abdominal pain. Many childhood diseases and syndromes present with abdominal pain, and several investigations may be required to differentiate between possible causes. The Case Study identifies some of the laboratory studies that would be ordered - urinalysis, CBC/Diff, and abdominal and chest x-rays. For an older child, a pregnancy test would be ordered if appropriate.

The Infant in the Emergency Department

Infancy is a time of rapid growth and psychosocial development. In a span of 12 months, infants triple their birth weight, increase their height by 50%, begin to walk, start using monosyllabic words, and become increasingly independent. Parents in the ED should be asked about their infant's activity levels and feeding patterns. Age Developmental Milestones Common Fears Approach Health Promotion Infant (birth to 1 year) Weight Regains birth weight by 3 wks Doubles by 5 mths Triples by 12 mths As infants learn to recognize their parents and family members, they become anxious if separated from family or approached by strangers. Environmental stressors include bright lights, sudden movements, loud sounds, or excessive temperatures. Infants may not like to be undressed, especially if associated with something unpleasant, and may struggle. If possible, have the parent do the undressing. Once infants are accustomed to their own environment, they may be wary of unfamiliar surroundings. Becomes very anxious if separated from family or approached by strangers. It is normal for the infant not to want to come to you or even look at you. Infants like movement, but shaking can cause brain injury. Speak in a lighter tone of voice. Provide sensory stimulation by rocking, holding, cuddling. Always support head. Keep the child and family together as much as possible. Separation from familiar faces and voices frightens the child. Assess infants and young children on a parent' s lap when possible. Offer distractions, such as an age-appropriate toy on which the child can focus attention. Have warm ambient temperature to prevent chilling, promote comfort. Warm hands, stethoscope. Check to see if the child's favourite blanket or toy is available prior to initiating treatments. 6-8 months Sleeps, feeds at regular times Transfers objects hand to hand Sits independently Rolls from back to stomach Babbles, laughs Looks in direction of new sound Responds to own name Likes rattles, squeeze toys, songs 9 months Uses pincer grasp Sits, stands with support Attempts to move by crawling Babbles different sounds Understands short instructions Likes books with short sentences and simple pictures, toys to bang together Commonly 'makes strange' with new people. 12 months Understands simple requests, questions Chatters Sits, crawls easily Stands, walks with support Seeks comfort by reaching for parent Likes books with simple pictures and short sentences, stacking toys, rhymes, clapping Pain Responses Increased HR, respiratory rate (RR), pallor, diaphoresis and dilated pupils Crying, unable to anticipate pain Respond in generalized manner Older infants have more localized response.

Part A: Toxic Ingestion

Infants tend to explore their world by putting anything and everything into their mouths, an intense curiosity that may lead to toxic ingestion. Young children (< 5 years of age) have the highest incidence of toxic ingestion compared to other age groups (Beier & Vandenbranden, 2012), although the incidence of accidental poisonings in the home has decreased due to improved child-proof containers, safe storage, public education, and federal regulations. Clinical manifestations of toxic ingestion vary depending on the substance ingested, thus identification of the substance is key to diagnosis and treatment. Treatment will vary according to the following factors: Gender Weight Substance ingested Amount ingested Information from the container label Oral intake since ingestion of substance Following stabilization of the ABCDEs, the management focus shifts to decreasing the absorption of the ingested substance and preventing complications. Routine gastric decontamination (e.g., gastric lavage, [activated] charcoal) is no longer recommended in the pediatric population, and should only be used in severe cases. Cathartic agents (e.g., Sorbitol) are often used in conjunction with charcoal; however, they are contraindicated in children < 5 years of age. In addition, inducing vomiting with syrup of ipecac administration is also no longer recommended (Tintinalli et al., 2011). Potential antidotes and pharmacologic treatment for specific toxins are identified below in Table 4.1. Table 4.1. Toxins and Antidotes/Pharmacologic Treatment Toxin Antidote/Pharmacologic Treatment Acetaminophen N-Acetylcysteine (NAC, Mucomyst) Methanol (e.g., windshield wiper fluid) Ethylene glycol (e.g., antifreeze) Ethanol, Fomepizole Benzodiazepines Flumazenil Opioids Naloxone (Narcan) Iron Deferoxamine (Desferal) Lead No antidote....chelation therapy Ethylenediaminetetraacetic acid (EDTA) Tricyclic antidepressants (TCAs) No antidote....multi-faceted treatment Fluid resuscitation Acidosis: NaHCO3 Cardiac dysrhythmias: NaHCO3, magnesium sulphate [MgSO4], Lidocaine, hypertonic saline, Phenytoin, lipid emulsion ASA No antidote....multi-faceted treatment Acidosis: NaHCO3 Electrolyte imbalance: potassium (K+) Hypoglycemia: Dextrose 25%

Meningitis

Meningitis is an inflammation of the meninges (protective membranes lining the brain and spinal cord), most often caused by a bacterial or viral infection. The age of the child is the most significant predictor of pathogen: neonate - group B Streptococcus, older child/adolescent - Neisseria meningitidis, Haemophilus influenzae. Vaccines for the latter pathogens have greatly reduced the incidence of bacterial meningitis. Viral meningitis, also referred to as aseptic meningitis, is usually self-limiting and less likely to result in serious morbidity or mortality. Bacterial meningitis is associated with significantly higher morbidity and mortality if not diagnosed early and treated promptly. Table 3.2. Clinical Manifestations of Meningitis in Infants and Children Infants Children Normothermia, hyper- or hypothermia Apnea Bulging fontanel Vomiting, diarrhea Poor sucking and feeding Paradoxical irritability Lethargy Seizures Rash (petechiae, purpura) Infants with meningitis seldom develop nuchal rigidity, and those < 3 months of age may be afebrile. Fever and chills Headache Nuchal rigidity Vomiting Altered mental status Seizures Rash (petechiae, purpura) Children > 2 years of age usually experience nuchal rigidity and headache with meningitis. Note: A purpuric or petechial rash on a child who has flu-like symptoms requires immediate medical intervention. This rash is characteristic of bacterial sepsis related to N. meningitidis, also known as meningococcemia. The bacteria invade and damage the dermal layer of the skin, release toxins, and cause widespread vascular injury. This vascular injury results in perivascular hemorrhage, presenting as purpura which is indicative of bleeding under the skin (does not blanch). Sequelae include seizures, hearing loss, DIC, vasculitis, pericarditis, and septic shock, to name a few. Please refer to the following website to view a meningococcal rash: http://www.phac-aspc.gc.ca/im/vpd-mev/meningococcal/professionals-professionnels-eng.php Medical and nursing management of the patient with meningitis, regardless of the cause, focuses on supporting the airway and ventilation, as well as maintaining hemodynamic stability and serial monitoring of neurological status. In the case of suspected bacterial meningitis, antibiotics should be administered after the LP is completed; however, if the LP is delayed for any reason, antibiotics should be given immediately (Le Saux, 2014). Symptomatic management may also required for the patient with meningitis, such as anticonvulsants, corticosteroids, and antipyretics, as well as ensuring appropriate isolation precautions are in place.

Case Study 9: Care of the Pediatric Patient with Intussusception

Sixteen-month-old Julia Hart is brought to the ED by her grandmother, who is concerned because she has been mostly sleeping since breakfast, intermittently crying inconsolably, and has vomited once. Julia is currently sleeping in her grandmother's arms, and you observe that she is of average size, her respirations are normal with no increased WOB, and her colour is pale. You decide Julia's condition is urgent when she starts to cry, vomits bile-coloured emesis, and draws her knees up to her chest. Her vital signs are as follows: HR 140 bpm RR 30 breaths/min BP 95/58 mmHg Temp 37.4°C (rectal) SpO2 97% on RA Capillary refill < 2 secs Julia opens her eyes to voice, withdraws to pain, and has an irritable cry. Her pupils are PERRL at 3 mm. Indicate whether Julia's vital signs are normal or abnormal for her age. a. Heart rate b. Respiratory rate c. Blood pressure d. Temperature e. SpO2 Julia's hands and feet feel warm to the touch. Her weight is 11 kg. When placed on the scale, she begins to cry and is inconsolable even when given back to her grandmother to be comforted. You apply O2 via nasal cannula at 1 litre and notify the emergency physician of Julia's urgent need for attention, While gathering a brief CIAMPEDS history from Mrs. Hart, you conduct a quick head-to-toe examination. You assess Julia's GCS score as 11/15 and note that she winces during your abdominal assessment. You hear bowel sounds and feel a sausage-shaped mass in her RUQ. Julia is a healthy child with no allergies, taking no medications, and her immunizations are up-to-date. She has not eaten today and was found listless in her crib. Mrs. Hart states Julia has "not hit her head or gotten into anything." You are concerned because this may be a surgical emergency and Julia keeps pulling out the nasal prongs. The emergency physician is involved in stabilizing an elderly patient with a cardiac rhythm disturbance. Which sign is most indicative of a surgical emergency? a. Paradoxical irritability b. Vomiting bile c. Intermittent abdominal pain d. Lack of fever Based on the ABCDEs, what would your first three (3) priorities be at this time? (List them in order of importance). a. Obtaining an order for analgesic b. Establishing IV access c. Obtaining blood for laboratory studies d. Assessing ABGs e. Reassessing ABCDEs f. Administering O2 via a method that is tolerated by patient You leave momentarily to gather some equipment. When you return, Mrs. Hart shows you a basin containing bile-coloured emesis and tells you Julia has vomited twice since you left. Your reassessment reveals a radial pulse weaker than the femoral, cool feet, capillary refill of 4 seconds, and HR of 155 bpm. What do these reassessment findings indicate? You explain to Mrs. Hart that you are going to start an IV to give Julia some fluid. You use a 22-gauge IV catheter in the dorsum of her hand and draw blood for CBC, glucose, electrolytes, and liver function analysis before establishing the IV. You also obtain a capillary glucose of 5 mmol/L. The normal value for a child this age is 3.3-5.5 mmol/L (Wilson & Hockenberry, 2012, p. 330). The emergency physician orders a NS fluid bolus to be infused over 20 minutes. You decide how much fluid Julia should receive over 20 minutes and verify this amount with the physician. What is the appropriate amount of fluid bolus for this patient? a. 500 ml b. 220 ml c. 110 ml d. 250 ml Which of the following cause circulating volume depletion? (Choose all that apply.) a. External blood loss b. Occult bleeding in the abdomen c. Vomiting d. Fluid shift into the interstitium While you are ensuring the IV is properly secured, Julia has a large bloody stool. As you are cleaning her up, you realize how serious her condition is. You estimate she has lost about 80 ml of bright red blood in the stool. You know that Julia's fluid replacement may require about three times this estimated loss, with very careful reassessment after each bolus. Which of the following statements about intussusception are True? (Choose all that apply.) a. It is reduced (untwisted) by barium enema. b. It always requires a bowel resection. c. It may present with currant jelly-like stools. d. It involves a malrotation (twisting) of the small bowel. After a second bolus of fluid, Julia's perfusion is much improved. Her skin appears less mottled, and her HR decreases to 120 bpm between bouts of pain. A NG tube has been inserted and is draining bile. The surgeon arrives, and after a brief examination, he obtains consent from Mrs. Hart for a laparotomy in the event the barium enema does not reduce the intussusception. Julia is transferred to the fluoroscopy suite. The barium enema was successful, and Julia is admitted for overnight observation. Julia's vital signs are all within normal range; however, given her lethargic state one could argue that her HR is tachycardic. According to the AHA (2016), the normal HR range for a child age 1 - 2 yrs is 98-140 bpm (awake) or 80-120 bpm (asleep). Keep in mind that these ranges are a guide only, and vital signs must be considered in the context of each clinical presentation. The correct answer is (b). Bile emesis indicates a possibility that there may be a bowel obstruction distal to the site where the bile duct empties into the duodenum. Paradoxical irritability (a) and intermittent abdominal pain (c) may indicate serious illness in children and should be carefully explored, but they have many causes and are not specific to a surgical abdomen. A lack of fever (d) is not as clear an indicator as bile emesis, but when combined with signs and symptoms of abdominal pain, it can be suggestive of a surgical emergency. The first three (3) priorities are reassessing ABCDEs (e), administering supplemental O2 via any means tolerated (f), and establishing IV access (b). The priorities in any emergently ill patient are always the same: assess and reassess ABCDEs for potentially life-threatening problems, and then intervene to correct those problems. O2 and an IV are indicated due to vomiting and loss of circulating volume. Analgesics, such as Morphine, (a) are used judiciously in children, and may not be given until the cause of pain is identified (with relative certainty), as they may mask symptoms and delay diagnosis and definitive treatment. If extreme lethargy persists, a CT of the head may be ordered; however, cardiopulmonary stabilization is the priority at this time. Obtaining laboratory and ABG studies (c and d) are secondary to assessing and maintaining the ABCDEs. These assessment findings indicate deterioration in Julia's status, related to volume loss and altered tissue perfusion. This is supported by: a weaker radial pulse, cool feet, delayed capillary refill, and tachycardia. Your immediate interventions should be directed toward improving perfusion, which includes ensuring adequate oxygenation and circulating fluid volume (IV fluid bolus[es]). Pediatric fluid boluses of isotonic crystalloid solution are based on weight (20 ml/kg) and infused over 5-20 mins. Julia's weight is 11 kg, thus 220 ml of fluid (b) would be the appropriate volume. Her vital signs and perfusion status should be reassessed following each bolus, and repeated as necessary at 20 ml/kg. All answers are correct. External blood loss would cause an absolute decrease in circulating blood volume, as would occult bleeding and vomiting. Fluid shifting into the interstitial space would also decrease circulating blood volume, as it moves from the intravascular to extravascular space. Correct answers are (a) and (c). Intussusception is treated with a barium enema which creates enough pressure to reduce the affected segment in most cases (may also be used as a diagnostic tool, but abdominal ultrasound is the 'gold standard' today). Intussusception often presents with stools that resemble currant jelly (bloody mucous). Surgical intervention (b) is not always required if intussusception is resolved by barium enema reduction. A malrotation of the bowel (d) is a volvulus.

Case Studies

Table 2.3. Pediatric Glasgow Coma Scale (GCS) Score Best Eye Opening Child (≥ 1 yr) Infant (< 1 yr) 4 Spontaneously Spontaneously 3 To verbal command To shout, speech 2 To pain To pain 1 No response No response Best Motor Response Child (≥ 1 yr) Infant (< 1yr) 6 Obeys Spontaneous movements 5 Localizes pain Withdraws to touch 4 Flexion - withdrawal Flexion - appropriate withdrawal 3 Flexion - abnormal (decorticate rigidity) Flexion - abnormal (decorticate rigidity) 2 Extension - decerebrate rigidity Extension - decerebrate rigidity 1 No response No response Best Verbal Response > 5 years 2-5 years 0-2 years 5 Oriented, converses Appropriate words/ phrases Cries appropriately, smiles, coos 4 Disoriented, converses Inappropriate words Cries and is consolable 3 Inappropriate words Persistent crying, screaming Persistent inappropriate crying, screaming 2 Incomprehensible sounds Moans, grunts to pain Moans, grunts to pain 1 No response No response No response * If patient is intubated, unconscious, or preverbal, the most important part of this scale is motor response (AHA, 2016, p. 57; ENA, 2014, p. 75) Table 2.4. End-organ Perfusion Assessment Integument Capillary refill Comparative pulses Skin temperature and colour (Central/Peripheral perfusion status) Capillary refill is normally ≤ 2 seconds, prolongation is a sensitive and early indicator. Comparing central and peripheral pulses gives a qualitative assessment of perfusion status. Compromised perfusion may produce colour or temperature variation in extremities. Mottling, pallor, and peripheral cyanosis indicate poor perfusion. Gray or ashen colour indicates vasoconstriction and poor perfusion. Brain Alteration in LOC (Hypoxia) Lack of adequate perfusion to the brain produces changes in LOC that are valuable indicators of cerebral hypoxia or ischemia. These changes may include: Parent's statement that the child is "not acting normally." Child's failure to recognize caretakers. Confusion, irritability, lack of consolability Kidneys Urine output (Renal perfusion) Infants/Young Children: 1.5 - 2 ml/kg/hr Older children/Adolescents: 1 ml/kg/hr (AHA, 2016) Diapers: 1 g increase in diaper weight = 1 ml of urine A toddler should have a wet diaper every 4 hours, infant every 2 hours.

Case Studies

The tables below provide information that will assist you as you progress through this module. You can also download a .pdf of the tables. Table 3.3. Normal Pediatric Vital Signs Age Normal Heart Rates (beats/minute) Normal Respiratory Rates (breaths/minute) Normal Blood Pressure (mmHg) Heart Rate (Awake) Heart Rate (Asleep) Systolic Diastolic Neonate 100-205 90-160 30-60 67-84 35-53 Infant (1-12 mths) 100-180 90-160 30-53 72-104 37-56 Toddler (1-2 yrs) 98-140 80-120 22-37 86-106 42-63 Preschool (3-5 yrs) 80-120 65-100 20-28 89-112 46-72 School-age (6-11 yrs) 75-118 58-90 18-25 97-115 57-76 Preadolescent (10-11 yrs) 102-120 61-80 Adolescent (12-15 yrs) 60-100 50-90 12-20 110-131 64-83 (AHA, 2016) Table 3.4. Pediatric Glasgow Coma Scale (GCS) Score Best Eye Opening Child (≥ 1 yr) Infant (< 1 yr) 4 Spontaneously Spontaneously 3 To verbal command To shout, speech 2 To pain To pain 1 No response No response Best Motor Response Child (≥ 1 yr) Infant (< 1 yr) 6 Obeys Spontaneous movements 5 Localizes pain Withdraws to touch 4 Flexion - withdrawal Flexion - appropriate withdrawal 3 Flexion - abnormal (decorticate rigidity) Flexion - abnormal (decorticate rigidity) 2 Extension - decerebrate rigidity Extension - decerebrate rigidity 1 No response No response Best Verbal Response > 5 years 2-5 years 0-2 years 5 Oriented, converses Appropriate words/ phrases Cries appropriately, smiles, coos 4 Disoriented, converses Inappropriate words Cries and is consolable 3 Inappropriate words Persistent crying, screaming Persistent inappropriate crying, screaming 2 Incomprehensible sounds Moans, grunts to pain Moans, grunts to pain 1 No response No response No response * If patient is intubated, unconscious, or preverbal, the most important part of this scale is motor response (AHA, 2016, p. 57; ENA, 2014, p. 75) Table 3.5. Indicators of Illness in Infants Indicator Normal Moderately ill Severely ill Colour of mucous membranes, nail beds, palms of hands, soles of feet Pink Pale Cyanotic Hydration status Moist mucous membranes Good skin turgor Flat fontanel Light, amber-coloured urine Sticky, pasty mucous membranes Slightly doughy skin turgor Fontanel slightly sunken Dark-coloured urine Dry mucous membranes Tenting skin turgor Very sunken fontanel and eyes, anuria Response to stimulation Arouses easily, then stays awake and alert Arouses with repeated gentle stimulation, then falls back to sleep quickly if unstimulated Arouses only with noxious stimulation or does not arouse at all Behaviour Unchanged Fussy, but consolable Irritable, inconsolable if awake Cry Unchanged Whimpers, sobs, or whines High-pitched and screeching or weak and moaning

Select Genitourinary Emergencies

Urinary Tract Infection Urinary tract-related complaints are some of the most common pediatric presentations in the ED, with neonates (> premature), young children, and immunocompromised children among those most at risk. Febrile (> 39°C) infants between the ages of 2-36 months should be investigated for UTI if no other source of infection is observed (Robinson et al., 2020). Keep in mind that neonates (< 30 days of age) with fever higher than 38°C must be investigated with a different approach (i.e., full diagnostic or septic work-up), as other infectious diseases may be present (e.g., meningitis) (Hamilton et al., 2020). UTI is most often caused by bacteria from the gut, which results in inflammation of the urinary epithelium. In a healthy host defense state, bacteria are cleared via micturition (voiding) and washed out of the bladder. Acidic and concentrated urine, proteins, and urinary secretions also provide a bactericidal effect. Urine is also prevented from re-entering the bladder during micturition by contraction of the ureterovesical junction (where ureters meet bladder). In females, mucous-secreting glands are located in the distal 2/3 of the urethra; they trap bacteria before it ascends into the bladder. In males, several factors aid in host defense against UTI: length of the urethra, secretions from the prostate and accessory glands, and the urethral sphincter mechanism. Virulence of the pathogen significantly affects the development of a UTI. Some pathogens have adhesins or fimbriae that allow them to adhere to the uroepithelium or blood group antigen, rendering them safe from micturition and enabling them to 'hang around' for long periods of time (i.e. E. coli). UTIs are classified according to the site of origin or related complicating factors: Cystitis: bladder inflammation (most common site); typically postpubertal girls E. coli, Staphylococcus (bacteria, most common), Candida (fungus, less common) mild: hyperemic mucosa moderate: bleeding, pus/exudate formation, epithelial sloughing, ulcers severe: necrosis (rare) Pyelonephritis: upper urinary tract inflammation (ureter, renal pelvis, interstitium; not necessarily isolated to the kidney) usually due to ascending bladder pathogens (from urethra) usually focal inflammatory process leading to WBC infiltration in renal medulla, which results in renal edema and purulence severe: inflammation infiltrates cortex, leads to abscess and necrosis Uncomplicated UTI: occurs in a normal functioning urinary system Complicated UTI: occurs in urinary system with defects, or altered host defense (McCance & Huether, 2019, p. 1254) Diagnosis of UTI is based on the history, clinical presentation, urinalysis, and urine C & S. In more serious cases, such as in patients who are immunocompromised or have renal dysfunction, laboratory studies (i.e. CBC/Diff, BUN, Cr, blood C & S) and renal imaging (i.e. renal and bladder ultrasound (RBUS), voiding cystourethrogram [VCUG]) may be indicated. Note, however, that RBUS is indicated for all children < 2 years of age with first [febrile] UTI in order to assess for the presence of hydronephrosis, vesicoureteral reflux [VUR]), or structural anomalies (Robinson et al., 2020). Note: Obtaining a sterile urine sample can be a challenge in the pediatric population. Urine bag (U-bag) specimens have high rates of contamination (up to 63%), thus are not recommended for urine sample collection. If the child is toilet-trained, a midstream urine sample should be obtained, if possible, otherwise a catheter specimen is likely required. Note: In the ED, urine samples are often analyzed via dipstick urinalysis; this method provides rapid results and is convenient and generally cost-effective. The disadvantages of dipstick urinalysis relate to its qualitative results and time sensitivity. In addition, false-positive and false-negative results are not uncommon; if the results are positive for nitrites, for example, the sample may have been contaminated during collection or the dipstick may have been exposed to the air for an extended period of time prior to use. Gram-ve bacteria produce an enzyme that reduces urine nitrates into nitrites, thus their presence indicates likely infection; however, the absence of nitrites does not rule out infection as some organisms do not convert urine nitrates to nitrites (i.e. staphylococci, streptococci). As a result, some EDs no longer utilize dipsticks due to potentially unnecessary treatment for UTI (i.e. antibiotics). Treatment for both pyelonephritis and cystitis involves antibiotics and increased fluids. A patient with pyelonephritis may require IV fluids, pain control (i.e. analgesics, anti-spasmodics), and antiemetics. In severe cases, a patient's deteriorating status may warrant admission to hospital, such as in neonatal urosepsis or severe pyelonephritis. The following Table provides information that will assist in differentiating gastroenteritis, appendicitis, and UTI. Table 4.3. Signs and Symptoms of Gastroenteritis, Appendicitis, and UTI Gastroenteritis Appendicitis UTI Pain Diffuse, cramp-like Periumbilical, then localizes to RLQ Flank pain Dysuria (In girls, dysuria is often due to labial or perineal irritation rather than infection per se; as they learn self-care and proper wiping from 'front-to-back', the labial/perineal areas often become irritated and inflamed resulting in dysuria, hesitancy) Vomiting Coincides with pain Usually follows pain Minimal Diarrhea Large volumes Small amounts Minimal Fever Varies Low-grade (early) Cystitis: typically afebrile Pyelonephritis: febrile (may be high) Course Intermittently feels better Progressively worsens Slow progression Physical exam Soft abdomen, hyperactive bowel sounds Positive Rovsing's sign Peritoneal signs (if ruptured) Costovertebral angle (CVA) tenderness, suprapubic pain Cystitis: > localized symptoms Pyelonephritis: systemic manifestations Testicular Torsion "Testicular torsion is one of the few true urological emergencies" (Quallich, 2013, p. 355), thus any male patient complaining of acute scrotal pain should be evaluated accordingly. It most commonly occurs in adolescence, between the ages of 12-18 years, during a period of rapid growth. Fifty percent of torsions occur in sleep due to contraction of the cremaster muscle; however, athletic events, strenuous activity, or trauma immediately prior to the onset of pain can also produce a testicular torsion. Congenital abnormality, an undescended testicle, cold weather, and sexual activity may also cause a testicular torsion. Pathophysiology Testicular torsion occurs due to twisting of the spermatic cord, which contains the blood vessels supplying the testes. Untreated strangulation of testicular blood supply results in infertility, as well as ischemia and necrosis within 24 hours. The following Table outlines the clinical signs/symptoms, as well as diagnosis and treatment modalities, for testicular torsion. Table 4.4. Signs/Symptoms, Diagnosis, and Treatment of Testicular Torsion Testicular Torsion Onset Sudden; often during sleep, may also occur during/ immediately prior to physical activity Pain Character Severe, unilateral scrotal pain; scrotal edema, erythema Ureteral Discharge No Other Signs/Symptoms History of similar episodes that resolved spontaneously Firm scrotal mass Elevated testicle (unilateral absence of cremaster reflex) Nausea, vomiting No urinary signs/symptoms Scrotal Elevation May cause intense pain Diagnosis Doppler ultrasound Treatment Urgent urology consult, bedside detorsion If unsuccessful, surgical exploration with orchidopexy Nursing Care & Interventions Analgesics Antiemetics Pre-operative teaching & preparation Psychosocial support

Gastrointestinal (GI) emergencies

one of the largest categories of diseases in the pediatric population. Emergency nurses must be prepared to care for the physical and psychosocial needs of these children and their families, as abdominal pain can be a very frightening experience. Obtaining a thorough history is imperative. General treatment priorities regarding GI emergencies are outlined below: Assessment and stabilization of ABCDEs Secure airway Support ventilation to avoid or correct hypoxia Monitor cardiovascular status and initiate support Recognize and prevent progressive neurologic deterioration Give supplemental O2 to avoid or treat hypoxia Initiate IV therapy for correction of hydration status, electrolyte imbalances, and medication administration Assessment and management of increasing ICP Administer Naloxone (0.1-0.8 mg/kg), in the case of opioid toxicity Administer glucose (Dextrose 25%) in hypoglycemia Establish and treat the underlying cause of neurologic dysfunction, such as administration of antidotes and charcoal for toxic ingestions A patient with an acute abdominal condition potentially requiring surgical intervention must remain NPO until assessed. Generally speaking, a patient experiencing abdominal pain with palpation or movement should be treated as requiring surgical intervention until the need for surgery is ruled out. The presence of bile emesis indicates the possibility of a bowel obstruction distal to the site where the bile duct empties into the duodenum, which may be a surgical emergency. Paradoxical irritability and intermittent abdominal pain may indicate serious illness in children and should be carefully explored, but the causes are many and nonspecific to a surgical abdomen. Disorders that may cause abdominal pain in children include toxic ingestion, appendicitis, urinary tract infection (UTI), intussusception, strangulated hernia, and testicular torsion. This module focuses on the nursing care of children with toxic ingestions, gastroenteritis, and the more common nontraumatic surgical emergencies (appendicitis, intussusception, volvulus, and pyloric stenosis).

The Preschooler in the Emergency Department

Age Developmental Milestones Common Fears Approach Health Promotion 4 years 5 years Understands 3-part related directions Asks many questions Speaks clearly enough to be understood most of the time Undoes buttons and zippers Is toilet-trained Throws ball overhand, uses scissors Has well-defined concept of external body (draws people with 2-4 parts) Goes down stairs alternating feet Likes to imitate other children Looks for adult approval Continues language, motor development Draws people with bodies Skips, runs fast, stands on one foot Separates more easily from parent Usually can complete washroom routines without help Apologizes for wrongful action Listens while being spoken to Is very curious Pretend play peaks Has vivid imagination, invents invisible friends or imagines monsters are hiding under beds. May need light left on if able to fall asleep. Fears being lost or abandoned, adults who look or act mean (even if that is not their intent). Vomiting, lacerations, and blood tests are seen as threats to body integrity. Pain Response Nonverbal Goal-directed aggression; active physical resistance (pushes health care workers away, hides); regression. Verbal Attempts to postpone, pretends to be another person; uses aggressive statements ("I hate you"; "Go away"). Identifies location of intensity of the pain ("My owie hurts a lot!").

General Management of Respiratory Emergencies

Alterations in normal breathing patterns can be stressful for parents as well as health care providers. Interventions include oxygenation, reducing respiratory effort, and hydration, as well as providing reassurance and explanations. Oxygen (O2) may be administered via several delivery systems, including nasal cannula, simple mask, partial rebreather mask, non-rebreather (NRB) mask, O2 hood, or O2 tent. Flow rates and percentage of O2 delivered vary with each device. The table below provides a general guideline regarding delivery of O2 according to each device. If a child will not tolerate wearing a mask, the following nursing strategies may be attempted: Hold the O2 tubing near the nose and mouth - the "blow-by" technique (Note: This technique is insufficient for a critically ill child). Try role-playing. For example, a preschool child may cooperate with astronaut role play and tolerate a "space mask." Cut a hole the size of the O2 tubing in the bottom of a styrofoam cup. Thread the O2 tubing through the hole, and have the child breathe from the cup. Try using a nasal cannula if it provides a sufficient flow rate. Do not exceed a flow rate of 2-4 L/min to avoid irritation of the mucous membranes. Table 1.1. Oxygen Rates and Percentages According to Device Type of Device Flow Rate (should be humidified) FiO2* (depends on flow rate) Nasal cannula .25 - 4 L/min** .22 - .60 Simple oxygen mask 5 - 10 L/min*** .40 - .60 Partial rebreather mask 10 - 12 L/min .50 - .60 Non-rebreather mask 10 - 15 L/min Up to .95 Oxygen hood (infants < 1 yr) 10 - 15 L/min nearly 1.00 Oxygen tent Greater than 10 L/min Greater than .50 Tracheostomy collar/mask 10 - 15 L/min .24 - nearly 1.00 If the nurse suspects a communicable disease, any child who presents with a cough and fever must be isolated with respiratory precautions in order to prevent transmission to other children. Nurses and families must: Use good hand-washing techniques Wear masks and gowns Keep the child's door closed In addition, hospital personnel must: Assign a private room for the child Place a sign outside the door that states "Respiratory Isolation" Determine if negative room pressure is warranted (e.g., airborne diseases)

Musko emergencies

Although all children can suffer musculoskeletal (MSK) injuries, preadolescents and adolescents are at higher risk due to their risk-taking activities and greater body mass. Children's immature cognitive development, increasing mobility, and underdeveloped motor skills can also predispose them to accidents during play and participation in sports. The epiphyseal growth plate is a thin layer of cartilage located between the metaphysis (area where cartilage is converted to bone) and the epiphysis (area of new cartilage) at the end of long bones. It is a vascular area that controls bone growth. The growing cells of the epiphysis are sensitive to nutrition and hormonal changes. Bone lengthening is governed by growth hormones secreted from the anterior pituitary gland. Bone growth is typically completed by the age of 14 years in boys, and 16 years in girls; however, maturation and bone shaping continue until approximately 21 years of age. The epiphyseal growth plate is the weakest part of a long bone, and is therefore prone to injury. Damage to this area may disrupt, stop, or cause uneven bone growth if the child has not reached skeletal maturity. Injuries to this area should be evaluated as soon as possible. Salter I and II injuries have the best prognosis for normal growth (see Figure 40.10 in Cerepani, 2020, p. 485). Common types of childhood fractures include the following: Transverse - fracture line is at a right angle to the shaft of the bone; cause is often a sharp, direct blow or stress on the bone during prolonged activity. Oblique - fracture line is diagonal across the bone, usually due to twisting force. Spiral - fracture line is circular and twists around the bone shaft, usually due to twisting force. Greenstick - incomplete fracture, portion of periosteum remains intact (one side of the bone breaks, while the other side bends); cause is usually compression force. An understanding of growth and development is important in the care of children with MSK injuries. Nurses play a vital role in educating children and caregivers regarding preventive and safety measures. The following strategies may be communicated to children and caregivers to prevent sports injuries: Be aware of risk factors. Ensure children have knowledgeable coaching and supervision. Ensure children wear protective equipment. Emphasize the importance of adequate conditioning, warm-ups, and cool-downs. Ensure playing conditions are safe.

Circulation

An infant's myocardium is less compliant (fibres are shorter and less elastic), has less contractile mass, and has a limited stroke volume (1.5 ml/kg/beat/minute) compared to 75-90 ml/beat/minute in adults. An infant is more dependent upon increasing heart rate (HR) in order to increase cardiac output. The autonomic nervous system (ANS) of an infant is not fully developed; therefore, infants are particularly sensitive to parasympathetic stimulation. Relative to their weight, infants have a higher cardiac output (200 ml/kg/minute) than adults (100 ml/kg/minute); however, they have limited ability to increase their cardiac output when required. The circulating blood volumes of infants (90 ml/kg) and children (80 ml/kg) are less than those of adults (70 ml/kg); therefore, the same amount of blood loss has a proportionately higher impact on a child. Children are able to compensate and maintain a cardiac output until compensatory mechanisms are exhausted; therefore, they may appear to be well for a longer time than adults. In the first few months of life, infants are less able to concentrate urine and have difficulty compensating for dehydration or hypovolemia. Normal urine output for infants and young children is 1.5 - 2 ml/kg/hour; older children and adolescents is 1 ml/kg/hour; and an adult is 0.5 ml/kg/hour. A greater percentage of the infant's total body weight is water. Daily water turnover involves more than ½ of the extracellular fluid. In the adult, ¹⁄5 of the extracellular fluid is exchanged daily. Loss of water, such as with dehydration, has a greater impact on a pediatric patient.

CIAMPEDS

Complaint. Fever, lethargy, decreased oral intake. Immunization / Isolation. Up-to-date, no known contact with infectious diseases. Allergies. No known allergies. Medications. Tommy takes no regular medications. He received one dose of children's Tylenol approximately 2 hours before arrival. Past medical history / Parents' impression. Tommy was a healthy full-term infant, and his growth and development are normal for his age. He has been healthy, with no hospitalizations and only one URTI at 5 months of age. He sees his pediatrician for regular checkups. His parents are very concerned. Events. Tommy's runny nose started yesterday. Today he has been lethargic and irritable when awake. He has felt warm for about 12 hours, but has not had vomiting or diarrhea. Diet / Diapers. Tommy ate well yesterday, with normal fluid intake. He ate cereal this morning and a few bites of banana in the early afternoon, but has taken only a few sips of apple juice since approximately noon. His last wet diaper was in the early afternoon. Symptoms. Flushed, febrile, lethargic.

Case Study 4

Ten-year-old Robbie Blair is brought to the ED with lower abdominal pain. He is diagnosed with appendicitis. Vital signs are stable as an IV is established for analgesic and antibiotics. Preparation for the operating room (OR) and hospital admission is started. How can you ascertain if the pain medication was effective? (Choose any that apply.) a. Observe for pain behaviour b. Draw a blood sample to check for therapeutic pain medication doses c. Ask if the pain has decreased d. Use a pain scale Suggest at least two (2) non-pharmacological techniques to help Robbie cope with pain. Identify at least two (2) major concerns school-age children have about admission to the hospital. How would you respond to them? The correct answers are (a), (c), and (d). Observation, discussion with the child, and administration of the pain tool would all be valid methods for assessment. At this age, children can generally tell you if the pain is decreasing and use a rating scale. Observe Robbie for pain responses, such as flinching or avoiding movement. School-age children have already developed several techniques for coping with pain. You can help them by encouraging the use of their own techniques and teaching additional methods, such as the following: Distraction with relaxation exercises, focused breathing, blowing, soft touch or talk, music or videos, storytelling, favourite sports, and guided imagery. This also helps reduce muscle tension. Positive self-talk, such as "I can make it." Help the child prepare by providing sensory and procedural information. School-age children fear death, pain, the unknown, and separation from their parents due to hospitalization. It is important to reduce anxiety with reassurance, comfort, and knowledge. Knowing what to expect and having some familiarity (e.g., clothes, toys, nurses, etc.) will build trust and help the child and family relax.

Disability

The Babinski reflex is normally present until a child starts to walk; therefore, the presence of this reflex is not always an abnormal finding, as it would be in an adult. The anterior fontanel generally remains open until about 18-24 months of age; the posterior fontanel typically closes by 3 months of age. Palpation of fontanels provides additional information regarding hydration. The infant's head can expand to accommodate gradual increases in intracranial pressure (ICP) prior to suture fusion. Recovery from a brain injury is unpredictable in children, since the brain continues to grow over the first years of life. The brain is 25% of its mature adult weight at birth, 75% by 2½ years, and 90% by 6 years of age. As indicated in the previous section, infants have an immature ANS. This not only makes infants more sensitive to parasympathetic stimulation, but also makes it difficult for infants to vasoconstrict to conserve body heat. An infant with normal neurologic function will flex all limbs bilaterally. Asymmetrical flexion of extremities is an indication of abnormal neurological functioning. Figure 1.5. Normal infant posture. Reprinted from The Newborn Examination, by K. Montgomery-Reagan, 2010 (http://slideplayer.com/slide/4555110/). Cranial nerves, protective reflexes such as the cough and gag reflexes, and immature reflexes are intact from birth. Examples of immature reflexes include the Moro (startle) and suck reflexes. An example of a protective reflex is the gag response. Just as assessment of the protective reflexes in adults provides information about neurological functioning, assessment of protective and immature reflexes in pediatric patients gives information about their neurological status. image06

Airway

The airway of the pediatric patient differs from that of an adult in several ways. Obligatory nose-breathers up to 4 - 6 months of age, thus any nasal blockage can cause significant respiratory distress. The infant's larynx is softer than that of an adult, thus it is possible to kink the larynx if the neck is hyperextended. The tongue is larger in proportion to the oral cavity, and may obstruct the upper airway of an unconscious infant. image01 _________________ Figure 1.1 - Anatomical differences in pediatric airways. Infants have small upper and lower airways (the size of their baby fingers) that are easily blocked by edema or secretions. image02 _______________ Figure 1.2. Airway diameter in neonate and older child. In infants and young children, the cricoid cartilage is the narrowest part of the larynx, whereas in adults the vocal cords are the narrowest portion of the larynx. The narrow cricoid cartilage helps to provide a natural seal if an infant or child requires intubation and mechanical ventilation. image03 _____________ Figure 1.3. Cricoid cartilage in adult and infant. The infant's trachea is shorter than that of an adult, which increases the risk of an endotracheal tube (ETT) migrating into a mainstem bronchus. Pediatric patients have more soft tissue in their airways; therefore, there is a greater proportion of tissue that can become edematous. Infants may have malformations of their airways that may not be detected until later in life. Compared to that of an adult, the child's larynx is positioned higher and more anterior which increases the risk of aspiration. Neck is short and wide, neck muscles/ligaments are weaker (airway cartilage also weaker). By about 8 years of age, children's respiratory anatomy and physiology approximate those of adults.

Pediatrics shock

inadequate tissue perfusion hypovolemic - most common type in children caused by inadequate blood volume from dehydration or hemmorhage cardiogenic - inadequate contractility or myocardial dysfunction distributive - CNS peripheral vascular disconnection: anaphylactic, neurogenic, or septic causes obstructive - caused by a circulator obstruction like a PE or cardiac tamponade compensated shock - early sign is tachycardia as children are not able to increase their SV so they increase the heart rate to increase their cardiac output -another sign is narrowing pulse pressure or an increase in DBP, decreased peripheral perfusion, oliguria and blood shunting to vital organs.RAAS activation, ADH is sent out in response to hypovolemia causing water to be reabsorbed and causing vasocontriction decompensated - hypotension, edema, DIC: petichia, Management of shock: airway - intubate if decreased LOC, O2, CPR if HR <60 Control any bleeding IV/vascular access volume replacement 20ml/kg Bicarb Serial assessments ABCDEFG Labs Capnography


Ensembles d'études connexes

The Renaissance in Quattrocento Italy

View Set

PSCI 2305 Soomo Webtext Chapter 1

View Set

history of law enforcement vocab

View Set

Computer Science 105 Chapter 6 and 7 Study Guide

View Set

Management of Financial Institutions : Intro, Chapter 1, Chapter 2

View Set

Business Administration- Human Relations/People Skills

View Set

Rico - Spanish III - VHL Grammar Review

View Set